Вы находитесь на странице: 1из 66

Solved Problems in Algebra

By: Engr. Marvin Kalngan

About the Author:


Engr. Marvin Kalngan finished his
Bachelor’s Degree in Civil
Engineering at Mountain Province
State Polytechnic College – Tadian
Campus. He has doctorate and
masteral degrees on online schooling
from different parts of the world. As a
professional civil engineer, he has
work experiences in the Philippines,
Thailand, Egypt, Libya, Qatar, Brunie,
Saudi Arabia, and South Korea. Most
are online jobs.
Algebra – Mathematics of Engineering 1
3
Fundamental Concepts of Algebra �−7, − , 0,0. 6� , √5, π, 7.3, √81� that are
4
whole numbers are 0 and √81.
1. Evaluate 10 + 6(x - 3)2 for x = 20.
answer: 0 and √𝟖𝟖𝟖𝟖
Solution:
Replace x with 20 then simplify.
c. Integers. The integers consist of the
10 + 6(x - 3)2
natural numbers,0 , and the negative of the
= 10 + 6(20 - 3)2
natural numbers. The elements of the set
= 10 + 6(172) 3
= 10 + 6(289) �−7, − , 0,0. 6� , √5, π, 7.3, √81� that are
4
= 10 + 1734 integers are -7, 0, and √81.
= 1744
answer: -7, 0, and √𝟖𝟖𝟖𝟖
2. Find the intersection:
{7, 8, 9, 10, 11}⋂{6, 8, 10, 11} d. Rational numbers. All numbers in the set
3
�−7, − , 0,0. 6� , √5, π, 7.3, √81� that can be
Solution: 4
The elements common to {7, 8, 9, 10, 11} expressed as the quotient of integers are
and {6, 8, 10, 11} are {8, 10, 11}. Thus, rational numbers. These include
−7 3 0
{7, 8, 9, 10, 11}⋂{6, 8, 10, 11} −7 �−7 = 1 � , − 4 , 0 �0 = 1�, and
= {8, 10, 11} 9
√81 �√81 = 1�. Furthermore, all numbers
3. Find the union: in the set that are terminating or repeating
{7, 8, 9, 10, 11}⋃{6, 8, 10, 11, 12} decimals are also rational numbers. These
include 6� and 7.3.
Solution:
𝟑𝟑
To find {7, 8, 9, 10, 11}⋃{6, 8, 10, 11, 12}, �, 𝟕𝟕. 𝟑𝟑, 𝐚𝐚𝐚𝐚𝐚𝐚 √𝟖𝟖𝟖𝟖
answer: −𝟕𝟕, − 𝟒𝟒 , 𝟎𝟎, 𝟎𝟎. 𝟔𝟔
start by listing all the elements from the first
set, namely, 7, 8, 9, 10, and 11. Now list all e. Irrational numbers. The irrational
the elements from the second set that are not numbers in the set
in the first set, namely, 6 and 12. The union 3
is the set consisting of all these elements. �−7, − , 0,0. 6� , √5, π, 7.3, √81� are
4
Thus, √5�√5 ≈ 2.236� and π(π ≈ 3.14). Both
{7, 8, 9, 10, 11}⋃{6, 8, 10, 11, 12} √5 and π are only approximately equal to
= {6, 7, 8, 9, 10, 11, 12} 2.236 and 3.14 respectively. In decimal
form, √5 and π neither terminate nor have
4. Consider the following set of numbers: blocks or repeating digits.
3
�−7, − , 0,0. 6� , √5, π, 7.3, √81�
4
List the numbers in the set that are: answer: √𝟓𝟓 and 𝛑𝛑
a. natural numbers
b. whole numbers f. Real numbers. All the numbers in the
3
c. integers given set �−7, − 4 , 0,0. 6� , √5, π, 7.3, √81�
d. rational numbers
are real numbers.
e. irrational numbers
f. real numbers 𝟑𝟑
�, √𝟓𝟓, 𝛑𝛑, 𝟕𝟕. 𝟑𝟑, √𝟖𝟖𝟖𝟖
answer: −𝟕𝟕, − 𝟒𝟒 , 𝟎𝟎, 𝟎𝟎. 𝟔𝟔
Solution:
a. Natural numbers. The natural numbers are 5. Rewrite each expression without absolute
the numbers used for counting. The only value bars.
natural number in the set
3
�−7, − , 0,0. 6� , √5, π, 7.3, √81� is √81 a. �√3 − 1�
4
because √81 = 9 (9 multiplied by itself, or
Solution:
92, is 81)
Because √3 ≈ 1.7, the number inside the
answer: √𝟖𝟖𝟖𝟖 absolute value bars, √3 − 1, is positive. The
absolute value of a positive number is the
b. Whole numbers. The whole numbers number itself. Thus,
consist of the natural numbers and 0 (zero). �√3 − 1� = √𝟑𝟑 − 𝟏𝟏
The elements of the set
b. |2 − π|

Marvin Kalngan, Civil Engineer, Plumbing Engineer, AutoCAD Expert, College Lecturer
Algebra – Mathematics of Engineering 2
Solution: (6x 4 3 )(5x 2 7 )
y y = 6∙5∙x ∙x ∙y ∙y 4 2 3 7

Because π ≈ 3.14, the number inside the = 30 ∙ x 4+2 ∙ y 3+7


absolute value bars, 2 − π, is negative. The = 𝟑𝟑𝟑𝟑𝐱𝐱 𝟔𝟔 𝐲𝐲 𝟏𝟏𝟏𝟏
absolute value of x when x < 0 is -x. Thus,
|2 − π| = −(2 − π) = 𝛑𝛑 − 𝟐𝟐 12. Divide each expression using the
quotient rule.
|x|
6. x
if x < 0
(−2)7
a. (−2)4
Solution:
If x < 0, then |x| = -x. Thus, Solution:
|x| −x (−2)7
x
= x = −𝟏𝟏 = (−2)7−4 = (−2)3 = −𝟖𝟖
(−2)4

7. Find the distance between -5 and 3 on the 30x12 y9


real number line. b. 5x3 y7

Solution: Solution:
Because the distance between a and b is 30x12 y9 30 x12 y9
= ∙ ∙ y7 = 6 ∙ x12−3 ∙ y 9−7
given by |a - b|, the distance between -5 and 5x3 y7 5 x3
3 is = 𝟔𝟔𝐱𝐱 𝟗𝟗 𝐲𝐲 𝟐𝟐
|-5 - 3| = |-8| = 8
13. Use the negative-exponent rule to write
8. Find the distance between -4 and 5 on the each expression with a positive exponent.
real number line. Simplify, if possible.

Solution: a. 9−2
Because the distance between a and b is
given by |a - b|, the distance between -4 and Solution:
1 𝟏𝟏
5 is 9−2 = 92 = 𝟖𝟖𝟖𝟖
|-4 - 5| = |-9| = 9
b. (−2)−5
9. Simplify: 6(2x2 + 4x) + 10(4x2 + 3x)
Solution:
Solution: 1 1 𝟏𝟏
Use the distributive property to remove the (−2)−5 = 5
= = −
(−2) −32 𝟑𝟑𝟑𝟑
parenthesis, then combine like terms.
6(2x2 + 4x) + 10(4x2 + 3x) 1
c. 6−2
= 12x2 + 24x + 40x2 + 30x
= 52x2 + 54x
Solution:
1
10. Simplify: 8x + 2[5 – (x - 3)] 6−2
= 62 = 𝟑𝟑𝟑𝟑

Solution: d. 7x −5 y 2
Use the distributive property to remove the
parenthesis, then combine like terms. Work Solution:
in the innermost parenthesis first. 1 𝟕𝟕𝐲𝐲 𝟐𝟐
7x −5 y 2 = 7 ∙ x5 ∙ y 2 =
8x + 2[5 – (x - 3)] 𝐱𝐱 𝟓𝟓
= 8x + 2[5 – x + 3]
= 8x + 2(8 - x) 14. Simplify each expression using the
= 8x + 16 – 2x power rule.
= 6x + 16
a. (23 )2
11. Multiply each expression using the
product rule. Solution:
(23 )2 = 23∙2 = 26 = 𝟔𝟔𝟔𝟔
a. 22 ∙ 23
b. (y 5 )−3
Solution:
22 ∙ 23 = 22+3 = 25 = 𝟑𝟑𝟑𝟑 Solution:
𝟏𝟏
(y 5 )−3 = y 5(−3) = y −15 = 𝟏𝟏𝟏𝟏
𝐲𝐲
b. (6x 4 y 3 )(5x 2 y 7 )
Solution: c. (b−4 )−2
Marvin Kalngan, Civil Engineer, Plumbing Engineer, AutoCAD Expert, College Lecturer
Algebra – Mathematics of Engineering 3
Solution:
Solution: Raise the numerator and denominator to the
(b−4 )−2 = b (−4)(−2) = 𝐛𝐛𝟖𝟖 -3 power then simplify.
−3 −3
4x2 �4−3 ��x2 � �4−3 ��x2∙(−3) �
� � = =
15. Simplify. (−2y 2 )4 y y−3 y−3
�4 −3 ��x−6 � y3 𝐲𝐲 𝟑𝟑
= y−3
= (43 )(x6) = 𝟔𝟔𝟔𝟔𝐱𝐱𝟔𝟔
Solution:
(−2y 2 )4 = (−2)4 (y 2 )4 = (−2)4 (y 2∙4 )
18. Write each number in decimal notation.
= 𝟏𝟏𝟏𝟏𝐲𝐲 𝟖𝟖
a. 6.2 × 107
16. Simplify by raising each quotient to the
given power.
Solution:
3 4
We use the exponent on the 10 to determine
a. �− x� how far to move the decimal point and in
which direction. The exponent is positive so
Solution: we move the decimal point to the right.
3 4 (−3)4 (−3)(−3)(−3)(−3) 𝟖𝟖𝟖𝟖 Move the decimal point 7 places to the right.
�− x� = x4
= = 6.2 × 107 = 𝟔𝟔𝟔𝟔, 𝟎𝟎𝟎𝟎𝟎𝟎, 𝟎𝟎𝟎𝟎𝟎𝟎
x4 𝐱𝐱 𝟒𝟒

3
x2 b. −6.2 × 107
b. � 4 �
Solution:
Solution: We use the exponent on the 10 to determine
3 3
x2 �x2 � x2(3) 𝐱𝐱 𝟔𝟔 how far to move the decimal point and in
� � = = = 𝟔𝟔𝟔𝟔
4 43 64 which direction. The exponent is positive so
we move the decimal point to the right.
17. Simplify each exponential expression. Move the decimal point 7 places to the right.
−6.2 × 107 = −𝟔𝟔𝟐𝟐, 𝟎𝟎𝟎𝟎𝟎𝟎, 𝟎𝟎𝟎𝟎𝟎𝟎
a. (−3x 4 y 5 )3
c. 2.015 × 10−3
Solution:
Raise each factor inside the parentheses to Solution:
the third power, then simplify. We use the exponent on the 10 to determine
(−3x 4 y 5 )3 = (−3)3 (x 4 )3 (y 5 )3 how far to move the decimal point and in
= (−3)3 (x 4∙3 )(y 5∙3 ) which direction. The exponent is negative so
= −𝟐𝟐𝟐𝟐𝐱𝐱 𝟏𝟏𝟏𝟏 𝐲𝐲 𝟏𝟏𝟏𝟏 we move the decimal point to the left. Move
the decimal point 3 places to the left.
b. (−7xy 5 )(−2x 5 y 6 ) 2.015 × 10−3 = 𝟎𝟎. 𝟎𝟎𝟎𝟎𝟎𝟎𝟎𝟎𝟎𝟎𝟎𝟎

Solution: d. −2.015 × 10−3


Group factors with the same base then
simplify. Solution:
(−7xy 5 )(−2x 5 y 6 ) = (−7)(−2)xx 5 y 5 y 6 We use the exponent on the 10 to determine
= 14x1+5 y 5+6 how far to move the decimal point and in
= 𝟏𝟏𝟏𝟏𝐱𝐱 𝟔𝟔 𝐲𝐲 𝟏𝟏𝟏𝟏 which direction. The exponent is negative so
we move the decimal point to the left. Move
−35x2 y4 the decimal point 3 places to the left.
c. 5x6 y−8
−2.015 × 10−3 = −𝟎𝟎. 𝟎𝟎𝟎𝟎𝟎𝟎𝟎𝟎𝟎𝟎𝟎𝟎
Solution: 19. Write each number in scientific notation.
Group factors with the same base then
simplify.
−35x2 y4 −35 x2 y4
a. 34,970,000,000,000
=� � �x6� �y−8 �
5x6 y−8 5
= −7x 2−6 y 4−(−8) Solution:
= −7x −4 y12 Move the decimal point to get a number
−𝟕𝟕𝐲𝐲 𝟏𝟏𝟏𝟏 whose absolute value is between 1 and 10.
= Move the decimal point 13 places to the left.
𝐱𝐱 𝟒𝟒
34,970,000,000,000 = 3.497 × 1013
−3
4x2
d. � � b. -34,970,000,000,000
y
Solution:

Marvin Kalngan, Civil Engineer, Plumbing Engineer, AutoCAD Expert, College Lecturer
Algebra – Mathematics of Engineering 4
Move the decimal point to get a number b. √6x ∙ √3x = √6x ∙ 3x = √18x 2
whose absolute value is between 1 and 10. = √9x 2 ∙ 2 = √9x 2 √2 = 𝟑𝟑𝟑𝟑√𝟐𝟐
Move the decimal point 13 places to the left.
-34,970,000,000,000 = -3.497 × 1013 √48x3
100
23. Simplify each. � 9
and
√6x

c. 0.0000000000802
100 √100 𝟏𝟏𝟏𝟏
a. � 9
= = 𝟑𝟑
√9
Solution:
Move the decimal point to get a number
√48x3 48x3
whose absolute value is between 1 and 10. b. =� = √8x 2 = √4x 2 ∙ 2
√6x 6x
Move the decimal point 11 places to the
right. = 𝟐𝟐𝟐𝟐√𝟐𝟐
0.0000000000802 = 8.02 × 10-11
24. Add or subtract as indicated.
d. -0.0000000000802
a. 9√2 + 4√2 = (9 + 4)√2 = 𝟏𝟏𝟏𝟏√𝟐𝟐
Solution:
Move the decimal point to get a number b. √7x − 8√7x = (1 − 8)√7x = −𝟕𝟕√𝟕𝟕𝟕𝟕
whose absolute value is between 1 and 10.
Move the decimal point 11 places to the c. 7√3 + √12 = 7√3 + √4 ∙ 3
right. = 7√3 + 2√3
-0.0000000000802 = -8.02 × 10-11
= 𝟗𝟗√𝟑𝟑
20. As of June 17, 2020, the population of
the Philippines was approximately d. 4√50x − 6√32x
109,519,465. Express the population in = 4√25 ∙ 2x − 6√16 ∙ 2x
scientific notation. = 4 ∙ 5√2x − 6 ∙ 4√2x
= 20√2x − 24√2x
Solution: = −𝟒𝟒√𝟐𝟐𝟐𝟐
Move the decimal point to get a number
whose absolute value is between 1 and 10. 25. Rationalize the denominator.
Move the decimal point 8 places to the left.
109,519,465 = 1.09519465 × 108 15
a.
√6
21. Perform the indicated computations,
writing the answers in scientific notation. Solution:
If we multiply the numerator and
15
a. (6.1 × 105 )(4 × 10−9 ) denominator of by √6, the denominator
√6
becomes √6 ∙ √6 = √36 = √62 = 6.
Solution: √6
Regroup factors then simplify. Therefore, we multiply by 1. Choosing
√6
(6.1 × 105 )(4 × 10−9 ) for 1.
= (6.1 × 4) × (105 × 10−9 ) 15 √6 15√6 15√6 𝟓𝟓√𝟔𝟔
∙ = = =
= 24.4 × (105−9 ) √6 √6 √36 6 𝟐𝟐
= 24.4 × 10−4 12
= 𝟐𝟐. 𝟒𝟒𝟒𝟒 × 𝟏𝟏𝟏𝟏−𝟑𝟑 (scientific notation) b.
√8

1.8×104
b. Solution:
3×10−2
If we multiply the numerator and
12
Solution: denominator of by √8, the denominator
√8
Regroup factors then simplify. becomes √8 ∙ √8 = √64 = √82 = 8.
1.8×104 1.8 104 4−(−2)
= � 3 � × �10−2 � = 0.6 × 10 Therefore, we multiply by 1. Choosing
√8
3×10−2
√8
= 0.6 × 106 for 1.
= 𝟔𝟔 × 𝟏𝟏𝟏𝟏𝟓𝟓 (scientific notation) 12 √8 12√4∙2 12∙2√2 24√2
∙ = = = = 𝟑𝟑√𝟐𝟐
√8 √8 √64 8 8
22. Simplify each. √500 and √6x ∙ √3x
Alternate solution:
a. √500 = √100 ∙ 5 = √100√5 = 𝟏𝟏𝟏𝟏√𝟓𝟓 The smallest number that will produce the
square root of a perfect square in the

Marvin Kalngan, Civil Engineer, Plumbing Engineer, AutoCAD Expert, College Lecturer
Algebra – Mathematics of Engineering 5
12
denominator of is √2 because
√8 1� 3
b. 125 3 = √125 = 𝟓𝟓
√8 ∙ √2 = √16 = √42 = 4. We multiply
√2
by 1, choosing for 1. 1� 4
√2 c. −16 4 = −� √16� = −𝟐𝟐
12 √2 12√2 12√2
∙ = = = 𝟑𝟑√𝟐𝟐
√8 √2 √16 4 1� 3
d. (−27) 3 = √−27 = −𝟑𝟑
17
26. Rationalize the denominator: 5+√3 −1� 1 1 𝟏𝟏
e. 64 3 = 1 = 3 = 𝟒𝟒
64 �3 √64
Solution:
The conjugate of the denominator is 5 − √3. 30. Simplify:
3
If we multiply the numerator and a. 9 �2
2
denominator by 5 − √3, the simplified b. 27 �3
−3
denominator will not contain a radical. c. 81 �4
5−√3
Therefore, we multiply by 1, choosing 5−√3
Solution:
for 1.
17 5−√3 17�5−√3� 17�5−√3�
The numerator of the fraction exponent is
5+√3 5−√3
∙ = 2 = 25−3
the exponent in radical form. The
52 −�√3�
17�5−√3� 𝟖𝟖𝟖𝟖−𝟏𝟏𝟏𝟏√𝟑𝟑
denominator of the fraction exponent is the
= 22
= index in radical form.
𝟐𝟐𝟐𝟐

3� 3
27. Simplify: a. 9 2 = �√9� = 33 = 𝟐𝟐𝟐𝟐
3
a. √24
4 4 2
b. √8 ∙ √4 b. 27
2�
3
3
= � √27� = 32 = 𝟗𝟗
4 81
c. �16
−3� 1 1 1 𝟏𝟏
c. 81 4 = 3 = 4 3 = 33 = 𝟐𝟐𝟐𝟐
81 �4 � √81�
Solution:
3 3 3 𝟑𝟑
a. √24 = √8 ∙ 3 = √23 ∙ 3 = 𝟐𝟐 √𝟑𝟑 31. Simplify the following.
5
32x �3
4 4 4 4 4 a. 3
b. √8 ∙ √4 = √8 ∙ 4 = √32 = √16 ∙ 2 16x �4
4 𝟒𝟒 1� 3�
= √24 ∙ 2 = 𝟐𝟐 √𝟐𝟐 b. �5x 2 � �7x 4�

4
4 81 4 34 √34 𝟑𝟑 Solution:
c. �16 = �24 = 4 = 𝟐𝟐
√24 Group numerical factors and group variable
factors with the same base, then simplify.
3 3 5 5
28. Subtract: 5√16 − 11 √2 32x �3 32 x �3 5� −3�
a. 3 = �16� � 3 � = 2x 3 4
16x �4 x �4
𝟏𝟏𝟏𝟏�
Solution: = 𝟐𝟐𝐱𝐱 𝟏𝟏𝟏𝟏
3 3 3 3
5√16 − 11 √2 = 5√8 ∙ 2 − 11 √2
3 3 1� 3 1� 3�
= 5 √23 ∙ 2 − 11√2 b. �5x 2 � �7x �4 � =5∙7∙x 2 ∙x 4
3 3
= 2 ∙ 5√2 − 11√2 1� +3�
3 3 = 35x 2 4
= 10 √2 − 11 √2 𝟓𝟓�
𝟑𝟑 = 𝟑𝟑𝟑𝟑𝐱𝐱 𝟒𝟒
= − √𝟐𝟐
9
29. Simplify: 33. Simplify: √x 3
1
a. 81 �2
1 Solution:
b. 125 �3 9 3 1
√x 3 = x �9 = x �3 = √𝐱𝐱
𝟑𝟑
1
c. −16 �4
1
d. (−27) �3 34. Perform the indicated operations and
−1
e. 64 �3 simplify.

Solution: a. (7x3 – 8x2 + 9x - 6) – (2x3 – 6x2 – 3x + 9)


The denominator of the fraction exponent is
the index in radical form. Solution:
Apply the distributive property. Combine
1� like terms. Then, simplify.
a. 81 2 = √81 = 𝟗𝟗
(7x3 – 8x2 + 9x - 6) – (2x3 – 6x2 – 3x + 9)

Marvin Kalngan, Civil Engineer, Plumbing Engineer, AutoCAD Expert, College Lecturer
Algebra – Mathematics of Engineering 6
3 2 3 2
= 7x – 8x + 9x – 6 - 2x + 6x + 3x – 9 Solution:
= 7x3 - 2x3 – 8x2 + 6x2 + 9x + 3x – 6 – 9 Use the square of a binomial sum.
= 5x3 - 2x2 + 12x - 15 (a + b)2 = a2 + 2ab + b2

b. a. (x + 6)2 = x2 + 2(x)(6) + 62
(13x3 + 2x2 – 8x - 6) + (-9x3 + 7x2 – 5x + 3) = x2 + 12x + 36

Solution: b. (7x + 3)2 = (7x)2 + 2(7x)(3) + 32


Apply the distributive property. Combine = 49x2 + 42x + 9
like terms. Then, simplify.
(13x3 + 2x2 – 8x - 6) + (-9x3 + 7x2 – 5x + 3) 39. Find the square of the binomial
= 13x3 + 2x2 – 8x – 6 - 9x3 + 7x2 - 5x + 3 difference.
= 13x3 - 9x3 + 2x2 + 7x2 – 8x - 5x – 6 + 3 a. (y - 9)2
= 4x3 + 9x2 – 13x – 3 b. (6x - 8)2

35. Multiply: (2y + 3)(y2 + 4y + 5) Solution:


Use the square of a binomial sum.
Solution: (a - b)2 = a2 - 2ab + b2
Multiply the trinomial by each term of the
binomial. Apply the distributive property, a. (y - 9)2 = y2 – 2(y)(9) + 92
then simplify. = y2 – 18y + 81
(2y + 3)(y2 + 4y + 5)
= 2y(y2 + 4y + 5) + 3(y2 + 4y + 5) b. (6x - 8)2 = (6x)2 – 2(6x)(8) + 82
= 2y3 + 8y2 + 10y + 3y2 + 12y + 15 = 36x2 – 96x + 64
= 2y3 + 8y2 + 3y2 + 10y + 12y + 15
= 2y3 + 11y2 + 22y + 15 40. Subtract.
(5x3–9x2y+3xy2-4) – (3x3–6x2y–2xy2+3)
36. Multiply: (5x + 6)(4x + 3)
Solution:
Solution: Apply the distributive property and collect
Any two binomials can be quickly like terms.
multiplied using the FOIL method, F (5x3–9x2y+3xy2-4) – (3x3–6x2y–2xy2+3)
represents the product of the first terms in = 5x3–9x2y+3xy2-4-3x3+6x2y+2xy2-3
each binomial, O represents the product of = 5x3-3x3–9x2y+6x2y+3xy2+2xy2-4-3
the outside terms, I represents the product of = 2x3 – 3x2y + 5xy2 – 7
the inside terms, L represents the product of
the last terms in each binomial. 41. Perform the indicated operation.
(5x + 6)(4x + 3) a. (4y + 2x)(6x – 5y)
= 5x(4x + 3) + 6(4x + 3)
= 5x(4x) + 5x(3) + 6(4x) + 6(3) Solution:
F O I L Any two binomials can be quickly
2
= 20x + 15x + 24x + 18 multiplied using the FOIL method, F
= 20x2 + 39x + 18 represents the product of the first terms in
each binomial, O represents the product of
37. Multiply: the outside terms, I represents the product of
a. (5x4 + 6)(5x4 - 6) the inside terms, L represents the product of
b. (4m + 3)(4m - 3) the last terms in each binomial.
(4y + 2x)(6x – 5y)
Solution: = 4y(6x – 5y) + 2x(6x – 5y)
Use the product of the sum and difference of = 4y(6x) + 4y(-5y) + 2x(6x) + 2x(-5y)
two terms. F 0 I L
(a + b)(a - b) = a2 – b2 = 24xy – 20y2 + 12x2 – 10xy
= 24xy – 10xy – 20y2 + 12x2
a. (5x4 + 6)(5x4 - 6) = (5x4)2 - 62 = 25x8 – 36 = 14xy – 20y2 + 12x2

b. (4m + 3)(4m - 3) = (4m)2 - 32 = 16m2 – 9 b. (3a – 7b)2

38. Find the square of the binomial sum. Solution:


a. (x + 6)2 (x - y)2 = x2 + 2(x)(-y) + (-y)2
b. (7x + 3)2
(3a – 7b)2 = (3a)2 + 2(3a)(-7b) + (-7b)2
= 9a2 – 42ab + 49b2

Marvin Kalngan, Civil Engineer, Plumbing Engineer, AutoCAD Expert, College Lecturer
Algebra – Mathematics of Engineering 7
2
45. Factor: 8x – 10x – 3
42. Factor.
a. 18p3 + 27p2 Solution:
Step 1. Find two terms whose product is 8x2.
Solution: 8x2 – 10x – 3 = (8x )(x )
Determine first the greatest common factor 8x2 – 10x – 3 = (4x )(2x )
(GCF), 9 is the greatest number that divides
18 and 27, p2 is the greatest expression that Step 2. Find two last terms whose product is
divides p3 and p2. -3. The possible factorizations are 1(-3) and
The GCF of the two terms is 9p2. -1(3).

18p3 + 27p2 = 9p2(2p) + 9p2(3) Step 3. Try various combination of these


= 9p2(2p + 3) factors. The correct factorization of 8x2 –
10x – 3 is the one in which the sum of the
b. x2(x + 7) + 10(x + 7) Outside and Inside products is equal to -10x.
Here is a list of the possible factorizations:
Solution:
The greatest common factor (GCF) is the Possible Sum of Outside and
common binomial factor (x + 7). Factor out Factorizations of Inside Products
this common factor. 8x2 – 10x – 3 (should equal -10x)
(8x + 1)(x - 3) -24x + x = -23x
x2(x + 7) + 10(x + 7) = (x + 7)(x2 + 10) (8x - 3)(x + 1) 8x – 3x = 5x
(8x - 1)(x + 3) 24x – x = 23x
43. Factor: x3 + 4x2 + 3x + 12 (8x + 3)(x - 1) -8x + 3x = -5x
(4x + 1)(2x - 3) -12x + 2x = -10x
Solution: (4x - 3)(2x + 1) 4x – 6x = -2x
Group terms that have a common factor. (4x - 1)(2x + 3) 12x – 2x = 10x
Then factor out the greatest common factor (4x + 3)(2x - 1) -4x + 6x = 2x
(GCF) from the grouped terms.
x3 + 4x2 + 3x + 12 = (x3 + 4x2) + (3x + 12) Thus, 8x2 – 10x – 3 = (4x + 1)(2x - 3).
= x2(x + 4) + 3(x + 4)
= (x + 4)(x2 + 3) answer: (4x + 1)(2x - 3)
44. Factor: x2 + 6x + 8 46. Factor: 2x2 – 7xy + 3y2
Solution: Solution:
Step 1. Find two terms whose product is x2. Step 1. Find two first terms whose product is
x2 + 6x + 8 = (x )(x ) 2x2.
2x2 – 7xy + 3y2 = (2x )(x )
Step 2. Find two last terms whose product is
8. Step 2. Find two last terms whose product is
factors of 8: 3y2. The possible factorizations are (y)(3y)
8, 1 4, 2 -8, -1 -4, -2 and (-y)(-3y).
Step 3. Try various combinations of these Step 3. Try various combinations of these
factors. The correct factorization of x2 + 6x factors. The correct factorization of 2x2 –
+ 8 is the one in which the sum of the 7xy + 3y2 is the one in which the sum of the
Outside and Inside products is equal to 6x. Outside and Inside products is equal to -7xy.
Here is the list of the possible factorizations. Here is a list of possible factorizations.
Possible Sum of Outside and
Factorizations of Inside Products Possible Sum of Outside and
2
x + 6x + 8 (should equal to 6x) Factorizations of Inside Products
(x + 8)(x + 1) x + 8x = 9x 2x2 – 7xy + 3y2 (should equal -7xy)
(x + 4)(x + 2) 2x + 4x = 6x (2x + y)(x + 3y) 6xy + xy = 7xy
(x - 8)(x - 1) -x – 8x = -9x (2x + 3y)(x + y) 2xy + 3xy = 5xy
(x - 4)(x - 2) -2x – 4x = -6x (2x - y)(x – 3y) -6xy – xy = -7xy
(2x – 3y)(x - y) -2xy – 3xy = -5xy
Thus, x2 + 6x + 8 = (x + 4)(x + 2).
Thus, 2x2 – 7xy + 3y2 = (2x - y)(x – 3y).
answer: (x + 4)(x + 2)
answer: (2x - y)(x – 3y)

Marvin Kalngan, Civil Engineer, Plumbing Engineer, AutoCAD Expert, College Lecturer
Algebra – Mathematics of Engineering 8
47. Factor.
a. k2 – 9 x2 + 3x + 4 cannot be factored further, so the
b. 64s2 – 49 answer is
c. x4 - 81
answer: 4x(x2 + 3x + 4)
Solution:
Use the difference of two squares. 51. Factor: x2 – 25a2 + 8x + 16
a2 – b2 = (a + b)(a - b)
Solution:
a. k2 – 9 = k2 – 32 = (k + 3)(k - 3) Step 1. If there is a common factor, factor
out the GCF. Other than 1 or -1, there is no
b. 64s2 – 49 = 82s2 – 72 = (8s)2 - 72 common factor.
= (8s + 7)(8s - 7)
Step 2. Determine the number of terms and
4 2 2 2 2 2
c. x – 81 = (x ) - 9 = (x + 9)(x - 9) factor accordingly. There are four terms, we
= (x2 + 9)(x2 - 32) try factoring by grouping. It can be shown
= (x2 + 9)(x + 3)(x - 3) that grouping into two groups of two terms
does not result in a common binomial factor.
Let’s try grouping as a difference of squares.
48. Factor the perfect square trinomials. x2 – 25a2 + 8x + 16 = (x2 + 8x + 16) – 25a2
a. q2 + 6q + 9 = (x + 4)2 – (5a)2
= (x + 4 + 5a)(x + 4 – 5a)
Solution:
(a + b)2 = a2 + 2ab + b2 52. Factor and simplify.
−3 1
x(x + 1) �4 + (x + 1) �4
q2 + 6q + 9 = q2 + 2(q)(3) + 32 = (q + 3)2
Solution:
b. 25x2 – 60x + 36 Express each term as the product of the
greatest common factor and its other factor.
Solution: Then, factor out the greatest common factor.
(a - b)2 = a2 – 2ab + b2 −3 1
x(x + 1) �4 + (x + 1) �4
−3 −3 4
25x2 – 60x + 36 = (5x)2 – 2(5x)(6) + 62 = x(x + 1) �4 + (x + 1) �4 (x + 1) �4
−3 4
= (5x - 6)2 = (x + 1) �4 �x + (x + 1) �4 �
−3
= (x + 1) �4 [x + (x + 1)]
49. Factor. −3
a. x3 + 27 = (x + 1) �4 (2x + 1)
𝟐𝟐𝟐𝟐+𝟏𝟏
= 𝟑𝟑�
(𝐱𝐱+𝟏𝟏) 𝟒𝟒
Solution:
Use the sum of two cubes.
53. Simplify the rational expressions.
a3 + b3 = (a + b)(a2 – ab + b2) x3 +x2 x2 +6x+5
a. b. x2 −25
x+1
x3 + 27 = x3 + 33 = (x + 3)(x2 – 3x + 32)
= (x + 3)(x2 – 3x + 9) Solution:
a. Factor the numerator and divide out the
b. 64x3 – 125 common factor.
x3 +x2 x2 (x+1)
= = 𝐱𝐱 𝟐𝟐
Solution: x+1 x+1
Use the difference of two cubes.
a3 - b3 = (a - b)(a2 + ab + b2) b. Factor the numerator and denominator.
Then, divide out the common factor.
x2 +6x+5 (x+1)(x+5) 𝐱𝐱+𝟏𝟏
64x3 – 125 = 43x3 – 53 = (4x)3 - 53 = (x−5)(x+5) = 𝐱𝐱−𝟓𝟓
x2 −25
= (4x - 5)[(4x)2 + (4x)(5) + 52]
= (4x - 5)(16x2 + 20x + 25) x−7 x2 −1
54. Find the product: x−1 ∙ 3x−21
50. Factor: 4x3 + 12x2 + 16x
Solution:
Solution: Factor as many numerators and
If there is a common factor, factor out the denominators as possible. Then, divide
GCF. Since 4x is common to all the three numerators and denominators by common
terms, we factor it out. factors.
4x3 + 12x2 + 16x = 4x(x2 + 3x + 4)

Marvin Kalngan, Civil Engineer, Plumbing Engineer, AutoCAD Expert, College Lecturer
Algebra – Mathematics of Engineering 9
x−7 x2 −1 x−7 x2 −12

x−1 3x−21
= x−1 ∙ 3x−21
x−7 (x+1)(x−1) 𝐱𝐱+𝟏𝟏
Step 2. List the factors of the first
= x−1 ∙ = denominator.
3(x−7) 𝟑𝟑
5, x, x + 3
y2 −2y−8 y−4
55. Find the quotient: y2 −9
÷ y+3 Step 3. Add any unlisted factors from the
second denominator. One factor of x2 + 6x +
Solution: 9 is already in our list. That factor is x + 3.
To find the quotient of two rational However, the other factor of x + 3 is not
expressions, invert the divisor and multiply. listed in step 2. We add a second factor of x
y2 −2y−8 y−4 y2 −2y−8 y+3 + 3 to the list. We have
y2 −9
÷ y+3 = y2 −9

y−4 5, x, x + 3, x + 3
(y−4)(y+2) y+3
= ∙
y2 −32 y−4
(y−4)(y+2) y+3 Step 4. The least common denominator is
= ∙
(y+3)(y−3) y−4 the product of all factors in the final list.
𝐲𝐲+𝟐𝟐 Thus, the LCD is
= 𝐲𝐲−𝟑𝟑
5(x)(x + 3)(x + 3) = 5x(x + 3)2
5w+1 4w−2
56. Find the difference: w2−9 − w2 −9 answer: 5x(x + 3)2
y+2 4
Solution: 59. Subtract: 2y−3 − y+3
5w+1 4w−2 5w+1−(4w−2) 5w+1−4w+2
w2 −9
− w2−9 = 2 −9
= w2 −9
w
w+3 w+3 𝟏𝟏 Solution:
= w2 −32 = (w+3)(w−3) = 𝐰𝐰−𝟑𝟑
Step 1. Find the least common denominator.
Factor each denominator completely.
57. Find the least common denominator 2y – 3 = 1(2y - 3)
(LCD). y + 3 = 1(y + 3)
x+2 4
2x−3
and x+3
Step 2. List the factors of the first
Solution: denominator.
Step 1. Factor each denominator completely. 1, 2y – 3
2x – 3 = 1(2x - 3)
x + 3 = 1(x + 3) Step 3. Add any unlisted factor from the
second denominator. One factor of the
Step 2. List the factors of the first second denominator is already in our list.
denominator. That factor is 1. However, the other factor,
1, 2x – 3 y + 3, is not listed in step 2. We add y + 3 to
the list. We have
Step 3. Add any unlisted factor from the 1, 2y – 3, y + 3
second denominator. One factor of the
second denominator is already in our list. Step 4. The least common denominator is
That factor is 1. However, the other factor, the product of all factors in the final list.
x + 3, is not listed in step 2. We add x + 3 to Thus, the LCD is
the list. We have 1(2y - 3)(y + 3) or (2y - 3)(y + 3)
1, 2x – 3, x + 3
Step 5. Write equivalent expressions with
Step 4. The least common denominator is the LCD as denominators. We must rewrite
the product of all factors in the final list. The each rational expression with a denominator
LCD is of (2y - 3)(y + 3). We do so by multiplying
1(2x - 3)(x + 3) or (2x - 3)(x + 3) both numerator and denominator of each
rational expression by any factor needed to
answer: (2x - 3)(x + 3) convert the expression’s denominator into
the LCD.
y+2 y+3 (y+2)(y+3)
58. Find the least common denominator ∙
2y−3 y+3
= (2y−3)(y+3)
(LCD). 4 2y−3 4(2y−3)
7 9 ∙
y+3 2y−3
= (y+3)(2y−3)
5x2 +15x
and x2+6x+9
y+3 2y−3
Solution: Because y+3 = 1 and 2y−3 = 1, we are not
Step 1. Factor each denominator completely. changing the value of either rational
5x2 + 15x = 5x(x + 3) expression, only its appearance. We are now
x2 + 6x + 9 = (x + 3)(x + 3) ready to perform the indicated subtraction.
Marvin Kalngan, Civil Engineer, Plumbing Engineer, AutoCAD Expert, College Lecturer
Algebra – Mathematics of Engineering 10
1 1 1 1 y 1 y 1
1+y=1+y=1∙y+y=y+y
y+2 4 (y+2)(y+3) 4(2y−3)
− y+3 = − (y+3)(2y−3) y+1
2y−3 (2y−3)(y+3) = y
(y+2)(y+3)−4(2y−3)
= (2y−3)(y+3)
Multiply in the numerator using FOIL and Step 2. Subtract to get a single rational
the distributive property. Then collect like expression in the denominator.
1 1 1 1 y 1 y 1
terms in the numerator. 1−y=1−y=1∙y−y=y−y
y2 +3y+2y+6−8y+12
= (2y−3)(y+3) =
y−1
y
𝐲𝐲 𝟐𝟐 −𝟑𝟑𝟑𝟑+𝟏𝟏𝟏𝟏
= (𝟐𝟐𝟐𝟐−𝟑𝟑)(𝐲𝐲+𝟑𝟑)
Step 3. Perform the division indicated by the
x+3 2 main fraction bar: Invert the divisor and
60. Find the sum. x2 + x2−1 multiply. If possible, simplify.
+x−2

1 y+1
Solution: 1+
y y y+1 y y(y+1) 𝐲𝐲+𝟏𝟏
Step 1. Find the least common denominator 1 = y−1 = y
∙ y−1 = = 𝐲𝐲−𝟏𝟏
1− y(y−1)
y y
(LCD). Start by factoring the denominators.
x2 + x – 2 = (x + 2)(x - 1) 1 1

x2 – 1 = x2 – 12 = (x + 1)(x - 1) 62. Simplify. x+h x
h

The factors of the first denominator are x + Solution:


2 and x – 1. The only factor from the second We will use the method of multiplying each
denominator that is not listed is x + 1. Thus, 1 1
the LCD is of the three terms x+h , x , h by the least
(x + 2)(x - 1)(x + 1) common denominator. The least common
denominator is x(x + h).
1 1 1 1
Step 2. Write equivalent expressions with − � − �x(x+h)
x+h x
x+h x
=
the LCD as denominators. We must rewrite h hx(x+h)
1 1
each rational expression with a denominator ∙x(x+h)− ∙x(x+h)
= x+h x
of (x + 2)(x - 1)(x + 1). We do so by hx(x+h)
x(x+h) x(x+h)
multiplying both the numerator and the −
denominator by any factor/factors needed to = x+h x
hx(x+h)
convert the expression’s denominator into x−(x+h) x−x−h −h
= = hx(x+h) = hx(x+h)
hx(x+h)
the LCD. −1 𝟏𝟏
x+3 x+1 (x+3)(x+1) = x(x+h) = − 𝐱𝐱(𝐱𝐱+𝐡𝐡)

(x+2)(x−1) x+1
= (x+2)(x−1)(x+1)
2 x+2 2(x+2)

(x+1)(x−1) x+2
= (x+1)(x−1)(x+2)
Variables and Expression
x+1 x+2
Because x+1 = 1 and x+2 = 1, we are not
changing the value of either rational 1. Write an algebraic expression for each
expression, only its appearance. We are now verbal expression.
ready to perform the indicated addition.
a. ten more than a number x
x+3 2 x+3 2 answer: x + 10
2
x +x−2
+ 2
x −1
= (x+2)(x−1)
+ (x+1)(x−1)
(x+3)(x+1) 2(x+2) b. the difference of 9 and 4 times a number
= (x+2)(x−1)(x+1) + (x+1)(x−1)(x+2)
(x+3)(x+1)+2(x+2)
n
= (x+2)(x−1)(x+1) answer: 9 – 4n
Multiply in the numerator using FOIL and
the distributive property. Then collect like c. one fourth of the size of the original area a
𝟏𝟏 𝐚𝐚
terms in the numerator. answer: 𝟒𝟒 𝐚𝐚 or 𝟒𝟒
x2 +x+3x+3+2x+4 𝐱𝐱 𝟐𝟐 +𝟔𝟔𝟔𝟔+𝟕𝟕
= (x+2)(x−1)(x+1)
= (𝐱𝐱+𝟐𝟐)(𝐱𝐱−𝟏𝟏)(𝐱𝐱+𝟏𝟏)

1+
1 Order of Operations
y
61. Simplify. 1
1−
y Step 1. Work separately above and below
any fraction bar.
Solution: Step 2. If grouping symbols such as
Step 1. Add to get a single rational parentheses, brackets, or absolute value bars
expression in the numerator. are present, start with the innermost set and
work outward.
Marvin Kalngan, Civil Engineer, Plumbing Engineer, AutoCAD Expert, College Lecturer
Algebra – Mathematics of Engineering 11
Step 3. Evaluate all powers, roots, and Check:
absolute values. Replace x with 8 in the original equation.
Step 4. Multiply or divide in order from left 2x + 4 = 20
to right. 2(8) + 4 = 20
Step 5. Add or subtract in order from left to 16 + 4 = 20
right. 20 = 20 (The statement is true.)

1. Evaluate each expression. Because the check results in a true


statement, we conclude that the solution set
a. 2 × 4 + 3 × 5 of the given equation is {8}.

Solution: 2. Solve and check:


2 × 4 + 3 × 5 = 8 + 15 = 𝟐𝟐𝟐𝟐 4(x - 5) – 27 = 18 – 5(x + 4)

b. 15 ÷ 3 ∙ 5 − 42 Solution:
Simplify the algebraic expression on each
Solution: side, then collect like terms.
15 ÷ 3 ∙ 5 − 42 = 5 ∙ 5 − 16 = 25 − 16 4(x - 5) – 27 = 18 – 5(x + 4)
= 𝟗𝟗 4x – 20 – 27 = 18 – 5x – 20
4x – 47 = -5x – 2
c. [(1 + 3)2 + 4] × 2 Add 5x to both sides.
4x – 47 + 5x = -5x – 2 + 5x
Solution: 9x – 47 = -2
[(1 + 3)2 + 4] × 2 = [(4)2 + 4] × 2 Add 47 to both sides.
= (16 + 4) × 2 9x – 47 = -2
= (20) × 2 9x – 47 + 47 = -2 + 47
= 𝟒𝟒𝟒𝟒 9x = 45
Divide both sides by 9.
9x 45
d. 10 ÷ 5 + 2|3 − 4| 9
= 9
x=5
Solution:
10 ÷ 5 + 2|3 − 4| = 10 ÷ 5 + 2| − 1| Check:
= 10 ÷ 5 + 2 ∙ 1 Substitute 5 for x in the original equation.
=2+2 4(x - 5) – 27 = 18 – 5(x + 4)
= 𝟒𝟒 4(5 - 5) – 27 = 18 – 5(5 + 4)
4(0) – 27 = 18 – 5(9)
2. Evaluate 7 + 5(x - 4)3 for x = 6. -27 = 18 – 45
-27 = -27 (The statement is true.)
Solution:
Replace x with 6 then simplify. The true statement -27 = -27 verifies that the
7 + 5(x - 4)3 solution set is {5}.
= 7 + 5(6 - 4)3
= 7 + 5(23) 3. Solve: 4x − 2x − 5 = 4 + 6x + 3
= 7 + 5(8) Solution:
= 7 + 40 4x − 2x − 5 = 4 + 6x + 3
= 47 2x − 5 = 7 + 6x
−4x = 12
𝐱𝐱 = −𝟑𝟑
Linear Equations, Inequalities, and
Applications 4. Solve: 2(x − 5) + 3x = x + 6

1. Solve and check: 2x + 4 = 20 Solution:


2(x − 5) + 3x = x + 6
Solution: 2x − 10 + 3x = x + 6
2x + 4 = 20 5x − 10 = x + 6
2x + 4 – 4 = 20 – 4 (Subtract 4 from both 4x = 16
sides.) 𝐱𝐱 = 𝟒𝟒
2x = 16 (Simplify.)
2x 16 x+2 x−1
2
= 2 (Divide both sides by 2.) 5. Solve and check: − =4
6 8
x=8
Solution:

Marvin Kalngan, Civil Engineer, Plumbing Engineer, AutoCAD Expert, College Lecturer
Algebra – Mathematics of Engineering 12
4 3 1
The fractional terms have denominators of 6
x2 +4x−12
= x+6 + x−2
and 8. The smallest number that is divisible
Factor the denominator on the left side and
by 6 and 8 is 24. We begin by multiplying
find the LCD of the denominators on the
both sides of the equation by the LCD, 24.
x+2 x−1 right side.
24 � 6 − 8 � = 24(4) 4 3 x−2 1 x+6
= x+6 ∙ x−2 + x−2 ∙ x+6
x+2 x−1 (x+6)(x−2)
24 � 6 � − 24 � 8 � = 96 4 3(x−2) 1(x+6)
(x+6)(x−2)
= (x+6)(x−2) + (x+6)(x−2)
4(x + 2) − 3(x − 1) = 96 4 3(x−2)+(x+6)
4x + 8 – 3x + 3 = 96 (distributive property) (x+6)(x−2)
= (x+6)(x−2)
x + 11 = 96 (collect like terms) The denominator on the left side cancels out
x = 85 with the denominator on the right side. So
4 = 3(x - 2) + (x + 6)
Check: 4 = 3x – 6 + x + 6
Substitute 85 for x in the original equation. 4 = 4x
x+2 x−1
6
− 8 =4 1=x
85+2 85−1
6
− 8
=4
8. Determine whether the equation shown is
87 84
6
− 8 =4 an identity, conditional equation, or an
4 = 4 (The statement is true.) inconsistent equation.
a. x = x + 7
The true statement 4 = 4 verifies that the b. 2y + 3 = 17
solution set is {85}. c. z + 3 = z + 2 + 1

x−4 2x+4 Solution:


6. Solve: 4
+ 8
=5
a. x = x + 7: inconsistent equation
An inconsistent equation is an equation that
Solution: is not true for even one real number.
x−4 2x+4
4
+ 8 =5
Multiply both sides of the equation by 8 to b. 2y + 3 = 17: conditional equation
remove denominators. An equation that is not an identity, but that
x−4 2x+4 is true for at least one real number, is called
� 4 + 8 = 5� 8
a conditional equation.
2(x − 4) + 1(2x + 4) = 40
2x − 8 + 2x + 4 = 40 c. z + 3 = z + 2 + 1: identity
4x − 4 = 40 An equation that is true for all real numbers
4x = 44 for which both sides are defined is called an
𝐱𝐱 = 𝟏𝟏𝟏𝟏 identity.
y 3
7. Solve: y−3 = y−3 + 9 9. A 50 liter mixture of acid and water
contains 10 liter of acid. What is the percent
Solution: of acid in the mixture?
With denominators of y – 3, y – 3, and 1, the
least common denominator (LCD) is y – 3. Solution:
10
Multiply both sides of the equation by y – 3. x = 50 × 100% = 𝟐𝟐𝟐𝟐%
Then simplify.
y
(y − 3) � � = (y − 3) �
3
+ 9� 10. The toll to a bridge costs ₱350.
y−3 y−3
Commuters who use the bridge frequently
y = 3 + 9(y - 3) have the option of purchasing a monthly
y = 3 + 9y – 27 discount pass for ₱1500. With the discount
y = 9y – 24 pass, the toll is reduced to ₱200. For how
24 = 9y – y many bridge crossings per month will the
24 = 8y total monthly cost without the discount pass
3=y be the same as the total monthly cost with
the discount pass?
8. Consider the equations below. Find all
values of x for which y = z. Solution:
4 3 1
y = x2+4x−12 and z = x+6 + x−2 let x = number of bridge crossings per
month
Solution:
Set the expressions that define y and z equal The monthly cost without the discount pass
to each other. is the toll, ₱350, times the number of bridge

Marvin Kalngan, Civil Engineer, Plumbing Engineer, AutoCAD Expert, College Lecturer
Algebra – Mathematics of Engineering 13
crossings per month, x. The monthly cost Solution:
with the discount pass is the cost of the pass, let a = number of apples in the large box
₱1500, plus the toll, ₱200, times the number b = number of bananas in the large box
of bridge crossings per month, x.
350x = 1500 + 200x a + b = 530 (equation 1)
350x – 200x = 1500 b = a + 8 (equation 2)
150x = 1500
x = 10 Substituting,

11. An electronics store marked up a laptop a + a + 8 = 530


computer from their cost of $1200 to a 2a = 522
selling price of $1464. What was the percent a = 261
markup?
15. A man has $40,000 to invest. He will put
Solution: part of the money in an account paying 4%
“Markup” is a name for an increase. Let x be interest and the remainder into stocks paying
the percent increase. 6% interest. The total annual income from
amount of increase these investments should be $2040. How
percent increase = base
1464−1200 much should he invest at each rate?
x= 1200
= 𝟎𝟎. 𝟐𝟐𝟐𝟐 or 22%
Solution:
12. A computer store is having a terrific sale x = amount to invest at 4%
on digital cameras. After a 40% price 40000 – x = amount to invest at 6%
reduction, you purchase a digital camera for
¥201. What was the camera’s price before Use the formula for simple interest.
the reduction? I = Pin

Solution: 2040 = x(0.04)(1) + (40000 - x)(0.06)(1)


let x = price of the digital camera prior to 2040 = 0.04x – 2400 - 0.06x
the reduction x = $18000

The camera’s original price minus the 40% 40000 – 18000 = $22000
reduction is the reduced price, ¥201.
x – 0.4x = 201 16. The length of a football field is 200 feet
0.6x = 201 more than the width. If the perimeter of the
x = ¥335 field is 1040 feet, what are its dimensions?

13. The length of a rectangle is 1 cm more Solution:


than twice the width. The perimeter of the let x = width of the football field
rectangle is 110 cm. Find the length and the x + 200 = length of the football field
width of the rectangle.
We know that a rectangle’s perimeter is the
Solution: sum of twice its length and twice its width.
let L = length of the rectangle So,
W = width of the rectangle P = 2L + 2W
P = perimeter of the rectangle 1040 = 2(x + 200) + 2x
1040 = 2x + 400 + 2x
L = 1 + 2W Transpose 400 to the left side of the
P = 2L + 2W equation.
110 = 2(1 + 2W) + 2W 1040 – 400 = 4x
110 = 2 + 4W + 2W 640 = 4x
160 = x
108 = 6W
W = 18 cm It follows that
L = 1 + 2(18) = 1 + 36 = 37 cm x + 200 = 160 + 200 = 360 feet

14. There are 530 apples and bananas in a answer: length = 360 feet
large box. The number of bananas is 8 more width = 160 feet
than the number of apples. How many
apples are there in the large box? 17. A chemist must mix 8 liters of a 40%
acid solution with some 70% solution to get

Marvin Kalngan, Civil Engineer, Plumbing Engineer, AutoCAD Expert, College Lecturer
Algebra – Mathematics of Engineering 14
a 50% solution. How much of the 70% If the bus travels 20 mph faster than Marvin
solution should be used? rides his bike, how far is it to his workplace?

Solution: Solution:
8 x =8+x d = Vt
3
40% 70% = 50% bike: d = V�4�
1
8(0.4) + x(0.7) = (8 + x)(0.5) bus: d = (V + 20) �4�
3.2 + 0.7x = 4 + 0.5x d=d
x = 4 liters 3 1
V�4� = (V + 20) �4�
18. Solve for P in the equation A = P + Prt. 3V = V + 20
2V = 20
Solution: V = 10 mph
We need to isolate P. Factor out P on the 3 30
right side of the equation. d = (10)�4� = 4
= 7.5 miles
A = P + Prt
A = P(1 + rt) 22. Find three consecutive integers such that
the sum of the first and third, increased by 3,
Dividing both sides by 1 + rt gives is 50 more than the second.
𝐀𝐀
𝐏𝐏 = 𝟏𝟏+𝐫𝐫𝐫𝐫
Solution:
The consecutive integers are: x, x + 1, x + 2
19. The octane rating of gasoline is a x + x + 2 + 3 = x + 1 + 50
measure of its antiknock qualities. For a 2x + 5 = x + 51
standard fuel, the octane rating is the percent x = 46
of isooctane. How many liters of pure The three consecutive integers are 46, 47,
isooctane should be mixed with 200 L of and 48.
94% isooctane to get a mixture that is 98%
isooctane? 23. Solve: x – 7 < -12

Solution: Solution:
200 L x = 200 + x x – 7 < -12
94% 100% = 98% x < -12 + 7
x < -5
200(0.94) + x(1) = (200 + x)(0.98)
24. Solve: 14 + 2x ≤ 3x
188 + x = 196 + 0.98x
x = 400 liters
Solution:
14 + 2x ≤ 3x
20. Two cars leave the same place at the
14 ≤ x
same time, one going east and the other
x ≥ 14
west. The eastbound car averages 40 mph,
while the westbound car averages 50 mph.
25. Solve: -3(x + 4) + 2 ≥ 7 – x
In how many hours will they be 300 miles
apart? (mph means miles per hour)
Solution:
-3(x + 4) + 2 ≥ 7 – x
Solution:
-3x – 12 + 2 ≥ 7 – x
-3x – 10 ≥ 7 – x
-2x ≥ 17
Divide both sides by -2.
−17
x≤ 2

Note: Reverse the direction of the inequality


d = Vt = d1 + d2 symbol when multiplying or dividing each
300 = 50t + 40t side of an inequality by a negative number.
10
t = 3 hours ≈ 3.33 hours
26. Solve: 3 < x + 2 < 8
3
21. Marvin can bike to work in 4 hour. Solution:
1
When he takes the bus, the trip takes 4 hour. 3<x+2<8

Marvin Kalngan, Civil Engineer, Plumbing Engineer, AutoCAD Expert, College Lecturer
Algebra – Mathematics of Engineering 15
3 2
1<x<6 (x + x ) – (4x + 4) = 0
x2(x + 1) – 4(x + 1) = 0
27. Let A = {1,2,3,4} and B = {2,4,6}.
Find A⋂B and A⋃B Factor out the common binomial factor, x +
1, from each term.
Solution: x2(x + 1) – 4(x + 1) = 0
⋂ = means intersection (x + 1)(x2 - 4) = 0
⋃ = means union
A⋂B = {2,4} Set each factor equal to zero and solve the
A⋃B = {1,2,3,4,6} resulting equations.
x + 1 = 0 or x2 - 4 = 0
28. Solve: |2x + 1| = 7 x = -1 x2 = 4
x = ±√4
Solution: x=±2
2x + 1 = 7 or 2x + 1 = -7
2x = 6 or 2x = -8 33. Solve: √2x − 1 + 2 = x
x = 3 or x = -4
Solution:
29. Solve: |2x + 1| > 7 Isolate the radical on one side.
√2x − 1 + 2 = x
Solution:
√2x − 1 = x − 2
2x + 1 > 7 or 2x + 1 < -7
2x > 6 or 2x < -8
Raised both sides to the 2nd power. Then
x > 3 or x < -4
simplify.
2
30. Solve: |x + 6| = |2x - 3| �√2x − 1� = (x − 2)2
2x − 1 = x 2 − 4x + 4
Solution: x 2 − 4x − 2x + 4 + 1 = 0
x + 6 = 2x – 3 or x + 6 = -(2x - 3) x 2 − 6x + 5 = 0
9 = x or x + 6 = -2x + 3 (x - 5)(x - 1) = 0
3x = -3 x - 5 = 0 or x – 1 = 0
x = -1 x=5 x=1

31. Solve by factoring: 3x4 = 27x2 Check the proposed solutions in the original
equation.
Solution: Check 5.
Move all nonzero terms to one side and √2x − 1 + 2 = x
obtain zero on the other side. Subtracting �2(5) − 1 + 2 = 5
27x2 from both sides of the equation gives
3x4 - 27x2 = 0 √9 + 2 = 5
3+2=5
Factor out 3x2. 5 = 5 (The statement is true.)
3x4 - 27x2 = 0
3x2(x2 – 9) = 0 Check 1.
√2x − 1 + 2 = x
Set each factor equal to zero and solve the �2(1) − 1 + 2 = 1
resulting equations. √1 + 2 = 1
3x2 = 0 or x2 – 9 = 0 1+2=1
2
x =0 x2 = 9 3 = 1 (The statement is false.)
x = ±√0 x = ±√9
x=0 x = ±𝟑𝟑 Thus, 1 is an extraneous solution. The only
solution is 5, and the solution set is {5}
32. Solve by factoring: x3 + x2 = 4x + 4
34. Solve: √3x + 1 − √x + 4 = 1
Solution:
Move all nonzero terms to one side and Solution:
obtain zero on the other side. Isolate a radical on one side. We can isolate
x3 + x2 = 4x + 4 √3x + 1 on one side.
x3 + x2 - 4x – 4 = 0 √3x + 1 − √x + 4 = 1
√3x + 1 = 1 + √x + 4
Group terms that have a common factor.
Then factor out the common factors.
Marvin Kalngan, Civil Engineer, Plumbing Engineer, AutoCAD Expert, College Lecturer
Algebra – Mathematics of Engineering 16
3
y = 24⁄3 = √24
Square both sides. Then simplify. And 3
y = √16
isolate again a radical on one side of the y ≈ 𝟐𝟐. 𝟓𝟓𝟓𝟓
equation.
2 2
�√3x + 1� = �1 + √x + 4� b. Our goal is to isolate m2⁄3 on one side.
3x + 1 = 1 + 2√x + 4 + x + 4 Then we raise both sides of the equation to
3 3 2
2x – 4 = 2√x + 4 the 2 power because 2 is the reciprocal of 3.
3 1
m2⁄3 − 4 = − 2
Square both sides. Then simplify. 3 1
(2x – 4)2 = �2√x + 4�
2 m2⁄3 = 4 − 2
1
4x2 – 16x + 16 = 4(x + 4) m2⁄3 = 4
4x2 – 16x + 16 = 4x + 16 Raised both sides to 2.
3
4x2 – 20x = 0
3⁄2 1 3⁄2
Dividing both sides by 4 gives, �m2⁄3 � = ± �4�
x2 – 5x = 0
1 3⁄2
m = ± �4�
Solve the resulting equation.
1 3 1
x2 – 5x = 0 m = ±��4� = ±�64
x(x - 5) = 0 𝟏𝟏
x = 0 or x – 5 = 0 m = ± 𝟖𝟖
x=0 x=5
Note: The symbol ± is necessary because in
Check: 3
2
, the denominator 2 is even.
x=0
√3x + 1 − √x + 4 = 1 36. Solve: x4 – 8x2 – 9 = 0
�3(0) + 1 − √0 + 4 = 1
√1 − √4 = 1 Solution:
1–2=1 let y = x2
-1 = 1 (False.) x4 – 8x2 – 9 = 0
(x2)2 – 8x2 – 9 = 0
x=5 y2 – 8y – 9 = 0
√3x + 1 − √x + 4 = 1 (y - 9)(y + 1) = 0
�3(5) + 1 − √5 + 4 = 1 y – 9 = 0 or y + 1 = 0
y=9 y = -1
√16 − √9 = 1
4–3=1
Replace y with x2.
1 = 1 (True.)
y=9 y = -1
2
x =9 x2 = -1
The check indicates that 0 is not a solution.
It is an extraneous solution brought about by x = ±√9 x = ±√−1
squaring each side of the equation. The only x = ±𝟑𝟑 x = ±𝒊𝒊
solution is 5 and the solution set is {5}.
The solution set is {−3,3, −𝑖𝑖, 𝑖𝑖}.
answer: x = 5
37. Solve: 5x 2⁄3 + 11x 1⁄3 + 2 = 0
35. Solve.
a. 3y 3⁄4 − 6 = 0 Solution:
3 1 let y = x 1⁄3
b. m2⁄3 − 4 = − 2
5x 2⁄3 + 11x 1⁄3 + 2 = 0
2
Solution: 5�x 1⁄3 � + 11x 1⁄3 + 2 = 0
a. Our goal is to isolate y 3⁄4 on one side. 5y 2 + 11y + 2 = 0
Then we raise both sides of the equation to
4 4 3 Use the quadratic formula.
the 3 power because 3 is the reciprocal of 4.
a = 5; b = 11; c = 2
3y 3⁄4 − 6 = 0
3y 3⁄4 = 6 −b±√b2 −4ac −11±�112 −4(5)(2)
y= =
Dividing both sides by 3 gives, 2a 2(5)
−11±√121−40 −11±9
y 3⁄4 = 2 y= =
4 10 10
Raised both sides to 3.
4⁄3
�y 3⁄4 � = 24⁄3
Marvin Kalngan, Civil Engineer, Plumbing Engineer, AutoCAD Expert, College Lecturer
Algebra – Mathematics of Engineering 17
−11+9 −11−9 𝟏𝟏
y= 10
or y= 10
x = − 𝟐𝟐 x=1
1
y = −5 y = −2
1
The solution set is �− 2 , 1�.
Replace y with x 1⁄3 .
1
x 1⁄3 = − 5 x 1⁄3 = −2 41. Solve: 3 – 2x ≤ 11
3 1 3 3
�x 1⁄3 � = �− 5� �x 1⁄3 � = (−2)3 Solution:
𝟏𝟏 3 – 2x ≤ 11
𝐱𝐱 = − 𝟏𝟏𝟏𝟏𝟏𝟏 𝐱𝐱 = −𝟖𝟖
Subtracting 3 from both sides gives,
1
-2x ≤ 8
The solutions are − 125 and -8 and the Divide both sides by -2 and reverse the
1 direction of the inequality symbol.
solution set is �− 125 , −8�. −2x 8
−2
≥ −2
38. Solve: (x 2 − 5)2 + 3(x 2 − 5) − 10 = 0 x ≥ -4

Solution: Note: If we multiply or divide both sides of


let y = x 2 − 5 an inequality by the same negative quantity,
(x 2 − 5)2 + 3(x 2 − 5) − 10 = 0 reverse the direction of the inequality
y2 + 3y – 10 = 0 symbol.
(y + 5)(y - 2) = 0
y + 5 = 0 or y – 2 = 0 42. Solve: 2x – 4 > -x + 5
y = -5 y=2
Solution:
Replace y with x2 – 5. 2x – 4 > -x + 5
y = -5 y=2 Adding x to both sides gives,
2
x – 5 = -5 x2 – 5 = 2 3x – 4 > 5
2
x =0 x2 = 7 Adding 4 to both sides gives,
3x > 9
x=0 x = ±√𝟕𝟕
x>3
The solutions are −√7, 0 and √7, and the
solution set is �−√7, 0, √7�.
Linear Equations and Functions
39. Solve: |2x – 3| = 11
1. Find the x and y-intercepts of
Solution: 4x – y = -3.
Rewrite the equation without absolute value
bars. Solution:
|u| = c is equivalent to To find the x-intercept, let y = 0.
u = c or u = -c 4x – y = -3
4x – 0 = -3
|2x – 3| = 11 4x = -3
2x – 3 = 11 or 2x – 3 = -11 −3
x= 4
2x = 14 2x = -8
−3
x=7 x = -4 x-intercept: � 4 , 0�

The solution set is {7, −4}. To find the y-intercept, let x = 0.


4x – y = -3
40. Solve: 5|1 – 4x| - 15 = 0 4(0) – y = -3
-y = -3
Solution: y=3
5|1 – 4x| - 15 = 0 y-intercept: (0, 3)
5|1 – 4x| = 15
Dividing both sides by 5 gives, 2. Find the coordinates of the midpoint of
|1 – 4x| = 3 line segment PQ with endpoints P(4, -3) and
Q(6, -1).
Rewrite the equation without absolute value
bars. Solution:
|1 – 4x| = 3 Use the midpoint formula.
1 – 4x = 3 or 1 – 4x = -3 x +x 4+6
x = 12 2 = 2 = 5
-2 = 4x 4 = 4x
Marvin Kalngan, Civil Engineer, Plumbing Engineer, AutoCAD Expert, College Lecturer
Algebra – Mathematics of Engineering 18
7. Write an equation of the line with slope
y1 +y2 −3−1 4
y= = = -2 − 5 and y-intercept (0, -2).
2 2

midpoint: (5, -2) Solution:


The slope-intercept form of the equation of a
3. Find the slope of the line through the line with slope m and y-intercept (0, b) is
points (2, -1) and (-5, 3). y = mx + b.
4
So the equation of the line is y = − 5 x − 2.
Solution:
y −y 3−(−1) 4 4
m = x2 −x1 = −5−2 = −7 = − 7 8. Write an equation of the line with slope 3
1
2 1

and passing through the point (-2, 5). Give


4. Find the slope of the graph of your answer in standard form.
3x – 5y = 8.
Solution:
Solution: From the point-slope form of the equation of
Put the given equation in slope-intercept a line, we have
form, y = mx + b, where m is the slope and y – y1 = m(x – x1)
b is the y-intercept. 1
y – 5 = 3 (x + 2)
3x – 5y = 8
-5y = -3x + 8 Multiply both sides of the equation by 3 to
Dividing both sides by -5 gives eliminate the denominator.
3 8 1
y = 5x − 5 �y − 5 = 3 (x + 2)� (3)
3 3y – 15 = x + 2
The slope is m = 5
x – 3y = -17 (standard form: Ax + By = C)
5. Determine whether the lines through 9. Write an equation of the line passing
(-2, 1) and (4, 5) and through (3, 0) and through the points (-4, 3) and (5, -7). Give
(0, -2) are parallel. the final answer in standard form.
Solution: Solution:
Find the slope of the first line. y −y −7−3 −10 10
5−1 4 2 m = x2 −x1 = 5−(−4) = 9 = − 9
m1 = 4−(−2) = 6 = 3 2 1
From the point-slope form of the equation of
the line, we have
Find the slope of the second line. y – y1 = m(x – x1)
−2−0 −2 2
m2 = 0−3 = −3 = 3 10
y – 3 = − 9 (x + 4)
Multiply both sides of the equation by 9 to
Since the two slopes are equal, the two lines eliminate the denominator.
are parallel. 10
�y − 3 = − 9 (x + 4)� (9)
6. Determine whether the lines 2y = 3x – 6 9y – 27 = -10(x + 4)
and 2x + 3y = -6 are perpendicular. 9y – 27 = -10x – 40
10x + 9y = -13 (standard form:Ax + By = C)
Solution:
Find the slope of the first line by putting the Note: We can also use (5, -7) for (x1, y1).
equation to slope intercept form, y = mx + b.
2y = 3x – 6 10. Write an equation of the line passing
3 through the point (-3, 6) and
y = 2x − 3
3 i. parallel to the line 2x + 3y = 6
The slope of the first line is 2 ii. perpendicular to the line 2x + 3y = 6
Give final answers in slope-intercept form.
Find the slope of the second line.
2x + 3y = -6 Solution:
3y = -2x – 6 i. Find the slope of the line 2x + 3y = 6 by
2
y = −3 – 2 putting the equation into slope-intercept
2 form, y = mx + b.
The slope of the second line is − 3 2x + 3y = 6
3y = -2x + 6
Since the product of the slopes is 2
y = −3x + 2
3 2
�2� �− 3� = −1, the lines are perpendicular. 2
The slope is m = − 3

Marvin Kalngan, Civil Engineer, Plumbing Engineer, AutoCAD Expert, College Lecturer
Algebra – Mathematics of Engineering 19
The slopes of parallel lines are equal. From 1. Determine whether the ordered pair (4, 2)
the point-slope form of the equation of the is a solution of the given system.
line, we have x+y=6
y – y1 = m(x – x1) 4x – y = 14
2
y – 6 = − 3 (x + 3)
Solution:
Multiply both sides of the equation by 3 to
Replace x with 4 and y with 2 in each
eliminate the denominator.
2 equation of the system.
�y − 6 = − 3 (x + 3)� (3) x+y=6
3y – 18 = -2(x + 3) 4 + 2 =? 6
3y – 18 = -2x – 6 6=6
3y = -2x + 12
2 4x – y = 14
y = − 3x + 4
4(4) – 2 =? 14
ii. The slope of perpendicular lines are
16 – 2 =? 14
negative reciprocals. Since the slope of the
2 14 = 14
line 2x + 3y = 6 is − 3, the slope of the other
3
line is 2 Since (4, 2) makes both equations true,
From the point-slope form of the equation of (4, 2) is a solution of the system.
the line, we have
y – y1 = m(x – x1) 2. Solve the system of equations.
3 2x – y = 6 (1)
y – 6 = 2 (x + 3) x=y+2 (2)
Multiply both sides by 2 to eliminate the
denominator. Solution:
3
�y − 6 = 2 (x + 3)� (2) Substitute equation 2 in equation 1.
2x – y = 6
2y – 12 = 3(x + 3)
2(y + 2) – y = 6
2y – 12 = 3x + 9
2y + 4 – y = 6
2y = 3x + 21
3 21 y=2
y = 2x + 2
Now solve for x by substituting 2 for y in
11. Let f(x) = -x2 + 5x – 3. Find equation 2.
i. f(4) x=y+2
ii. f(q) x=2+2
x=4
Solution:
i. Just replace the variable x with 4. 3. Solve the system of equations.
f(x) = -x2 + 5x – 3 3x + 2y = 13 (1)
f(4) = -42 + 5(4) – 3 = -16 + 20 – 3 = 1 4x – y = -1 (2)

ii. Just replace the variable x with q. Solution:


f(x) = -x2 + 5x – 3 i. By substitution method.
f(q) = -q2 + 5q – 3 Solve for y in equation 2 then substitute in
equation 1.
12. Let g(x) = 2x + 3. Find g(a + 1). 4x – y = -1
4x + 1 = y
Solution:
Replace x with “a + 1”. 3x + 2y = 13
g(x) = 2x + 3 3x + 2(4x + 1) = 13
g(a + 1) = 2(a + 1) + 3 = 2a + 2 + 3 = 2a + 5 3x + 8x + 2 = 13
11x = 11
13. Given the function f(x) = √25 − x 2 , x=1
evaluate f(3).
Now solve for y by substituting x = 1 in any
Solution: of the equations.
f(x) = √25 − x 2 y = 4x + 1
y = 4(1) + 1
f(3) = √25 − 32 = √25 − 9 = √16 = 4
y=4+1
y=5
Systems of Linear Equations
The solution set is (x,y) = (1, 5).

Marvin Kalngan, Civil Engineer, Plumbing Engineer, AutoCAD Expert, College Lecturer
Algebra – Mathematics of Engineering 20
If we multiply equation 4 by 4 and multiply
ii. By elimination method. equation 6 by 5 then add the resulting
3x + 2y = 13 (1) equations, variable x will be eliminated.
4x – y = -1 (2) Multiplying equation 4 by 4 gives
4(5x + 3y = 11)
Multiply equation 2 by 2, then add the result 20x + 12 y = 44 (eq. 7)
to equation 1.
2(4x – y = -1) Multiplying equation 6 by 5 gives
8x – 2y = -2 (3) 5(-4x – 5y = -14)
-20x – 25y = -70 (eq. 8)
Now, add equation 1 and equation 3.
3x + 2y = 13 (1) Adding equations 7 and 8 then simplifying,
+ gives
8x – 2y = -2 (3) -13y = -26
11x = 11 y=2

11x = 11 Now substitute y = 2 in any of the equations


x=1 to solve any variable.

Now solve for y by substituting x = 1 in any Substituting y = 2 in equation 4 then


of the equations. simplifying, gives
3x + 2y = 13 5x + 3y = 11
3(1) + 2y = 13 5x + 3(2) = 11
3 + 2y = 13 5x + 6 = 11
2y = 10 5x = 5
y=5 x=1

The solution set is (x,y) = (1, 5). Now substitute y = 2 and x = 1 in any of the
equations to solve for z. Substituting y = 2
4. Solve the system of equations. and x = 1 in equation 1 then simplifying,
2x + y − z = 1 gives
�3x + 2y + z = 10 2x + y – z = 1
2x − y + 2z = 6 2(1) + 2 – z = 1
2+2–z=1
Solution: 4–1=z
2x + y − z = 1 (eq. 1) 3=z
�3x + 2y + z = 10 (eq. 2)
2x − y + 2z = 6 (eq. 3) Hence, the solution set is (x,y,z) = (1,2,3)
Choose any two equations and eliminate any
variable. Best way is to eliminate a variable 5. Solve the system.
from the first two equations. If we add the 4x + 8y + z = 2 (1)
first two equations we can eliminate z. x + 7y – 3z = -14 (2)
Adding the first two equations gives 2x – 3y + 2z = 3 (3)
5x + 3y = 11 (eq. 4)
Solution:
Now we need to eliminate z involving the Multiply equation (1) by 3 and add the result
third equation. We can either eliminate z to equation (2) to eliminate variable z.
from equation 3 and equation 1 or from 3(4x + 8y + z = 2)
equation 3 and equation 2. If we multiply 12x + 24y + 3z = 6
equation 2 by -2 then add the result to
equation 3, we can eliminate z. Multiplying Adding the above equation to equation (2)
equation 2 by -2, gives gives,
-2(3x + 2y + z = 10) 12x + 24y + 3z = 6
-6x – 4y – 2z = -20 (eq. 5) +
x + 7y – 3z = -14
Adding equations 3 and 5, gives 13x + 31y = -8 (4)
-4x – 5y = -14 (eq. 6)
Multiply equation (1) by -2 then add the
So the variable z was eliminated from the result to equation (3) to eliminate variable z.
three equations. We are down to two -2(4x + 8y + z = 2)
equations (eq. 4 and eq. 6) -8x – 16y – 2z = -4

Marvin Kalngan, Civil Engineer, Plumbing Engineer, AutoCAD Expert, College Lecturer
Algebra – Mathematics of Engineering 21
Adding the above equation to equation (3)
gives, Adding the results gives,
-8x – 16y – 2z = -4 12x – 24y = -10
+ +
2x – 3y + 2z = 3 24y + 3z = 0
-6x – 19y = -1 (5) 12x + 3z = -10 (eq. 4)

So the variable z was eliminated from the Now, multiply eq. 3 by 3 then add the result
three equations. We are down to two to eq. 4 to eliminate variable z.
equations: equation (4) and equation (5)
3(9x – z = 12)
We choose to eliminate x. Multiply equation 27x – 3z = 36
(4) by 6 and equation (5) by 13 then add the
results. 27x – 3z = 36
+
6(13x + 31y = -8) 12x + 3z = -10
78x + 186y = - 48 39x = 26

13(-6x – 19y = -1) 39x = 26


-78 – 247y = -13 26 2
x = 39 = 3

Adding the results gives, 2


78x + 186y = - 48 Substitute 3 for x in eq. 3 to solve for z.
+
-78 – 247y = -13 9x – z = 12
2
-61y = -61 9�3� – z = 12
y=1 18
3
- z = 12
Substitute 1 for y in either equation (4) or 6 – z = 12
(5) to find x. -6 = z
13x + 31y = -8
13x + 31(1) = -8 Substitute -6 for z in eq. 2 to solve for y.
13x + 31 = -8 8y + z = 0
13x = -39 8y – 6 = 0
x = -3 8y = 6
6 3
y=8=4
Now substitute y = 1 and x = -3 in any of the
three original equations to solve for z. Thus, the solution set is
4x + 8y + z = 2 2 3
4(-3) + 8(1) + z = 2 (x,y,z) = �3 , 4 , −6�
-12 + 8 + z = 2
z=6 7. A soccer field has a perimeter of 320
yards and length of 40 yards more than the
Hence, the solution set is (x,y,z) = (-3,1,6) width. What are the dimensions of this
soccer field?
6. Solve the system of equations.
6x – 12y = -5 (eq. 1) Solution:
8y + z = 0 (eq. 2) P = 320 yards
9x – z = 12 (eq. 3) L = 40 + W

Solution: P = 2L + 2W
Since eq. 3 is missing in variable y, one way 320 = 2(40 + W) + 2W
to begin is to eliminate y again, using eq. 1 320 = 80 + 2W + 2W
and eq. 2. 240 = 4W
60 = W
Multiply eq. 1 by 2 and eq. 2 by 3 then add W = 60 yards
the results to eliminate variable y.
2(6x – 12y = -5) L = 40 + W
12x – 24y = -10 L = 40 + 60 = 100
L = 100 yards
3(8y + z = 0)
24y + 3z = 0

Marvin Kalngan, Civil Engineer, Plumbing Engineer, AutoCAD Expert, College Lecturer
Algebra – Mathematics of Engineering 22
8. At a school concert, the total value of v(car) = 8km/hr + v(truck)
tickets sold was $1,506. Student tickets sold
for $6 each and adult tickets sold for $9 d
working formula: v = t
each. The number of adult tickets sold was 5
where: v = speed, d = distance, t = time
less than three times the number of student
tickets sold. How many student tickets and d(car)
how many adult tickets were sold? v(car) = t(car)
d(car) 250
t(car) = v(car) = v(car)
Solution:
Let “s” be the number of student tickets sold
d(truck) 225
and “a” be the number of adult tickets sold t(truck) = v(truck) = v(truck)

a = 3s – 5
t(car) = t(truck)
250 225
6s + 9a = 1506 v(car)
= v(truck)
6s + 9(3s - 5) = 1506
6s + 27s – 45 = 1506 But: v(car) = 8 + v(truck), so
33s = 1551
s = 47 250 225
8+ v(truck)
= v(truck)
Substitute: v(truck)*250 = [8 + v(truck)](225)
a = 3s – 5 v(truck)*250 = 1800 + v(truck)*225
a = 3(47) – 5 v(truck)*25 = 1800
a = 136 v(truck) = 72 kph

9. How many ounces each of 5% It follows that,


hydrochloric acid and 20% hydrochloric v(car) = 8 + v(truck)
acid must be combined to get 10 oz of v(car) = 8 + 72
solution that is 12.5% hydrochloric acid? v(car) = 80 kph

Solution: 11. A company produces three flat screen


Let x = number of ounces of the 5% solution television sets: models X, Y, and Z.
y = number of ounces of 20% solution • Each model X set requires 2 hour of
electronics work, 2 hour of assembly time,
x + y = 10 (equation 1) and 1 hour of finishing time.
0.05x + 0.2y = 0.125(10) • Each model Y requires 1 hour of
0.05x + 0.2y = 1.25 (equation 2) electronics work, 3 hour of assembly time,
and 1 hour of finishing time.
Solve for x in equation 1 then substitute in • Each model Z requires 3 hour of
equation 2. electronics work, 2 hour of assembly time,
x + y = 10 and 2 hour of finishing time. There are 100
x = 10 – y hour available for electronics, 100 hour
0.05x + 0.2y = 1.25 available for assembly, and 65 hour
0.05(10 - y) + 0.2y = 1.25 available for finishing per week. How many
0.5 – 0.05y + 0.2y = 1.25 of each model should be produced each
0.15y = 0.75 week if all available time must be used?
y=5
Solution:
It follows that, Let x = number of model X produced per
x = 10 – y week
x = 10 – 5 y = number of model Y produced per
x=5 week
z = number of model Z produced per
10. A car travels 250 km in the same time week
that a truck travels 225 km. If the rate of the x y z Totals
car is 8 km per hour faster than the rate of Hours of Electronics 2 1 3 100
the truck, find both rates. Work
Hours of Assembly 2 3 2 100
Solution: Time
d(car) = 250 km Hours of Finishing 1 1 2 65
d(truck) = 225 km Time
t(car) = t(truck)

Marvin Kalngan, Civil Engineer, Plumbing Engineer, AutoCAD Expert, College Lecturer
Algebra – Mathematics of Engineering 23
2x + y + 3z = 100 (1) We simplify first equation 3.
2x + 3y + 2z = 100 (2) M + 5 = 3(L + 5)
x + y + 2z = 65 (3) M + 5 = 3L + 15
M – 3L = 10
Subtract equation 1 from equation 2 to
eliminate x. However, M = 2S
2x + 3y + 2z = 100 M – 3L = 10
- 2S – 3L = 10
2x + y + 3z = 100
2y - z = 0 (4) However, L = S – 5
2S – 3L = 10
Multiply equation 3 by -2 then add the result 2S – 3(S - 5) = 10
in equation 1 to eliminate x. 2S – 3S + 15 = 10
-2(x + y + 2z = 65) -S = -5
-2x – 2y – 4z = -130 S=5

-2x – 2y – 4z = -130 13. The ratio of the ages of two brothers is


+ 1:3. Thirteen years from now the ratio will
2x + y + 3z = 100 be 1:2. What is the sum of their present
-y - z = - 30 (5) ages?

Subtract equation 5 from equation 4 to Solution:


eliminate z. At present,
2y – z = 0 A 1
B
=3
-
-y – z = -30
B = 3A (eq. 1)
3y = 30
Thirteen years from now,
3y = 30 A+13 1
y = 10 B+13
=2

Substitute y = 10 in equation 5. 2A + 26 = B + 13
-y – z = -30 B – 2A = 26 – 13
-10 – z = -30 B – 2A = 13 (eq. 2)
20 = z
Substitute (eq. 1) into (eq. 2),
Substitute y = 10 and z = 20 in any of the 3A – 2A = 13
original equations. A = 13
x + y + 2z = 65
x + 10 + 2(20) = 65 It follows that
x + 10 + 40 = 65 B = 39
x + 50 = 65
x = 15 So the sum of their ages is
13 + 39 = 52
The solution set is (x, y, z) = (15, 10, 20).
The company should produce 15 model X, 14. A father is three times as old as his son.
10 model Y, and 20 model Z sets per week. Four years ago, he was four times as old as
his son was at that time. How old is his son?
12. Marvin is twice as old as his friend Sky.
Sky is 5 years older than Lee. In 5 years, Solution:
Marvin will be three times as old as Lee. Let F = present age of the father
Find the present age of Sky. S = present age of his son

Solution: F = 3S (1)
Let S = present age of Sky F – 4 = 4(S - 4) (2)
M = present age of Marvin
L = present age of Lee Substitute equation 1 into equation 2.
F – 4 = 4(S - 4)
M = 2S (1) 3S – 4 = 4(S - 4)
L=S–5 (2) 3S – 4 = 4S – 16
M + 5 = 3(L + 5) (3) 16 – 4 = 4S – 3S
12 = S

Marvin Kalngan, Civil Engineer, Plumbing Engineer, AutoCAD Expert, College Lecturer
Algebra – Mathematics of Engineering 24
Let t = tens digit
15. One-half of Marvin’s age two years from u = units digit
now plus one-third of his age three years ago 10t + u = value of the number
is twenty years. How old is Marvin now?
t+u=7 (1)
Solution: 10t + u = 12t – 2 (2)
Let M = present age of Marvin
Solve for t in equation 1 then substitute in
M + 2 = Marvin’s age two years from now equation 2.
M – 3 = Marvin’s age three years ago
t+u=7
(1/2)(M + 2) + (1/3)(M - 3) = 20 t=7–u
M/2 + 1 + M/3 – 1 = 20
M/2 + M/3 = 20 10t + u = 12t – 2
Multiply both sides by 6 to eliminate 10(7 - u) + u = 12(7 - u) – 2
denominators. 70 – 10u + u = 84 – 12u – 2
6(M/2 + M/3 = 20) 12u – 10u + u= 84 – 2 – 70
3M + 2M = 120 3u = 12
5M = 120 u=4
M = 24
It follows that
16. In three more years, Miguel’s t=7–u
grandfather will be six times as old as t=7–4
Miguel was last year. When Miguel’s t=3
present age is added to his grandfather’s
present age, the total is 68. How old is each The number is 34.
one now?
Challenge: Solve it in your own way.
Solution:
Let M = Miguel’s present age 18. The tens digit of a number is twice the
G = grandfather’s present age units digit. If the digits are reversed, the new
G + 3 = grandfather’s age three years number is 27 less than the original. Find the
from now original number.
M – 1 = Miguel’s age last year
Solution:
G + 3 = 6(M - 1) (1) t = tens digit
M + G = 68 (2) u = units digit
10t + u = original number
Solve M in equation 2 then substitute in 10u + t = reverse number
equation 1.
t = 2u (1)
M + G = 68 10u + t = 10t + u – 27 (2)
M = 68 – G Simplify equation 2.
10u + t = 10t + u – 27
G + 3 = 6(M - 1) 9u – 9t = -27
G + 3 = 6(68 – G - 1)
G + 3 = 6(67 - G) Substitute equation into the above equation.
G + 3 = 402 – 6G 9u – 9t = -27
7G = 399 9u – 9(2u) = -27
G = 57 9u – 18u = -27
-9u = -27
It follows that u=3
M = 68 – G
M = 68 – 57 It follows that,
M = 11 t = 2u
t = 2(3)
17. The sum of the digits of a two digit t=6
number is 7. The value of the number is 2
less than 12 times the tens digit. Find the So the original number is 63.
number.
19. The sum of the digits of a three-digit
Solution: number is 6. The hundreds digit is twice the

Marvin Kalngan, Civil Engineer, Plumbing Engineer, AutoCAD Expert, College Lecturer
Algebra – Mathematics of Engineering 25
units digit, and the tens digit equals the sum
of the other two. Find the number. The number is 42.

Solution: 21. When 6 times a number is increased by


h = hundreds digit 4, the result is 40. Find the number.
t = tens digit
u = units digit Solution:
100h + 10t + u = the number Let n be the number.

h+t+u=6 (1) 6n + 4 = 40
h = 2u (2) 6n = 36
t=h+u (3) n=6

Substitute equation 2 and equation 3 in 22. The sum of two consecutive integers is
equation 1. 15. Find the integers.
h+t+u=6
2u + h + u + u = 6 Solution:
Simplify. Let x be the first integer, then the second
4u + h = 6 integer is x + 1.

However, h = 2u. Substitute. x + (x + 1) = 15


4u + h = 6 x + x + 1 = 15
4u + 2u = 6 2x + 1 = 15
6u = 6 2x = 14
u=1 x=7

It follows that, The second integer is


h = 2u x+1=7+1=8
h = 2(1)
h=2 The integers are 7 and 8.

t=h+u 23. The sum of the first and third of three


t=2+1 consecutive odd integers is 131 less than
t=3 three times the second integer. Find the three
integers.
The number is 231.
Solution:
20. The ratio of the units digit to the tens x = first odd integer
digit of a two-digit number is one-half. The x + 2 = second odd integer
tens digit is 2 more than the units digit. Find x + 4 = third odd integer
the number.
x + (x + 4) = 3(x + 2) - 131
Solution: x + x + 4 = 3x + 6 - 131
t = tens digit 2x + 4 = 3x – 125
u = units digit 129 = x
u 1
t
=2 (1) It follows that
x + 2 = 129 + 2 = 131
t=u+2 (2)
x + 4 = 129 + 4 = 133
Cross-multiply equation 1.
u 1
t
= 2 The integers are 129, 131, and 133.
2u = t
24. If one number is three times as large as
t=t another number and when the smaller
u + 2 = 2u number is increased by 19, the result is 6
2=u less than twice the larger number. Find the
numbers.
It follows that,
2u = t Solution:
2(2) = t x = larger number
4=t y = smaller number

Marvin Kalngan, Civil Engineer, Plumbing Engineer, AutoCAD Expert, College Lecturer
Algebra – Mathematics of Engineering 26

x = 3y (1) T T
�12 + 16 = 1� 192
y + 19 = 2x – 6 (2)
16T + 12T = 192
28T = 192
Substitute x = 3y in equation 2. 48
y + 19 = 2x – 6 T = 7 ≈ 6.86 days
y + 19 = 2(3y) – 6
y + 19 = 6y – 6 27. Rivin, Veral and Pogi can finish painting
25 = 5y the fence in 2 hours. If Rivin does the job
y=5 alone he can finish it in 5 hours. If Veral
does the job alone he can finish it in 6 hours.
It follows that How long will it take for Pogi to finish the
x = 3y job alone?
x = 3(5)
x = 15 Solution:
T T T
A
+B+C =1
The numbers are 15 and 5.
where: T = time working together
A = time for person A working alone
25. Find four consecutive even integers so
B = time for person B working alone
that the sum of the first two added to twice
C = time for person C working alone
the sum of the last two is equal to 742.
2 2 2
Solution: 5
+6+C=1
x = first even integer
x + 2 = second even integer Multiply both sides by the product of 5, 6
x + 4 = third even integer and C to remove denominators.
x + 6 = fourth even integer 5 × 6 × C = 30C

[x + (x + 2)] + 2[(x + 4) + (x + 6)] = 742 2 2 2


30C �5 + 6 + C = 1�
x + x + 2 + 2(x + 4 + x + 6) = 742
2x + 2 + 2(2x + 10) = 742 12C + 10C + 60 = 30C
2x + 2 + 4x + 20 = 742 C = 7.5 hours
6x + 22 = 742
x = 120 So, Pogi can finish the job alone in 7.5
hours.
It follows that,
x + 2 = 120 + 2 = 122 28. A tank can be filled by pipe A in 3 hours
and by pipe B in 5 hours. When the tank is
x + 4 = 120 + 4 = 124 full, it can be drained by pipe C in 4 hours.
if the tank is initially empty and all three
x + 6 = 120 + 6 = 126 pipes are open, how many hours will it take
to fill up the tank?
The integers are 120, 122, 124 and 126.
Solution:
T T T
26. Pedro and Ceasar are asked to paint a A
+B−C =1
house. Pedro can paint the house by himself where: T = time to fill up the tank
in 12 hours and Ceasar can paint the house A = time for pipe A to fill the tank
by himself in 16 hours. How long would it alone
take to paint the house if they worked B = time for pipe B to fill the tank
together? alone
C = time for pipe C to drain the tank
Solution: alone
T T
A
+B=1 Note: Pipe C is for draining the tank so it is
where: T = time working together negative.
A = time for person A working alone T T T
B = time for person B working alone 3
+5−4=1

T T
12
+ 16 = 1 Multiply both sides of the equation by the
Multiply both sides by the product of 12 and product of 3, 5, and 4 to remove
16 to remove denominators. denominators.
12 × 16 = 192 3 × 5 × 4 = 60

Marvin Kalngan, Civil Engineer, Plumbing Engineer, AutoCAD Expert, College Lecturer
Algebra – Mathematics of Engineering 27
T T T 9T = 21
60 �3 + 5 − 4 = 1� 21 7 1
20T + 12T – 15T = 60 T = 9 = 3 = 2 3 hours or 2 hours and 20 min
17T = 60
T = 3.53 hours The swimming pool will be filled at
9:00 + 2 hours and 20 mins = 11:20 am
29. Phoebe and Anne can do a certain job in
3 hours. On a given day, they worked 31. How long will it take a bus traveling 72
together for one hour then Anne left and km/hr to go 36 km?
Phoebe finishes the rest of the work in 8
more hours. How long will it take for Solution:
Phoebe to do the job alone? d = Vt
where d = distance, V = speed, t = time
Solution:
P = number of hours for Phoebe to do the Plug in the given values into the formula.
job alone d = Vt
A = number of hours for Anne to do the job 36 km = (72 km/hr)(t)
alone t = 0.5 hour

3 3 32. Mr. Lee leaves a certain place at 9 a.m.


P
+A=1 (1)
and drives west at an average speed of 50
1 1 8 miles per hour. Mr. Vhin leaves the same
P
+A+P=1 (2) place at 9:30 a.m. and drives west at an
average speed of 60 miles per hour. At what
Simplify equation 2. time will Mr. Vhin overtake Mr. Lee, and
1 1 8 how many miles will they each have gone?
P
+A+P=1
9 1
P
+A=1 Solution:
1 9
A
= 1−P Let “t” stands for the time Mr. Vhin drives
before overtaking Mr. Lee. Then Mr. Lee
1
Substitute the above equation into equation drives for “t + 2 hours” before being
1. Then simplify. overtaken.
3 3
P
+ A
=1
3 9 d = Vt
P
+ 3 �1 − P� = 1 where d = distance, V = speed, t= time
3 27
P
+3− P
=1
1
−24
= −2 Mr. Lee: d = 50�t + 2�
P
-24 = -2P Mr. Vhin: d = 60t
P = 12 hours
Because they travelled the same distance,
30. A swimming pool can be filled by pipe d=d
1
A in 3 hours and by pipe B in 6 hours. At 60t = 50�t + 2�
9:00 am pipe A started. At what time will 60t = 50t + 25
the swimming pool be filled if pipe B is 10t = 25
started at 10:00 am? t = 2.5 hours

Solution: It follows that,


T T−1
A
+ B
=1 d = 60t = 60(2.5) = 150 miles
T = time to fill the pool or
1 1
A = time for pipe A to fill the pool alone d = 50�t + 2� = 50�2.5 + 2� = 150 miles
B = time for pipe B to fill the pool alone
Note: The T-1 is for pipe B because it was 33. Two trains, each having a speed of 30
delayed for 1 hour. kph, are headed at each other on the same
T T−1
straight track. A bird that can fly 60 kph
3
+ 6 =1 flies off the front of one train when they are
Multiply both sides by the product of 3 and 60 km apart and heads directly for the other
6 to remove denominators. train. On reaching the other train, the bird
T T−1 flies directly back to the first train, and so
18 �3 + 6 = 1�
forth. What is the total distance the bird
6T + 3(T - 1) = 18 travels before the trains collide?
6T + 3T – 3 = 18
Marvin Kalngan, Civil Engineer, Plumbing Engineer, AutoCAD Expert, College Lecturer
Algebra – Mathematics of Engineering 28
36. Two planes are 2800 km apart. They
Solution: move towards each other meeting after 3.5
hours. One plane flies at 75 kph slower than
the other. What was the speed of each
plane?

Solution:

The two trains are headed to each other until


collision. The time frame until collision is
also the time frame the bird travels. Since
the speed of the two trains are equal, they
will collide at the center. The time frame
until collision is
d = Vt We know that d = Vt.
30 km = (30 kph)(t) d = d1 + d2
t = 1 hour 2800 = 3.5V + 3.5(V - 75)
2800 = 3.5V + 3.5V – 262.5
The total travel distance of the bird is 3062.5 = 7V
dB = VBt = (60 kph)(1 hour) = 60 km V = 437.5 kph

34. Two trains start from the same station It follows that the speed of the slower plane
and runs in opposite directions. One runs at is
a uniform rate 33 kph while the other at 35
kph. In how many hours will they be 195 km V – 75 = 437.5 – 75 = 362.5 kph
apart?
37. Car A which is running at the speed of
Solution: 60 km/hr leaves station N which is 200 km
away from station S where it is heading. Car
B leaves station S heading to station N at the
speed of 40 km/hr. How long will the two
cars meet?

Solution:
d = Vt
d1 + d2 = 195
35t + 33t = 195
t = 2.87 hours ≈ 3 hours

35. A messenger traveling 65 miles per hour


pursues a truck which has a start of 2 hours d = Vt
and overtakes it in 3 hours. Find the speed of d = d1 + d2 = VAt + VBt
the truck in mph. 200 = 60t + 40t
200 = 100t
Solution: t = 2 hours

38. A car and an emergency vehicle are


heading toward each other. The car is
traveling at a speed of 30 miles per hour.
The emergency vehicle is traveling at a
speed of 50 miles per hour. If the vehicles
are 1000 feet apart, in how many seconds
d = Vt will the driver of the car first hear the siren?
where: d = distance, V = speed, t = time A siren can be heard from up to 440 feet.

The distance travelled by the messenger in 3 Solution:


hours is equal to the distance travelled by
the truck in 2 + 3 = 5 hours.
dm = dt
(65)(3) = (Vt)(5)
Vt = 39 mph

Marvin Kalngan, Civil Engineer, Plumbing Engineer, AutoCAD Expert, College Lecturer
Algebra – Mathematics of Engineering 29

Their initial distance from each other is


1000 ft. The siren can be heard by the car if
their distance is 440 ft.

Let the speed of the car be C and the speed


of the emergency vehicle be E. Now,
convert these two speeds to feet per second.

miles 5280 feet 1 hour


C = 30 hour
× 1 mile
× 3600 seconds
feet
= 44 seconds

miles 5280 feet 1 hour From the figure above,


E = 50 hour
× 1 mile
× 3600 seconds
feet
x + 30 = 35 + (x/12)
= 73.33 seconds x = 5.45 minutes

From the figure, time: 7:05.45


1000 = d1 + d2 + 400
working formula: d = Vt; where V = speed 41. At what time after 8:00 will the hour
d = distance and t = time hand and the minute hand form a right
1000 = 73.33t + 44t + 440 angle?
t = 4.77 seconds
Solution:
39. A man riding on a motorcycle runs 40
kph, is followed 5 hrs later by a car which
runs 60 kph. In how many hours will the car
overtake the motorcycle?

Solution:

Working Formula:
d = Vt; where V = speed, d = distance and
t = time
The total distance travelled by the car and If the minute hand moves a distance of x, the
man are equal. But the man is advance by 5 hour hand moves a distance of x/12. A
hours. So straight line (180 degrees) is equivalent to
dcar = dman 30 minutes so a 90 degree angle is
60t = 40(5 + t) equivalent to 15 minutes.
60t = 200 + 40t From the figure above,
20t = 200 x + 15 = 40 + x/12
t = 10 hours x = 27.27 minutes

40. At what time after 7:00 will the hour time: 8:27.27
hand and the minute hand be in a straight
line? 42. What is the time after 1:00 o’clock will
the hands of the clock forms a straight line
Solution: for the first time?
If the minute hand moves a distance of x, the
hour hand moves a distance of x/12. A Solution:
straight line (180 degrees) is equivalent to
30 minutes.

Marvin Kalngan, Civil Engineer, Plumbing Engineer, AutoCAD Expert, College Lecturer
Algebra – Mathematics of Engineering 30

If the minute hand moves a distance of x


minutes, the hour hand moves a distance of
x/12 minutes. Convert first the 120° to
minutes.
30° = 5 minutes
5 minutes
120° × 30° = 20 min
From the figure, we have
x
x = 12 + 20
x = 21.82 minutes

44. What is the time when the hands after


2:00 o’clock are perpendicular with each
other?

Solution:
If the minute hand moves a distance of x
minutes, the hour hand moves a distance of If the minute hand moves a distance of x
x/12 minutes. minutes, the hour hand moves a distance of
From the figure above, x/12 minutes.
x
x = 5 + 2 + 30 conversion: 5 min = 30°
x = 38.18 minutes or 38:11

time = 1:38:11

43. After how many minutes from 12:00


o’clock will the minute hand and the hour
hand of a clock make an angle of 120° for
the first time?

Solution:

Since the hour hand and the minute hand are


perpendicular to each other, they form a 90°
angle. Convert the 90° to minutes. We know
that 5 minutes is 30°, so
5 min
90° × 30° = 15 min

From the figure above,


x
x = 10 + 12 + 15

From the figure above,


x = 27.27 mins or 27:16

time = 2:27:16

45. At what time after 12:00 noon will the


hour hand and the minute hand of a clock
form an angle of 120° for the first time?

Solution:

Marvin Kalngan, Civil Engineer, Plumbing Engineer, AutoCAD Expert, College Lecturer
Algebra – Mathematics of Engineering 31
0.2(20) – 0(x) = 0.3(20 - x)
4 = 6 – 0.3x
x = 6.67 ounces

49. Nara wants to make a 100 ml of 5%


alcohol solution mixing a quantity of a 2%
alcohol solution with a 7% alcohol solution.
What are the quantities of each of the two
solutions (2% and 7%) she has to use?

Solution:

x y 100 ml
2% 7% 5%
Let y be the angle formed in a 120 degree
angle. We know that 30 min = 180 degrees, x + y = 100 (eq. 1)
so
y 30 min
120°
= 180° 0.02x + 0.07y = 0.05(100)
y = 20 minutes 0.02x + 0.07y = 5 (eq. 2)

From the figure, Solve for x in equation 1 then substitute in


x equation 2.
x = 12 + 20
x = 100 – y
x = 21.818 minutes
Time: 12:21.818 0.02x + 0.07y = 5
0.02(100 - y) + 0.07y = 5
46. How many liters of a 70% alcohol 2 – 0.02y + 0.07y = 5
solution must be added to 50 liters of a 40% y = 60 ml
alcohol solution to produce a 50% alcohol
solution? It follows that,
x = 100 – y
Solution: x = 40 ml
x 50 liters = x + 50
70% 40% = 50% 50. A store owner wants to mix cashews and
almonds. Cashews cost 2 dollars per pound
0.7x + 0.4(50) = 0.5(x + 50) and almonds cost 5 dollars per pound. He
0.7x + 20 = 0.5x + 25 plans to sell 150 pounds of a mixture. How
0.2x = 5 many pounds of each type of nuts should be
x = 25 liters mixed if the mixture will cost 3 dollars?

47. How many ounces of pure water must be Solution:


added to 50 ounces of a 15% saline solution Let “a” be the number of pounds of almonds
to make a saline solution that is 10% salt? Let “c” be the number of pounds of cashews

Solution: a c = 150
x 50 ounces = x + 50 $5 $2 = $3
0% 15% = 10%
0(x) + 0.15(50) = 0.1(x + 50) a + c = 150 (eq. 1)
0 + 7.5 = 0.1x + 5
x = 25 ounces 5a + 2c = 3(150)
5a + 2c = 450 (eq. 2)
48. Priya has 20 ounces of a 20% of salt
solution. How much water should he Solve for “a” in equation 1, then substitute
evaporate to make it a 30% solution? in equation 2.
a + c = 150
Solution: a = 150 – c

Let x be the amount of water evaporated. 5a + 2c = 450


5(150 - c) + 2c = 450
20 ounces x = 20 - x 750 – 5c + 2c = 450
20% 0% = 30% c = 100 pounds

Marvin Kalngan, Civil Engineer, Plumbing Engineer, AutoCAD Expert, College Lecturer
Algebra – Mathematics of Engineering 32
It follows that, Solution:
a = 150 – 100
1
a = 50 pounds Use: (am )n = amn and a−n = an

51. 50 liters of a 15% alcohol solution is 1 1


mixed with 150 liters of a 35% alcohol (23 x −2 )−2 = (8x −2 )−2 = =
solution. What is the concentration of the (8x −2 )2 82 x −4
x4 x4
mixture? = 82 = 64

Solution: 2 −1
3x2 4x3
f. � � �y−2 �
y
50 liters 150 liters = 50 + 150 = 200
15% 35% x% Solution:
1
0.15(50) + 0.35(150) = x(200) Use: (am )n = amn and a−n = an
7.5 + 52.5 = 200x
and am ∙ an = am+n
x = 30%

Exponents, Polynomials, and 3x2


2
4x3
−1
32 x4 4−1 x−3
Polynomial Functions � � �y−2� =� �� �
y y2 y2
32 x4 1
=� � �4 1 y 2 x 3 �
1. Simplify each exponential expression so y2
32 x4 1
that no negative exponents appear in the =� � �4y2 x3�
y2
final result. Assume that all variables 32 x4
represent nonzero real numbers. = 4y2 x3y2
9x4
2
a. 3 ∙ 3 −5 = 4y4 x3
9x
= 4y4
Solution:
1
Use: am ∙ an = am+n and a−n = an 2. Write each number in scientific notation.
1 1
32 ∙ 3−5 = 32−5 = 3−3 = 33 = 27 a. 970,000

b. x −3 ∙ x −4 ∙ x 2 Solution:
Move the decimal point 5 places to the left.
Solution:
1 970,000 = 9.7 × 105
Use: am ∙ an = am+n and a−n = an
b. 0.00000000072
−3 −4 2 −3−4+2 −5 1
x ∙x ∙x =x =x = x5
Solution:
c. (x −4 )6 Move the decimal point 10 places to the
right.
Solution:
1 0.00000000072 = 7.2 × 10-10
Use: (am )n = amn and a−n = an
3. Write each number in standard notation.
1
(x −4 )6 = x −24 = 24 a. 9.9 × 106
x

x−4 y2 Solution:
d. x2y−5
Move the decimal point 6 places to the right.

Solution: 9.9 × 106 = 9900000


1
Use: am ∙ an = am+n and a−n = an b. 7.2 × 10-7

x−4 y2 y2 y5 y7 Solution:
= = x6 Move the decimal point 7 places to the left.
x2 y−5 x4 x2

e. (23 x −2 )−2 7.2 × 10-7 = 0.00000072

Marvin Kalngan, Civil Engineer, Plumbing Engineer, AutoCAD Expert, College Lecturer
Algebra – Mathematics of Engineering 33
5 3 2 5 3
4. Add: (3a – 9a + 4a ) + (-8a + 8a + 2) Solution:

Solution: (f + g)(x) = f(x) + g(x)


= (x2 – 3x + 7) + (-3x2 – 7x + 7)
(3a5 – 9a3 + 4a2) + (-8a5 + 8a3 + 2) = x2 – 3x + 7 - 3x2 – 7x + 7
= 3a5 – 9a3 + 4a2 - 8a5 + 8a3 + 2 = -2x2 – 10x + 14
= -5a5 - a3 + 4a2 + 2
b. (f - g)(x)
5. Subtract:
(-6m2 – 8m + 5) – (-5m2 + 7m - 8) Solution:

Solution: (f - g)(x) = f(x) – g(x)


= (x2 – 3x + 7) - (-3x2 – 7x + 7)
(-6m2 – 8m + 5) – (-5m2 + 7m - 8) = x2 – 3x + 7 + 3x2 + 7x – 7
= -6m2 – 8m + 5 + 5m2 - 7m + 8 = 4x2 + 4x
= - m2 – 15m + 13
9. Let f(x) = x2 and g(x) = x + 3.
6. Let f(x) = 4x3 – x2 + 5. Find each value. Find (f ∘ g)4.

a. f(3) Solution:

Solution: (f ∘ g)4 = f(g(4)) = f(4 + 3) = f(7) = 72 = 49

f(x) = 4x3 – x2 + 5 10. Let f(x) = x2 and g(x) = x + 3.


f(3) = 4(3)3 – 32 + 5 Find (g ∘ f)4.
f(3) = 108 – 9 + 5
f(3) = 104 Solution:

b. f(-4) (g ∘ f)4 = g(f(4)) = g(42) = g(16)


= 16 + 3 = 19
Solution:
11. Find the product. -4a3(3a5)
f(x) = 4x3 – x2 + 5
f(-4) = 4(-4)3 – (-4)2 + 5 Solution:
f(-4) = 4(-64) – 16 + 5
f(-4) = - 267 -4a3(3a5) = -12a8

7. The number of students enrolled in public 12. Find the product. 2m2z4(8m3z2)
schools (grade 12) in a certain country
during the years 1990 through 2006 can be Solution:
modeled by the polynomial function defined
by 2m2z4(8m3z2) = 16m5z6
P(x) = -0.01774x2 + 0.7871x + 41.26
where x = 0 corresponds to the year 1990, x 13. Find the product. 2x2(x + 1)(x - 3)
= 1 corresponds to 1991, and so on, and P(x)
is in millions. Use this function to Solution:
approximate the number of public school in
2006. 2x2(x + 1)(x - 3) = 2x2(x2 - 3x + x - 3)
Solution: = 2x2(x2 - 3x + x - 3)
= 2x2(x2 - 2x - 3)
Since x = 16 corresponds to 2006, we must = 2x4 – 4x3 – 6x2
find P(16).
P(x) = -0.01774x2 + 0.7871x + 41.26 14. Expand: (2x + 3y)2
P(16) = -0.01774(16)2 + 0.7871(16) + 41.26
P(16) ≈ 49.3 million Solution:

8. Find each of the following for the (2x + 3y)2 = (2x)2 + 2(2x)(3y) + (3y)2
polynomial functions defined by = 4x2 + 12xy + 9y2
f(x) = x2 – 3x + 7 and g(x) = -3x2 – 7x + 7
15. Find each product.
a. (f + g)(x)
a. [(3x - 2) + 5y][(3x - 2) – 5y]

Marvin Kalngan, Civil Engineer, Plumbing Engineer, AutoCAD Expert, College Lecturer
Algebra – Mathematics of Engineering 34
2
x − 3x + 8
Solution: x 2 + 3x + 1 )x 4 + 0x 3 + 0x 2 + 0x − 16
−(x 4 + 3x 3 + x 2 )
We know that a2 – b2 = (a + b)(a - b), so
−3x 3 − x 2 + 0x
[(3x - 2) + 5y][(3x - 2) – 5y]
−(−3x 3 − 9x 2 − 3x)
= (3x - 2)2 – (5y)2
= 9x2 – 12x + 4 – 25y2 8x 2 + 3x − 16
−(8x 2 + 24x + 8)
b. [(2z + r) + 1]2 −21x − 24
answer:
Solution: x 2 − 3x + 8 remainder − 21x − 24
or it can be written as:
[(2z + r) + 1]2 −21x−24
x 2 − 3x + 8 + x2+3x+1
= (2z + r)2 + 2(2z + r)(1) + 12
= 4z2 + 4zr + r2 + 4z + 2r + 1 Steps:
1. Divide x4 by x2. The answer is x2. Write
c. (m + n)3 x2 above 0x2.
2. Multiply x2 by the divisor (x2 + 3x + 1).
Solution: The answer is x4 + 3x3 + x2.
3. Subtract (x4 + 3x3 + x2) from (x4 + 0x3 +
(m + n)3 = (m + n)2(m + n) 0x2). The answer is -3x3 – x2. Now bring
= (m2 + 2mn + n2)(m + n) down 0x.
= m3 + 2m2n + n2m + m2n + 2mn2 + n3 4. Divide -3x3 by x2. The answer is -3x.
= m3 + 3m2n + 3mn2 + n3 Write -3x above 0x.
5. Multiply -3x by the divisor (x2 + 3x + 1).
d. (2a + b)4 The answer is -3x3 – 9x2 – 3x.
6. Subtract -3x3 – 9x2 – 3x from -3x3 – x2 +
Solution: 0x. The answer is 8x2 + 3x. Now bring down
– 16.
(2a + b)4 = (2a + b)2(2a + b)2 7. Divide 8x2 by x2. The answer is 8. Write 8
= (4a2 + 4ab + b2)(4a2 + 4ab + b2) above -16. Multiply 8 by the divisor (x2 + 3x
= 16a4 + 16a3b + 4a2b2 + 16a3b 16a2b2 + + 1). The answer is 8x2 + 24x + 8.
4ab3 + 4a2b2 + 4ab3 + b4 8. Subtract 8x2 + 24x + 8 from 8x2 + 3x –
= 16a4 + 32a3b + 24a2b2 + 8ab3 + b4 16. The answer is -21x – 24. This is now the
remainder.
15. Let f(x) = 3x + 4 and g(x) = 2x2 + x, find
(fg)(x) and (fg)(-1). 19. Divide x2 – 3x – 10 by x + 2.

Solution: Solution:
Divide by long division.
a. (fg)(x) = f(x)×g(x) x−5
2
x + 2 )x − 3x − 10
= (3x + 4)(2x2 + x)
= 6x3 + 3x2 + 8x2 + 4x −(x 2 + 2x)
= 6x3 + 11x2 + 4x −5x − 10
−(−5x − 10)
b. (fg)(-1) = 6(-1)3 + 11(-1)2 + 4(-1) 0
= -6 + 11 – 4
=1 Steps:
15x2 −12x+6
16. Divide. 3
1. x2 divided by x is x. Write x above -3x.
2. Multiply x by (x + 2). The answer is (x2 +
Solution: 2x). Write the answer as shown above.
15x2 −12x+6 15x2 12x 6 3. Subtract (x2 + 2x) from x2 – 3x. The
3
= − + 3 = 5x2 – 4x + 2 answer is -5x. Now bring down -10.
3 3
4. Divide -5x by x. The answer is -5. Write
5x3 −9x2 +10x
17. Divide. -5 above -10.
5x2
5x3 −9x2 +10x 5x3 9x2 10x 9 2 5. Multiply -5 by (x + 2). The answer is (-5x
5x2
= 5x2 − 5x2 + 5x2 = 5x - 5 + x - 10). Subtract this from -5x – 10. The
answer is 0 (zero). So the quotient is x – 5.

18. Divide x4 – 16 by x2 + 3x + 1. Factoring


Solution: 1. Factor out the greatest common factor.
Marvin Kalngan, Civil Engineer, Plumbing Engineer, AutoCAD Expert, College Lecturer
Algebra – Mathematics of Engineering 35
Another Solution:
a. 10x – 20
Factor out a.
Solution: -a3 + 3a2 – 5a = a(-a2 + 3a - 5)

10x – 20 = 10(x - 2) 3. Factor: 3x – 3y – ax + ay

b. 56y + 35w Solution:

Solution: 3x – 3y – ax + ay
= (3x – 3y) – (ax - ay) Group the terms.
56y + 35w = 7(8y + 5w) = 3(x - y) – a(x - y) Factor out 3 and a.
= (x - y)(3 - a) Factor out x – y.
c. 9x2 + 12x3
4. Factor: 6ax + 12bx + a + 2b
Solution:
Solution:
2 3 2
9x + 12x = 3x (3 + 4x)
6ax + 12bx + a + 2b
4 3 5
d. 32m – 24m + 40m = (6ax + 12bx) + (a + 2b) Group the terms
= 6x(a + 2b) + (a + 2b) Factor each group
Solution: = (a + 2b)(6x + 1) Factor out a + 2b

32m4 – 24m3 + 40m5 = 8m3(4m – 3 + 5m2) 5. Factor: x2y2 – 10 – 2y2 + 5x2

e. 25x2y3 + 30y5 – 15x4y7 Solution:

Solution: x2y2 – 10 – 2y2 + 5x2


= (x2y2 + 5x2) – (2y2 + 10) Rearrange and
25x2y3 + 30y5 – 15x4y7 group the terms.
= 5y3(5x2 + 6y2 – 3x4y4) = x2(y2 + 5) – 2(y2 + 5) Factor out the
common factors in each group.
2. Factor out the greatest common factor. = (y2 + 5)(x2 - 2) Factor out y2 + 5.

a. (x - 5)(x + 6) + (x - 5)(2x + 5) 6. Factor: x2 + 2x – 35

Solution: Solution:
Find pairs of integers whose product is -35.
The greatest common factor is x – 5. Factor -35(1)
out x – 5. 35(-1)
(x - 5)(x + 6) + (x - 5)(2x + 5) 7(-5)
= (x – 5)[(x + 6) + (2x + 5)] 5(-7)
= (x - 5)(x + 6 + 2x + 5)
= (x - 5)(3x + 11) Write the sums of those integers.
-35 + 1 = -34
b. p(r + 2s)2 – q(r + 2s)3 35 – 1 = 34
7 – 5 = 2 (Coefficient of the middle term)
Solution: 5 – 7 = -2

The greatest common factor is (r + 2s)2. The integers 7 and -5 have the necessary
Factor out (r + 2s)2. product and sum, so
p(r + 2s)2 – q(r + 2s)3
= (r + 2s)2[p - q(r + 2s)] x2 + 2x – 35 = (x + 7)(x - 5)
= (r + 2s)2(p - qr – 2qs)
7. Factor x2 + 6ax – 16a2.
c. -a3 + 3a2 – 5a
Solution:
Solution: Find pairs of integers whose product is
-16a2.
Factor out -a. 16a(-a)
-a3 + 3a2 – 5a = -a(a2 – 3a + 5) -16a(a)
8a(-2a)

Marvin Kalngan, Civil Engineer, Plumbing Engineer, AutoCAD Expert, College Lecturer
Algebra – Mathematics of Engineering 36
-8a(2a) Since a = 2, b = 5 and c = -12, and the
-4a(4a) product ac is 2(-12) = -24. Find two integers
whose product is -24 and whose sum is 5.
Write the sums of the pairs of expressions, The necessary integers are 8 and -3, so we
looking for a sum of 6a. write 5y as 8y – 3y.
16a – a = 15a
-16a + a = -15a 2y2 + 5y – 12 = 2y2 + 8y – 3y – 12
8a – 2a = 6a = (2y2 + 8y) – (3y + 12)
-8a + 2a = -6a = 2y(y + 4) - 3(y + 4)
-4a + 4a = 0 = (y + 4)(2y - 3)

The expressions 8a and -2a have the Now replace y with x + 3.


necessary product and sum, so (y + 4)(2y - 3)
x2 + 6ax – 16a2 factors as (x + 8a)(x – 2a). = (x + 3 + 4)[2(x + 3) – 3]
= (x + 7)(2x + 6 - 3)
Check by multiplying the factors: = (x + 7)(2x + 3)
(x + 8a)(x – 2a) = x2 – 2ax + 8ax – 16a2
= x2 + 6ax – 16a2 (original answer: (x + 7)(2x + 3)
polynomial)
11. Factor 6x4 + 11x2 + 3.
3 2
8. Factor 16y – 32y – 48y.
Solution:
Solution:
Let y = x2, so y2 = x4
Factor out the GCF, 16y. 6x4 + 11x2 + 3 = 6y2 + 11y + 3
16y3 – 32y2 – 48y = 16y(y2 – 2y - 3)
Since a = 6, b = 11 and c = 3, and the
Now factor y2 – 2y – 3. Look for two product ac is 6(3) = 18. Find two integers
integers whose product is -3 and whose sum whose product is 18 and whose sum is 11.
is -2. The necessary integers are -3 and 1. The necessary integers are 9 and 2, so we
So write 11y as 9y + 2y.
16y3 – 32y2 – 48y = 16y(y2 – 2y - 3)
= 16y(y - 3)(y + 1) 6y2 + 11y + 3 = 6y2 + 9y + 2y + 3
= (6y2 + 9y) + (2y + 3) [group the terms]
answer: 16y(y - 3)(y + 1) = 3y(2y + 3) + (2y + 3) [factor by grouping]
= (2y + 3)(3y + 1) [factor out common
9. Factor 12r2 – 5r – 2. factor]

Solution: Now substitute x2 for y.

Since a = 12, b = -5 and c = -2, and the (2y + 3)(3y + 1) = (2x2 + 3)(3x2 + 1)
product ac is 12(-2) = -24. Find two integers
whose product is -24 and whose sum is -5 answer: (2x2 + 3)(3x2 + 1)
(since the middle term, b, has coefficient of
-5). The necessary integers are -8 and 3, so
we write -5r as 3r – 8r. Special Factoring
12r2 – 5r – 2 = 12r2 + 3r – 8r - 2 Difference of Squares:
= (12r2 + 3r) – (8r + 2) [group the terms] x2 - y2 = (x + y)(x - y)
= 3r(4r + 1) – 2(4r + 1) [factor by grouping]
= (4r + 1)(3r - 2) [factor out common Perfect Square Trinomial:
factor] x2 + 2xy + y2 = (x + y)2
x2 - 2xy + y2 = (x - y)2
answer: (4r + 1)(3r - 2)
Difference of Cubes:
10. Factor 2(x + 3)2 + 5(x + 3) - 12. x3 – y3 = (x - y)(x2 + xy + y2)

Solution: Sum of Cubes:


x3 + y3 = (x + y)(x2 - xy + y2)
Let (x + 3) be y.
2(x + 3)2 + 5(x + 3) – 12 = 2y2 + 5y – 12 1. Factor each polynomial using “Difference
of Squares”

Marvin Kalngan, Civil Engineer, Plumbing Engineer, AutoCAD Expert, College Lecturer
Algebra – Mathematics of Engineering 37
3
a. y – 8
2 2 2
a. x – 36 = x – 6 = (x + 6)(x - 6)
Solution:
2 2 2 2
b. 4a – 64 = 4(a - 16) = 4(a - 4 ) y3 – 8 = y3 – 23 = (y - 2)(y2 + 2y + 22)
= 4(a + 4)(a - 4) = (y - 2)(y2 + 2y + 4)

c. 16x2 – 49y2 = (4x - 7y)(4x + 7y) b. 27x3 – 8y3

d. 81k2 - (a - 2)2 = (9k)2 - (a - 2)2 Solution:


= [9k + (a - 2)][9k - (a - 2)] 27x3 – 8y3 = (3x)3 - (2y)3
= (9k + a - 2)(9k - a + 2) = (3x – 2y)[(3x)2 + (3x)(2y) + (2y)2]
= (3x – 2y)(9x2 + 6xy + 4y2)
e. x4 – 81 = (x2)2 - 92 = (x2 - 9)(x2 + 9)
= (x2 - 32)(x2 + 9) 5. Factor each polynomial using “Sum of
= (x - 3)(x + 3)(x2 + 9) Cubes”

2. Factor each perfect square trinomial. a. y3 + 8

a. 144p2 – 120p + 25 Solution:


Here, 144p2 = (12p)2 and 25 = 52. The sign y3 + 8 = y3 + 23 = (y + 2)(y2 - 2y + 22)
on the middle term is negative (-), so if = (y + 2)(y2 - 2y + 4)
144p2 – 120p + 25 is a perfect square
trinomial, the factored form will have to be b. 27x3 + 8y3
(12p - 5)2.
Solution:
Determine twice the product of the two 27x3 + 8y3 = (3x)3 + (2y)3
terms to see if this is correct. = (3x + 2y)[(3x)2 - (3x)(2y) + (2y)2]
2(12p)(-5) = -120p = (3x + 2y)(9x2 - 6xy + 4y2)

This is the middle term of the given


trinomial. Solving Equations by Factoring
144p2 – 120p + 25 factors as (12p - 5)2 1. (x + 6)(2x - 3) = 0

b. (r + 5)2 + 6(r + 5) + 9 Solution:


The product of x + 6 and 2x – 3 is zero. By
Solution: the zero-factor property, the following must
(r + 5)2 + 6(r + 5) + 9 hold true.
= (r + 5)2 + 6(r + 5) + 32 x + 6 = 0 or 2x – 3 = 0
= [(r + 5) + 3]2
Solve each of these equations.
c. m2 – 8m + 16 – p2 x + 6 = 0; x = -6
or
Solution: 3
2x – 3 = 0; 2x = 3; x = 2
Since there are four terms, use factoring by
grouping. The first three terms here form a
perfect square trinomial. Group them Check:
together, and factor as follows. let x = -6
(x + 6)(2x - 3) = 0
m2 – 8m + 16 – p2 (-6 + 6)[2(-6) – 3] = 0
= (m2 – 8m + 16) – p2 0=0
Factor the perfect square trinomial. 3
= (m – 4)2 – p2 let x = 2
Factor the difference of squares. 3 3 3
�2 + 2� ��2 × 2 − 3�� = 0
= [(m – 4) + p] [(m – 4) - p]
= (m – 4 + p) (m – 4 - p) 0=0

answer: (m – 4 + p) (m – 4 - p) Both solutions check, so the solution set is


𝟑𝟑
�−𝟔𝟔, �.
𝟐𝟐
4. Factor each polynomial using “Difference
of Cubes” 2. Solve the equation 2x2 + 3x = 2.

Marvin Kalngan, Civil Engineer, Plumbing Engineer, AutoCAD Expert, College Lecturer
Algebra – Mathematics of Engineering 38
Solution: 6. A piece of sheet metal is in the shape of a
2x2 + 3x = 2 parallelogram. The longer sides of the
2x2 + 3x – 2 = 0 parallelogram are each 8 m longer than the
(2x - 1)(x + 2) = 0 distance between them. The area of the piece
2x – 1 = 0 is 48 m2. Find the length of the longer sides
2x = 1 and the distance between them.
1
x=2
Solution:
or

x+2=0
x = -2

3. Solve 4x2 – 20x = 0.

Solution:
Let x = distance between the longer sides, so
4x2 – 20x = 0
x + 8 = length of each longer sides
4x(x - 5) = 0
4x = 0 or x – 5 = 0
A[parallelogram] = base × height
x = 0 or x = 5
48 = (x + 8)(x)
48 = x2 + 8x
The solution set is {0, 5}.
x2 + 8x – 48 = 0
(x + 12)(x - 4) = 0
4. Solve (2x + 1)(x + 1) = 2(1 - x) + 6
x + 12 = 0 or x – 4 = 0
x = -12 or x = 4
Solution:
(2x + 1)(x + 1) = 2(1 - x) + 6
Since length can never be negative, x = 4 m.
2x2 + 2x + x + 1 = 2 – 2x + 6
It follows that: x + 8 = 4 + 8 = 12 m
2x2 + 3x + 1 = 8 – 2x
2x2 + 5x - 7 = 0
answer: 12 m and 4 m
Since a = 2, b = 5 and c = -7, and the
7. A small rocket is launched vertically
product ac is 2(-7) = -14. Find two integers
upward from ground level with an initial
whose product is -14 and whose sum is 5.
velocity of 128 ft per sec, its height in feet
The necessary integers are 7 and -2, so we
after t seconds is a function defined by
write 5x as 7x – 2x.
h(t) = -16t2 + 128t
If air resistance is neglected, after how many
2x2 + 5x - 7 = 0
seconds will the rocket be 220 ft above the
2x2 + 7x – 2x - 7 = 0
ground?
(2x2 + 7x) - (2x + 7) = 0
x(2x + 7) - (2x + 7) = 0
Solution:
(2x + 7)(x - 1) = 0
h(t) = -16t2 + 128t
2x + 7 = 0 or x – 1 = 0
7 220 = -16t2 + 128t
x = − 2 or x = 1 16t2 - 128t + 220 = 0
4t2 – 32t + 55 = 0
𝟕𝟕
The solution set is �− 𝟐𝟐 , 𝟏𝟏�. (2t - 5)(2t - 11) = 0
2t – 5 = 0 or 2t – 11 = 0
t = 2.5 sec or t = 5.5 sec
5. Solve -x3 + x2 = -6x.
The rocket will reach a height of 220 ft
Solution:
twice: on its way up at 2.5 sec and again on
-x3 + x2 = -6x
its way down at 5.5 sec.
-x3 + x2 + 6x = 0
Multiplying both sides by -1 gives
x3 - x2 - 6x = 0
x(x2 – x - 6) = 0 Rational Expressions and Functions
x(x - 3)(x + 2) = 0
x = 0 or x – 3 = 0 or x + 2 = 0 1. Write each rational expression in lowest
x = 0 or x = 3 or x = -2 terms.

(x+5)(x+2)
The solution set is {-2, 0, 3}. a. (x+2)(x−3)

Marvin Kalngan, Civil Engineer, Plumbing Engineer, AutoCAD Expert, College Lecturer
Algebra – Mathematics of Engineering 39
Solution:
Solution: x + 2 cancels out 2z 5z2 2z 18 4
(x+5)(x+2) x+5 9
÷ = × 5z2 = 5z
18 9
(x+2)(x−3)
= x−3
𝟒𝟒
answer: 𝟓𝟓𝟓𝟓
x+5
answer: x−3
8k−16 3k−6
b. 3k
÷ 4k2
a2 −a−6
b. a2 +5a−6
Solution:
Solution: Factor the numerator and 8k−16 3k−6 8k−16 4k2
3k
÷ 4k2
= 3k
× 3k−6
denominator. 8(k−2) 4k2
= × 3(k−2)
3k
a2 −a−6 (a−3)(a+2) a−3 8 4k2 32k
a2 +5a−6
= (a+3)(a+2) = a+3 = 3k × =
3 9

𝐚𝐚−𝟑𝟑 𝟑𝟑𝟑𝟑𝟑𝟑
answer: 𝐚𝐚+𝟑𝟑 answer: 𝟗𝟗

x3 −27 4. Add or subtract the rational expressions.


c. x−3
3y x 𝟑𝟑𝟑𝟑+𝐱𝐱
Solution: Factor the difference of cubes in a. 5
+5= 𝟓𝟓
the numerator.
7 11 7−11 −4 −𝟐𝟐
b. 2r2 − 2r2 = 2r2
= 2r2 = 𝐫𝐫 𝟐𝟐
x3 −27 x3 −33 (x−3)�x2 +3x+32 �
x−3
= =
x−3 x−3
(x−3)�x2 +3x+9� 4 x 4+x 4+x
= = x2 + 3x + 9 c. x2 +2x−8 + x2+2x−8 = x2 +2x−8 = (x+4)(x−2)
x−3
𝟏𝟏
= 𝐱𝐱−𝟐𝟐
answer: 𝐱𝐱 𝟐𝟐 + 𝟑𝟑𝟑𝟑 + 𝟗𝟗
5. Find the least common denominator
2. Multiply the rational expressions. (LCD) for each group of denominators.
5p−5 3p2
a. × 10p−10 a. 5xy2, 2x3y
p
5xy2 = 5 ∙ x ∙ y 2 [factored form]
2x3y = 2 ∙ x 3 ∙ y [factored form]
Solution:
5p−5 3p2 5(p−1) 3p2
p
× 10p−10 = × 10(p−1) Greatest exponent of x is 3 and greatest
p
5�3p2 � 3p exponent of y is 2. So the LCD is
= 10p
= 2
LCD = 5 ∙ 2 ∙ x 3 ∙ y 2 = 10x3y2
𝟑𝟑𝟑𝟑
answer: 𝟐𝟐
b. k – 3, k
m2 +2m−15 m2 −4 Each denominator is already factored, so the
b. m2 −5m+6
∙ m2+5m LCD is the product of k – 3 and k.

Solution: LCD = k(k - 3)


m2 +2m−15 m2 −4
m2 −5m+6 m2 +5m

(m+5)(m−3) (m−2)(m+2)
c. y2 – 2y – 8, y2 + 3y + 2
m+2
= (m−3)(m−2) ∙ = y2 – 2y – 8 = (y - 4)(y + 2) [factored form]
m(m+5) m
y2 + 3y + 2 = (y + 2)(y + 1) [factored form]
𝐦𝐦+𝟐𝟐
answer: 𝐦𝐦 LCD = (y - 4)(y + 2)(y + 1)

3. Divide the rational expressions. 6. Add or Subtract the Rational Expressions.

To divide two rational expressions, multiply 5 3


a. 2p + 8p
the first (the dividend) by the reciprocal of
the second (the divisor). The LCD for 2p and 8p is 8p, so
5 3 5 4 3 20 3
2z 5z2 + 8p = 2p ∙ 4 + 8p = 8p + 8p
a. 9
÷ 2p
18
20+3 𝟐𝟐𝟐𝟐
= 8p
= 𝟖𝟖𝟖𝟖

Marvin Kalngan, Civil Engineer, Plumbing Engineer, AutoCAD Expert, College Lecturer
Algebra – Mathematics of Engineering 40
2p(p−1)+5 2p2 −2p+5
p−1 p−1
6
b. r − r−3
5 = 3p2 −2
= 3p2 −2
p p
The LCD is r(r - 3). 2p2 −2p+5 3p2 −2
= p−1
÷ p
6 5 6 r−3 5 r 6(r−3) 5r 2p2 −2p+5 p 2p3 −2p2 +5p
r
− r−3 = r ∙ r−3 − r−3 ∙ r = − r(r−3) = × 3p2 −2 = (p−1)(3p2−2)
r(r−3) p−1
6(r−3)−5r 𝐫𝐫−𝟏𝟏𝟏𝟏 𝟐𝟐𝟐𝟐𝟑𝟑 −𝟐𝟐𝟐𝟐𝟐𝟐 +𝟓𝟓𝟓𝟓
= r(r−3)
= 𝐫𝐫(𝐫𝐫−𝟑𝟑) = 𝟑𝟑𝟑𝟑𝟑𝟑 −𝟐𝟐𝟐𝟐−𝟑𝟑𝟑𝟑𝟐𝟐 +𝟐𝟐

1 1
c. q−1 − q+1 8. Simplify the following rational
The LCD is (q - 1)(q + 1). expressions with negative exponents.

1 1 1 q+1 1 q−1 m−1 +p−2


q−1
− q+1 = q−1 ∙ q+1 − q+1 ∙ q−1 a. 2m−2 −p−1
q+1 q−1
= (q−1)(q+1) − (q−1)(q+1) 1
(q+1)−(q−1) q+1−q+1
Note that a−n = an. So
= (q−1)(q+1)
= (q−1)(q+1)
𝟐𝟐 1 1
= m−1 +p−2 +
m p2
(𝐪𝐪−𝟏𝟏)(𝐪𝐪+𝟏𝟏) = 2 1 . In here the LCD of the
2m−2 −p−1 −
m2 p
m+4 2m−3 numerator is mp2 and the denominator is
d. m2−2m−3 − m2−5m+6
pm2. So
Factor each denominator.
1 1 1 p2 1 m p2 m
m+4 2m−3 m−1 +p−2 + ∙ + ∙ +
m p2 m p2 p2 m mp2 mp2
m2 −2m−3 m2 −5m+6
− = = =
2m−2 −p−1 2 1
− 2 p 1 m2 2p m2
m+4 2m−3 m2 p ∙ − ∙ −
m2 p p m2 pm2 pm2
= (m−3)(m+1) − (m−3)(m−2) 2
p +m
mp2 p2 +m 2p−m2
= 2p−m2
= mp2
÷ pm2
The LCD is (m − 3)(m + 1)(m − 2). pm2
p2 +m pm2 �p2 +m��pm2 �
m+4 2m − 3 = mp2
× 2p−m2 = (mp2)(2p−m2)
− p3 m2 +pm3 pm�p2 m+m2 �
m2 − 2m − 3 m2 − 5m + 6 = 2mp3 −m3p2 = pm(2p2 −m2p)
m+4 2m−3
= (m−3)(m+1) − (m−3)(m−2) 𝐩𝐩𝟐𝟐 𝐦𝐦+𝐦𝐦𝟐𝟐
m+4 m−2 2m−3 m+1 = 𝟐𝟐𝐩𝐩𝟐𝟐 −𝐦𝐦𝟐𝟐 𝐩𝐩
= ∙
(m−3)(m+1) m−2
− ∙
(m−3)(m−2) m+1
(m+4)(m−2) (2m−3)(m+1)
= (m−3)(m+1)(m−2) − (m−3)(m−2)(m+1) 1 2 1 y 2 x2 y 2x2
x−2 −2y−1 − ∙ − ∙ −
x2 𝑦𝑦 x2 y y x2 yx2 yx2
(m+4)(m−2)−(2m−3)(m+1) b. = = =
= y−2x2 y−2x2 y−2x2 y−2x2
(m−3)(m+1)(m−2) y−2x2
m2 −2m+4m−8−�2m2 +2m−3m−3� yx2 y−2x2
= = = ÷ y − 2x 2
(m−3)(m+1)(m−2) y−2x2 yx2
m2 −2m+4m−8−2m2 −2m+3m+3 y−2x2 1 𝟏𝟏
= = × y−2x2 = 𝐲𝐲𝐲𝐲𝟐𝟐
(m−3)(m+1)(m−2) yx2
−𝐦𝐦𝟐𝟐 +𝟑𝟑𝟑𝟑−𝟓𝟓
= (𝐦𝐦−𝟑𝟑)(𝐦𝐦+𝟏𝟏)(𝐦𝐦−𝟐𝟐) 2 3 7
9. Solve. x − 2 = 2x
7. Simplify each complex fraction.
Solution:
x+1 The LCD of the denominators is 2x. So we
x+1 x−1 x+1 2x 𝟐𝟐(𝐱𝐱+𝟏𝟏)
a. x
x−1 = x
÷ 2x
= x
× x−1 = multiply both sides of the equation by 2x to
𝐱𝐱−𝟏𝟏
2x remove denominators.
1 𝑦𝑦 1 2𝑦𝑦 1 2𝑦𝑦+1
2+ 2∙ + + 2𝑦𝑦+1 3𝑦𝑦−2 2 3 7
y 𝑦𝑦 y 𝑦𝑦 y 𝑦𝑦
b. 2 = 𝑦𝑦 2 = 3𝑦𝑦 2 = 3𝑦𝑦−2 = ÷ �x − 2 = 2x� 2𝑥𝑥
3− 3∙ − − 𝑦𝑦 𝑦𝑦
y 𝑦𝑦 y 𝑦𝑦 y 𝑦𝑦
4 – 3x = 7
5 4 – 7 = 3x
2p+
p−1 -3 = 3x
c. 2
3p− -1 = x
p
Solution:
The LCD of the numerator is p – 1 and the The solution set is {-1}.
LCD of the denominator is p. So
2 1 6x
10. Solve. 3x+1 = x − 3x+1
5 p−1 5 2p(p−1) 5
2p+ 2p∙ + +
p−1 p−1 p−1 p−1 p−1
2 = p 2 = 3p2 2
3p− 3p∙ − − Solution:
p p p p p

Marvin Kalngan, Civil Engineer, Plumbing Engineer, AutoCAD Expert, College Lecturer
Algebra – Mathematics of Engineering 41
Divide both sides by (q – f) to isolate p on
2 1 6x
3x+1
= x − 3x+1 the left side of the equation.
p(q−f) fq
2 6x
+ 3x+1 = x
1 = q−f
q−f
3x+1 𝐟𝐟𝐟𝐟
2+6x 1 p = 𝐪𝐪−𝐟𝐟
3x+1
=x
Cross multiply.
x(2 + 6x) = 3x + 1 12. In 2008, about 15 of every 100
2x + 6x2 = 3x + 1 Americans had no health insurance. The
6x2 – x – 1 = 0 population at that time was about 302
a = 6, b = -1, c = -1 million. How many million Americans had
ac = 6(-1) = -6 no health insurance?
Find two integers whose product is -6 and
sum is -1. The necessary integers are -3 and Solution:
2. So we write -x as -3x + 2x. Let x = number of Americans that had no
6x2 – 3x + 2x – 1 = 0 health insurance
(6x2 – 3x) + (2x – 1) = 0
3x(2x – 1) + (2x - 1) = 0 By ratio and proportion, we have
x 15
(2x – 1)(3x + 1) = 0
302000000
= 100
2x – 1 = 0 or 3x + 1 = 0 x = 45,300,000
1 1
x = 2 or x = − 3
13. Marvin’s car uses 10 gal of gasoline to
Check: travel 340 km. He has 5 gal of gasoline in
1 the car, and he wants to know how much
x=2
2 1 6x more gasoline he will need to drive 1030
3x+1
= x − 3x+1 km. If we assume the car continues to use
1
2 1 6� �
2
gasoline at the same rate, how many more
1 = 1 − 1 gallons will he need?
3� �+1 2
3� �+1
2 2
2 1 3
3 = 1 −3
2
+1
2 2
+1 Solution:
4 4 Let x = additional number of gallons of gas
5
=5
needed
1
Since the left side is equal to the right side, 2 By ratio and proportion, we have
x+5 10
is a solution.
1030
= 340
1
340(x + 5) = 10(1030)
x = −3 340x + 1700 = 10300
2 1 6x x = 25. 3 gallons
3x+1
= x − 3x+1
1
2 1 6�− �
3
1 = 1 − 1
14. Marvin’s boat goes 10 miles against the
3�− �+1 − 3�− �+1 current in a river in the same time that it
3 3 3
2 −2
0
= −3 − 0 goes 15 miles with the current. If the rate of
We know that division by zero is undefined the current is 3 mph, find the rate of the boat
1 in still water.
so − 3 is not a solution.
Solution:
𝟏𝟏
The solution set is �𝟐𝟐�. Let x be the rate of the boat in still water.
Thus,
1 1 1 x – 3 = rate of the boat against the current
11. Solve for p. f = p + q
x + 3 = rate of the boat with the current

Solution: We know that,


The LCD is fpq, multiply the whole d
v = t ; where v = rate/speed, d = distance,
equation by fpq.
1 1 1 t = time
� f = p + q� fpq
pq = fq + fp Against the current:
10
Combine like terms (terms with p). x−3 = t
pq – fp = fq 10
Factor. t = x−3
p(q - f) = fq
With the current:

Marvin Kalngan, Civil Engineer, Plumbing Engineer, AutoCAD Expert, College Lecturer
Algebra – Mathematics of Engineering 42
10
x+3 = t
15 Roots, Radicals, and Root
t = x+3
Functions
t=t 1. Simplify.
10 15
x−3
= x+3
3
10x + 30 = 15x – 45 a. √64 = 4, because 43 = 64.
x = 15 mph
4
b. √16 = 2, because 24 = 16.
15. Letitia and Kareem are working on a
neighborhood cleanup. Kareem can clean up 3
c. √125 = 5, because 53 = 125.
all the trash in the area in 7 hours, while
Letitia can do the same job in 5 hours. How 3 8 2 2 3 𝟖𝟖
long will it take them if they work together? d. �27 = 3 = �3� = 𝟐𝟐𝟐𝟐

Solution:
1 2. Simplify each root.
rate of Kareem = 7
1 If n is an even positive integer, then
rate of Letitia = 5
n n
time working together = t √a = |a|
fractional part of the job done by Kareem If n is an odd positive integer, then
n n
1
= 7t √a = a
fractional part of the job done by Letitia 6
1 a. �(−3)6 = 3(n is even, use absolute value)
= 5t
5
1 1
b. �(−4)5 = -4 (n is odd)
7
t + 5t = 1
Multiply both sides by 35 to remove 3. Evaluate each exponential.
denominators.
3
1 1
�7 t + 5 t = 1� 35 a. 641⁄3 = √64 = 4
5t + 7t = 35 4
12t = 35 b. −2561⁄4 = − √256 = -4
t = 2.92 hours
c. 1001⁄2 = √100 = 10
16. Sky is paid an hourly wage. One week
3
he worked 43 hours and was paid $795.50. d. 363⁄2 = �361⁄2 � = 63 = 216
How much does he earn per hour?
1 1 1 1 𝟏𝟏
e. 16−3⁄4 = 163⁄4 = 3 = 4 3 = 23 = 𝟖𝟖
Solution: �161⁄4 � � √16�
795.5
In one hour he earns 43 = $18.5.
8 −2⁄3 1 1 1 1 𝟗𝟗
f. �27� = 8 2⁄3
= 2 = 2 2
= 4 = 𝟒𝟒
� � 3 8 � �
17. In the manufacture of a certain medical 27 �� �
27
3 9

syringe, the cost of producing the syringe


varies inversely as the number produced. If
3. Write each exponential as radical.
10,000 syringes are produced, the cost is $2
per syringe. Find the cost per syringe of
producing 25,000 syringes. a. 131⁄2 = √𝟏𝟏𝟏𝟏

𝟐𝟐 𝟓𝟓 𝟑𝟑
b. 6x 2⁄3 − (4x)3⁄5 = 6� √𝐱𝐱� − � √𝟒𝟒𝟒𝟒�
𝟑𝟑
Solution:
Let x = number of syringes produced
c = cost per syringe c. (a2 + b2 )1⁄2 = √𝐚𝐚𝟐𝟐 + 𝐛𝐛 𝟐𝟐
k
c = x ; where k = constant of proportionality 4. Write with only positive exponents.

2 = 10000
k a. 21⁄2 ∙ 21⁄4 = 21⁄2+ 1⁄4 = 𝟐𝟐𝟑𝟑⁄𝟒𝟒
(Product Rule: Add the exponents)
k = 20000
52⁄3 𝟏𝟏
k 20000 b. 57⁄3 = 52⁄3−7⁄3 = 5−5⁄3 = 𝟓𝟓𝟓𝟓⁄𝟑𝟑
c = x = 25000 = $0.8
(Quotient Rule: Subtract the exponents)

Marvin Kalngan, Civil Engineer, Plumbing Engineer, AutoCAD Expert, College Lecturer
Algebra – Mathematics of Engineering 43
a. √24 = √4 ∙ 6 = 𝟐𝟐√𝟔𝟔
4 4 4
�x1⁄2 y2⁄3 � �x1⁄2 � �y2⁄3 � x2 y8⁄3
c. = =
y y y b. √108 = √36 ∙ 3 = 𝟔𝟔√𝟑𝟑
2 8⁄3−1 𝟐𝟐 𝟓𝟓⁄𝟑𝟑
=x y = 𝐱𝐱 𝐲𝐲

−2⁄3
c. √10 No perfect square other than 1
x4 y−6 1
d. �x−2 y1⁄3� = 2⁄3
divides into 10, so √10 cannot be simplified
x4 y−6
� −2 1⁄3� further.
x y
1
= 2⁄3 2⁄3 3 3 𝟑𝟑
�x4 � �y−6 � d. √16 = √8 ∙ 2 = 𝟐𝟐 √𝟐𝟐
2⁄3 1⁄3 2⁄3
�x−2 � �y �
1 1 4 4 𝟒𝟒
= = x8⁄3+4⁄3 y−4−2⁄9 e. − √162 = − √81 ∙ 2 = −𝟑𝟑 √𝟐𝟐
x8⁄3 y−4
x−4⁄3 y2⁄9
𝐲𝐲 𝟑𝟑𝟑𝟑⁄𝟗𝟗 f. √16m3 = √16m2 ∙ m = 𝟒𝟒𝟒𝟒√𝐦𝐦
= 𝐱𝐱 𝟒𝟒
g. �200k 7 q8 = �100 ∙ 2 ∙ (k 3 )2 ∙ k ∙ (q4 )2
e. m3⁄4 �m5⁄4 − m1⁄4 �
= 𝟏𝟏𝟎𝟎𝐤𝐤 𝟑𝟑 𝐪𝐪𝟒𝟒 √𝟐𝟐𝟐𝟐
= �m5⁄4 ��m3⁄4 � − �m1⁄4 ��m3⁄4 �
= m5⁄4+3⁄4 − m1⁄4+3⁄4 3 3
h. �−8x 4 y 5 = �−8x 3 y 3 (xy 2 )
= 𝐦𝐦𝟐𝟐 − 𝐦𝐦 𝟑𝟑
= −𝟐𝟐𝟐𝟐𝟐𝟐�𝐱𝐱𝐲𝐲 𝟐𝟐
5. Write all radicals as exponentials. 4 4
i. − �32y 9 = − �16 ∙ 2 ∙ y 8 ∙ y = −𝟐𝟐𝐲𝐲 𝟐𝟐 𝟒𝟒�𝟐𝟐𝟐𝟐
3 4
a. √x 2 ∙ √4 = x 2⁄3 ∙ x 1⁄4 = x 2⁄3+1⁄4
3
= 𝐱𝐱 𝟏𝟏𝟏𝟏⁄𝟏𝟏𝟏𝟏 9. Multiply. √7 ∙ √2

√x3 x3⁄2 Solution:


b. 3 2
= x2⁄3 = x 3⁄2−2⁄3 = 𝐱𝐱 𝟓𝟓⁄𝟔𝟔 Because they have different indexes, we get
√x
the LCM of 2 and 3. The LCM is 6. Write
1⁄2 1⁄2 each radical as a 6th root.
c. � √z = � √z� = �z 1⁄4 � = 𝐳𝐳 𝟏𝟏⁄𝟖𝟖
4 4
6 6
√7 = 71⁄2 = 73⁄6 = √73 = √343
3 6 6
6. Multiply. √2 = 71⁄3 = 22⁄6 = √22 = √4
3 3 3 𝟑𝟑
a. √3 ∙ √12 = √3 ∙ 12 = √𝟑𝟑𝟑𝟑 Now we can multiply,
3 6 6 6 𝟔𝟔
√7 ∙ √2 = √343 ∙ √4 = √343 ∙ 4 = √𝟏𝟏𝟏𝟏𝟏𝟏𝟏𝟏
4 4 𝟒𝟒
b. 4�8y ∙ √3r 2 = �8y ∙ 3r 2 = �𝟐𝟐𝟐𝟐𝟐𝟐𝐫𝐫 𝟐𝟐
10. Add or subtract to simplify each radical
6 6 6 𝟔𝟔 expression.
c. √10m4 ∙ √5m = √10m4 ∙ 5m = √𝟓𝟓𝟎𝟎𝐦𝐦𝟓𝟓
3 3 3 3
4 5 a. 2 √16 − 5 √54 = 2 √8 ∙ 2 − 5 √27 ∙ 2
d. √2 ∙ √2 cannot be simplified using the 3 3
= 2 ∙ 2√2 − 5 ∙ 3√2
product rule for radicals because the indexes 3 3
4 and 5 are different. = 4 √2 − 15 √2
𝟑𝟑
= −𝟏𝟏𝟏𝟏 √𝟐𝟐
7. Simplify the following using quotient
rule.
75 √8 √25∙3 √2∙4
b. 2�16 + 4 =2 +4
16 √16 𝟒𝟒 √32 √16 √2∙16
a. �25 = = 𝟓𝟓 5√3 2√2
√25 =2 + 4 4√2
4
5√3 5√3 4
7 √7 √𝟕𝟕 = +2= +2
b. �36 = = 2 2
√36 𝟔𝟔 𝟓𝟓√𝟑𝟑 +𝟒𝟒
= 𝟐𝟐
3
3 8 3 −8 √−8 −𝟐𝟐
c. �− 125 = �125 = 3 = 𝟓𝟓
√125
3 3
3 5 3 4 √5 √4
c. 10 �x6 − 3�x9 = 10 3 6
−33
3 √x √x9
3 m6 √m6 𝐦𝐦𝟐𝟐
d. − �125 = − 3 =− 3
√5 √4
3
√125 𝟓𝟓 = 10 − 3 x3
x2
3 3
√5 x √4
8. Simplify the following radicals. = 10 ∙ − 3
x2 x x3

Marvin Kalngan, Civil Engineer, Plumbing Engineer, AutoCAD Expert, College Lecturer
Algebra – Mathematics of Engineering 44
3 3
x √5 √4 5m2 p2 �2p 𝟓𝟓𝐦𝐦𝟐𝟐 �𝟐𝟐𝐩𝐩
= 10 − 3 x3 = =
x3 p5 𝐩𝐩𝟑𝟑
𝟑𝟑 𝟑𝟑
𝟏𝟏𝟎𝟎𝐱𝐱 √𝟓𝟓−𝟑𝟑 √𝟒𝟒
= 𝐱𝐱 𝟑𝟑 3 27
d. �16
11. Multiply the radical expressions.
Solution:
a. �√5 + 3��√6 + 1� Use quotient rule and simplify the numerator
= √5 ∙ √6 + √5 ∙ 1 + 3 ∙ √6 + 3 ∙ 1 and denominator.
= √𝟑𝟑𝟎𝟎 + √𝟓𝟓 + 𝟑𝟑√𝟔𝟔 + 3 3 27
3
√27 3 3
� = 3 = 3 = 3
16 √16 √8∙2 2 √2
b. �7 − √3��√5 + √2�
= 7 ∙ √5 + 7 ∙ √2 - √3 ∙ √5 - √3 ∙ √2 Since 2(4) = 8 is a perfect cube, multiply the
3
= 𝟕𝟕√𝟓𝟓 + 𝟕𝟕√𝟐𝟐 - √𝟏𝟏𝟏𝟏 - √𝟔𝟔 numerator and denominator by √4.
3 3 3 3 𝟑𝟑
3 √4 3 √4 3 √4 3 √4 𝟑𝟑 √𝟒𝟒
c. �√10 + √3��√10 − √3� 3 ∙3 = 3 = 3 = 2(2) =
2 √2 √4 2 √2∙4 2 √8 𝟒𝟒
= √10√10 - √10√3 + √3√10 - √3√3
= 10 - √30 + √30 – 3 3
e. 1+√2
=7
2 Solution:
d. �√7 − 3� = �√7 − 3��√7 − 3� Multiply the numerator and denominator by
= √7√7 - 3√7 - 3√7 – 3(-3) 1 − √2, the conjugate of the denominator.
= 7 - 6√7 + 9
= 16 - 𝟔𝟔√𝟕𝟕 3 1−√2 3�1−√2� 3−3√2
∙ = �1+ = 1−√2+√2−2
1+√2 1−√2 √2��1−√2 �
3 3 3−3√2
e. �5 − √3��5 + √3� = = −�3 − 3√2�
−1
3 3 3 3
= 5(5) + 5 √3 - 5 √3 - √3� √3� = −𝟑𝟑 + 𝟑𝟑√𝟐𝟐
𝟑𝟑
= 25 - √𝟗𝟗
5
f. 4−√3
13. Rationalize each denominator.
Rationalizing a denominator is the process
Solution:
of removing radicals from a denominator so
Multiply the numerator and denominator by
that the denominator contains only rational
numbers. the conjugate of the denominator, 4 + √3.

3 5 4+√3 5�4+√3� 5�4+√3�


a. ∙ = �4− =
√7 4−√3 4+√3 √3��4+√3� 16+4√3−4√3−3
5�4+√3� 𝟓𝟓�𝟒𝟒+√𝟑𝟑�
= 16−3
= 𝟏𝟏𝟏𝟏
Solution:
Multiply the numerator and denominator by
√2−√3
√7. This is, in effect, multiplying by 1. g.
√5+√3

3 √7 3√7 𝟑𝟑√𝟕𝟕 Solution:


∙ = =
√7 √7 √7√7 𝟕𝟕
Multiply the numerator and denominator by
5√2 the conjugate of the denominator, √5 − √3.
b. √2−√3 √5−√3 �√2−√3��√5−√3�
√5 ∙ =�
√5+√3 √5−√3 √5+√3��√5−√3�
√2√5−√2√3−√3√5+√3√3
Solution: =
5√2 √5 5√2√5 5√10 √5√5−√5√3+√3√5−√3√3
∙ = = = √𝟏𝟏𝟎𝟎 √10−√6−√15+3
√5 √5 √5√5 5 = 5−3
√𝟏𝟏𝟎𝟎−√𝟔𝟔−√𝟏𝟏𝟏𝟏+𝟑𝟑
50m4 = 𝟐𝟐
c. � p5
14. Write each quotient in lowest terms.
Solution:
50m4 √50m4 √50m4 �p5 �p5 √50m4 6+2√5
� = = ∙ = a.
p5 �p5 �p5 �p5 �p5 �p5 4

�50m4 p5 �25∙2m4 p4 ∙p
= = Solution:
p5 p5

Marvin Kalngan, Civil Engineer, Plumbing Engineer, AutoCAD Expert, College Lecturer
Algebra – Mathematics of Engineering 45
6+2√5 2�3+√5� 𝟑𝟑+√𝟓𝟓 Solution:
= =
4 4 𝟐𝟐 Square both sides of the equation.
5y−�8y2 2
b. 6y �√4 − x� = (x + 2)2
4 – x = x2 + 4x + 4
Solution: x2 + 5x = 0
x(x + 5) = 0
5y−�8y2 5y−�4∙2∙y2 5y−2y√2 y�5−2√2� x = 0 or x + 5 = 0
= = = x = 0 or x = -5
6y 6y 6y 6y
𝟓𝟓−𝟐𝟐√𝟐𝟐
= 𝟔𝟔 Check:
x=0
15. Solve √3x + 4 = 8.
√4 − x = x + 2
Solution: √4 − 0 = 0 + 2
Square both sides of the equation. √4 = 2
2 = 2 [True]
2
�√3x + 4 = 8� x = -5
3x + 4 = 64
√4 − x = x + 2
3x = 64 – 4 = 60
x = 20 √4 + 5 = −5 + 2
√9 = −3
Check: 3 = -3 [False]

The solution set is {0}. The other


√3x + 4 = 8
proposed solution, -5, is extraneous.
√3 ∙ 20 + 4 = 8
√64 = 8 18. Solve √x 2 − 4x + 9 = x − 1
8 = 8 [True]
Solution:
Since 20 satisfies the original equation, the
Square both sides.
solution set is {20}.
2
16. Solve √5x − 1 + 3 = 0. �√x 2 − 4x + 9� = (x − 1)2
x2 – 4x + 9 = x2 – 2x + 1
Solution: -4x + 9 = -2x + 1
9 – 1 = -2x + 4x
√5x − 1 + 3 = 0 8 = 2x
4=x
√5x − 1 = −3
Square both sides of the equation.
2 Check:
�√5x − 1� = (−3)2 √x 2 − 4x + 9 = x − 1
5x – 1 = 9 √42 − 4 ∙ 4 + 9 = 4 − 1
5x = 9 + 1 3 = 3 [True]
5x = 10
x=2 The solution set is {4}.
Check:
19. Solve √5x + 6 + √3x + 4 = 2
Solution:
√5x − 1 + 3 = 0 Isolate one radical on one side of the
√5 ∙ 2 − 1 + 3 = 0 equation by subtracting √3x + 4 from each
√9 + 3 = 0 side.
3+3=0 √5x + 6 + √3x + 4 = 2
9 = 0 [False]
√5x + 6 = 2 − √3x + 4
Then square each side.
This false result shows that the proposed 2 2
solution 2 is not a solution of the original �√5x + 6� = �2 − √3x + 4�
equation. It is extraneous The solution set 5x + 6 = 4 −4√3x + 4 + (3x + 4)
is ∅. 5x + 6 = 4 −4√3x + 4 + 3x + 4
5x – 3x + 6 – 4 – 4 = −4√3x + 4
17. Solve √4 − x = x + 2 2x – 2 = −4√3x + 4

Marvin Kalngan, Civil Engineer, Plumbing Engineer, AutoCAD Expert, College Lecturer
Algebra – Mathematics of Engineering 46
Square both sides again.
2
(2x – 2)2 = �−4√3x + 4� c. −√−36 = −𝑖𝑖√36 = −𝟔𝟔𝟔𝟔
4x2 – 8x + 4 = 16(3x + 4)
4x2 – 8x + 4 = 48x + 64 d. √−54 = 𝑖𝑖√54 = 𝑖𝑖√9 ∙ 6 = 𝟑𝟑𝟑𝟑√𝟔𝟔
4x2 – 56x - 60 = 0
4x2 – 56x - 60 = 0 2. Multiply.
x2 – 14x – 15 = 0
(x – 15)(x + 1) = 0 a. √−4 ∙ √−9 = 𝑖𝑖√4 ∙ 𝑖𝑖√9 = 2𝑖𝑖 ∙ 3𝑖𝑖
x – 15 = 0 or x + 1 = 0 = 6𝑖𝑖 2 = 6(−1) = −𝟔𝟔
x = 15 or x = -1
b. √−5 ∙ √6 = 𝑖𝑖√5 ∙ √6 = 𝒊𝒊√𝟑𝟑𝟑𝟑
Check: Use the original equation.
x = 15 3. Divide.
√5x + 6 + √3x + 4 = 2
√5 ∙ 15 + 6 + √3 ∙ 15 + 4 = 2 a.
√−75
=
𝑖𝑖√75
=
𝑖𝑖√25∙3
=
5𝑖𝑖√3
= 𝟓𝟓
√−3 𝑖𝑖√3 𝑖𝑖√3 𝑖𝑖√3
√81 + √49 = 2
9+7=2 √−32 𝑖𝑖√32 𝑖𝑖√16∙2 4𝑖𝑖√2
16 = 2 [False] b. = = = = 𝟐𝟐𝟐𝟐
√8 √8 √4∙2 2√2

x = -1 4. Add the complex numbers.


√5x + 6 + √3x + 4 = 2
�5(−1) + 6 + �3(−1) + 4 = 2 a. (2 + 3𝑖𝑖) + (6 + 4𝑖𝑖) = 2 + 3𝑖𝑖 + 6 + 4𝑖𝑖
√1 + √1 = 2 = 𝟖𝟖 + 𝟕𝟕𝟕𝟕
2 = 2 [True]
b. (4 + 2𝑖𝑖) + (3 − 𝑖𝑖) + (−6 + 3𝑖𝑖)
The solution set is {-1}, 15 must be = 4 + 2𝑖𝑖 + 3 − 𝑖𝑖 − 6 + 3𝑖𝑖
rejected because it is an extraneous = 𝟏𝟏 + 𝟒𝟒𝟒𝟒
solution.
5. Subtract the complex numbers.
3 3
20. √z + 5 = √2z − 6
a. (6 + 5𝑖𝑖) − (3 + 2𝑖𝑖) = 6 + 5𝑖𝑖 − 3 − 2𝑖𝑖
Solution: = 𝟑𝟑 + 𝟑𝟑𝟑𝟑
Cube each side of the equation.
b. (7 − 3𝑖𝑖) − (8 − 6𝑖𝑖) = 7 − 3𝑖𝑖 − 8 + 6𝑖𝑖
3 3 3 3 = −𝟏𝟏 + 𝟑𝟑𝟑𝟑
� √z + 5� = � √2z − 6�
z + 5 = 2z – 6 c. (−9 + 4𝑖𝑖) − (−9 + 6𝑖𝑖)
6 + 5 = 2z – z = −9 + 4𝑖𝑖 + 9 − 6𝑖𝑖
11 = z = −𝟐𝟐𝟐𝟐
Check: 6. Multiply the complex numbers.
3 3
√z + 5 = √2z − 6 Note: 𝑖𝑖 2 = −1
3 3
√11 + 5 = √2 ∙ 11 − 6
3 3
√16 = √16 [True] a. 4𝑖𝑖(2 + 3𝑖𝑖) = 8𝑖𝑖 + 12𝑖𝑖 2 = 8𝑖𝑖 + 12(−1)
= 8𝑖𝑖 − 12 = −𝟏𝟏𝟏𝟏 + 𝟖𝟖𝟖𝟖
The solution set is {11}.
b. (3 + 5𝑖𝑖)(4 − 2𝑖𝑖)
= 12 − 6𝑖𝑖 + 20𝑖𝑖 − 10𝑖𝑖 2
Complex Numbers = 12 + 14𝑖𝑖 − 10(−1)
= 12 + 14𝑖𝑖 + 10
The imaginary number i is defined as = 𝟐𝟐𝟐𝟐 + 𝟏𝟏𝟏𝟏𝟏𝟏
𝑖𝑖 = √−1
2 7. Find each quotient.
where 𝑖𝑖 = −1
8+9𝑖𝑖
1. Write each number as a product of a real a. 5+2𝑖𝑖
number and i.
Solution:
a. √−100 = 𝑖𝑖√100 = 𝟏𝟏𝟎𝟎𝒊𝒊 Multiply the numerator and denominator by
the conjugate of the denominator. The
b. √−2 = 𝒊𝒊√𝟐𝟐 conjugate of 5 + 2𝑖𝑖 is 5 − 2𝑖𝑖.

Marvin Kalngan, Civil Engineer, Plumbing Engineer, AutoCAD Expert, College Lecturer
Algebra – Mathematics of Engineering 47

8+9𝑖𝑖 5−2𝑖𝑖 (8+9𝑖𝑖)(5−2𝑖𝑖) 40−16𝑖𝑖+45𝑖𝑖−18𝑖𝑖 2 Quadratic Equations


∙ = (5+2𝑖𝑖)(5−2𝑖𝑖) =
5+2𝑖𝑖 5−2𝑖𝑖 25−10𝑖𝑖+10𝑖𝑖−4𝑖𝑖 2
40+29𝑖𝑖−18𝑖𝑖 2 40+29𝑖𝑖−18(−1)
= = 1. Solve each equation.
25−4𝑖𝑖 2 25−4(−1)
40+29𝑖𝑖+18 58+29𝑖𝑖 29(2+𝑖𝑖)
= 25+4 = =
29 29 a. 3x2 – 5x – 28 = 0
= 𝟐𝟐 + 𝒊𝒊
Solution:
1+𝑖𝑖
b. 3x2 – 5x – 28 = 0
𝑖𝑖
(3x + 7)(x - 4) = 0
Solution: 3x + 7 = 0 or x – 4 = 0
−𝟕𝟕
Multiply the numerator and denominator by x = 𝟑𝟑 or x = 4
the conjugate of the denominator. The
conjugate of 𝑖𝑖 is −𝑖𝑖. To check, substitute each solution in the
−7
1+𝑖𝑖 −𝑖𝑖 −𝑖𝑖(1+𝑖𝑖) −𝑖𝑖−𝑖𝑖 2 −𝑖𝑖−(−1)
original equation. The solution set is � 3 , 4�.
∙ −𝑖𝑖 = = =
𝑖𝑖 −𝑖𝑖∙𝑖𝑖 −𝑖𝑖 2 −(−1)
−𝑖𝑖+1 Note: You can use the quadratic formula.
= 1
= −𝒊𝒊 + 𝟏𝟏
b. x2 = 5
8. Find each power of i.
Solution:
a. 𝑖𝑖 12 = (𝑖𝑖 2 )6 = (−1)6 = 1
x2 = 5
x2 – 5 = 0
b. 𝑖𝑖 39 = (𝑖𝑖 2 )19 ∙ 𝑖𝑖 = (−1)19 ∙ 𝑖𝑖 = −1 ∙ 𝑖𝑖
�x − √5��x + √5� = 0
= −𝒊𝒊
x − √5 = 0 or x + √5 = 0
9. Perform the indicated operations and 𝐱𝐱 = √𝟓𝟓 or 𝐱𝐱 = −√𝟓𝟓
write the result in standard form.
The solution set is �√5, −√5�.
a. √−18 − √−8
c. 4x2 – 48 = 0
Solution:
√−18 − √−8 = 𝑖𝑖√18 − 𝑖𝑖√8 Solution:
= 𝑖𝑖√9 ∙ 2 − 𝑖𝑖√4 ∙ 2 4x2 – 48 = 0
= 3𝑖𝑖√2 − 2𝑖𝑖√2 4x2 = 48
x2 = 12
= 𝒊𝒊√𝟐𝟐
x = √12 or x = −√12
2 𝐱𝐱 = 𝟐𝟐√𝟑𝟑 or 𝐱𝐱 = −𝟐𝟐√𝟑𝟑
b. �−1 + √−5�
To check, substitute each solution in the
Solution: original equation. The solution set is
(a + b)2 = a2 + 2ab + b2
2 �2√3, −2√3�.
�−1 + √−5�
2
= (−1)2 + 2(−1)�√−5� + �√−5� d. (x − 5)2 = 36
2
= 1 + 2(−1)�𝑖𝑖√5� + �𝑖𝑖√5� Solution:
= 1 − 2𝑖𝑖√5 + 5𝑖𝑖 2 (x − 5)2 = 36
However, 𝑖𝑖 2 = −1. So (x − 5)2 − 36 = 0
= 1 − 2𝑖𝑖√5 + 5(−1) (x − 5)2 − 62 = 0
= 1 − 2𝑖𝑖√5 − 5 [(x − 5) − 6][(x − 5) + 6] = 0
= −𝟒𝟒 − 𝟐𝟐𝟐𝟐√𝟓𝟓 (x − 5 − 6)(x − 5 + 6) = 0
(x − 11)(x + 1) = 0
−25+√−50 x − 11 = 0 or x + 1 = 0
c. x = 11 or x = -1
15

Solution: To check, substitute each solution in the


−25+√−50 −25 √−50 −5 𝑖𝑖√50 original equation. The solution set is
= + 15 = +
15 15 3 15 {11, -1}.
−5 𝑖𝑖√25∙2 −5 5𝑖𝑖√2
= 3
+ = +
15 3 15
𝟓𝟓 𝒊𝒊√𝟐𝟐 2. Solve each equation.
= − 𝟑𝟑 + 𝟑𝟑

Marvin Kalngan, Civil Engineer, Plumbing Engineer, AutoCAD Expert, College Lecturer
Algebra – Mathematics of Engineering 48
formula and simplifying gives the equation’s
2
a. x = -15 solutions

Solution: −b±√b2 −4ac −(−6)±�(−6)2 −4(9)(5)


x= =
x2 = -15 2a 2(9)
6±√36−180 6±√−144 6±𝑖𝑖√144
x = ±√−15 = = =
18 18 18
x = √−15 or x = −√−15 6±12𝑖𝑖 6(1±2𝑖𝑖) 1±2𝑖𝑖 𝟏𝟏 𝟐𝟐
= 18
= = = 𝟑𝟑 ± 𝟑𝟑 𝒊𝒊
𝐱𝐱 = 𝒊𝒊√𝟏𝟏𝟏𝟏 or 𝐱𝐱 = −𝒊𝒊√𝟏𝟏𝟏𝟏 18 3

1 2
The solution set is �𝑖𝑖√15, −𝑖𝑖√15 � The solutions are 3 ± 3 𝑖𝑖 and the solution set
1 2
or �±𝑖𝑖√15 �. is �3 ± 3 𝑖𝑖�.

b. (x + 2)2 = −16 4. Find k so that 9x2 + kx + 4 = 0 will have


exactly one rational solution.
Solution:
(x + 2)2 = −16 Solution:
x + 2 = ±√−16 The equation will have only one rational
x + 2 = √−16 or x + 2 = −√−16 solution if the discriminant is 0 (zero).
x + 2 = 𝑖𝑖√16 or x + 2 = −𝑖𝑖√16
x + 2 = 4𝑖𝑖 or x + 2 = −4𝑖𝑖 From the equation, a = 9, b = k, and c = 4.
x = −𝟐𝟐 + 𝟒𝟒𝟒𝟒 or x = −𝟐𝟐 − 𝟒𝟒𝟒𝟒
b2 – 4ac = 0
The solution set is {−2 + 4𝑖𝑖 , −2 − 4𝑖𝑖 } k2 – 4(9)(4) = 0
or {−2 ± 4𝑖𝑖 }. k2 – 144 = 0
k2 = 144
k = ±12
3. Solve each equation using the quadratic
k = 12 or k = -12
formula.

a. 4x2 = 8x – 1 The equation will have only one rational


Solution: solution if k = 12 or k = -12.
4x2 = 8x – 1
4x2 - 8x + 1 = 0 5. What term should be added to each
binomial so that it becomes a perfect square
a = 4, b = -8 and c = 1 trinomial? Write and factor the trinomial.

Substituting these values into the quadratic a. x2 + 8x


formula and simplifying gives the equation’s
solutions Solution:

−b±√b2 −4ac −(−8)±�(−8)2 −4(4)(1)


To complete the square, we must add a term
x= = to the binomial. The term that should be
2a 2(4)
8±√64−16 8±√48 8±√16∙3 8±4√3 added is the square of half the coefficient of
= = = = x.
8 8 8 8
4�2±√3� 𝟐𝟐±√𝟑𝟑 The coefficient of the x-term in x2 + 8x is 8.
= 8
= 𝟐𝟐 Half of 8 is 4, and 42 = 16. Add 16. The
result is a perfect square trinomial.
𝟐𝟐±√𝟑𝟑
The solutions are and the solution set is
𝟐𝟐
𝟐𝟐±√𝟑𝟑 x2 + 8x + 16 = (x + 4)2
� �.
𝟐𝟐
b. y2 – 7y
b. (9x + 3)(x − 1) = −8
Solution:
Solution:
(9x + 3)(x − 1) = −8 To complete the square, we must add a term
9x 2 − 9x + 3x − 3 = −8 to the binomial. The term that should be
9x 2 − 6x + 5 = 0 added is the square of half the coefficient of
y.
a = 9, b = -6 and c = 5 The coefficient of the y-term in y2 – 7y is -7.
7 7 2 49 49
Half of -7 is − 2, and �− 2� = 4 . Add 4
.
Substituting these values into the quadratic
The result is a perfect square trinomial.

Marvin Kalngan, Civil Engineer, Plumbing Engineer, AutoCAD Expert, College Lecturer
Algebra – Mathematics of Engineering 49
√5 1 √𝟓𝟓+𝟏𝟏
x= +3 =
𝟒𝟒𝟒𝟒 𝟕𝟕 𝟐𝟐 3 𝟑𝟑
y2 – 7y + 𝟒𝟒
= �𝐲𝐲 − 𝟐𝟐� or
√5 1 𝟏𝟏−√𝟓𝟓
3
x=− +3=
3 𝟑𝟑
c. z 2 + 5 z
𝟏𝟏±√𝟓𝟓
The solutions are and the solution set is
Solution: 𝟑𝟑
𝟏𝟏±√𝟓𝟓
� �.
𝟑𝟑
To complete the square, we must add a term
to the binomial. The term that should be 1 1 7
added is the square of half the coefficient of 7. Solve x + x−1 = 12
z.
3 3
The coefficient of the z-term in z 2 + 5 z is 5. Solution:
The LCD is x(x-1)(12). Multiply the whole
3 3 3 2 9 9
Half of 5 is 10, and �10� = 100. Add 100 . equation by the LCD to eliminate
The result is a perfect square trinomial. denominators.

1 1 7
𝟑𝟑 𝟗𝟗 𝟑𝟑 𝟐𝟐 �x + x−1 = 12� [x(x − 1)(12)]
𝐳𝐳 𝟐𝟐 + 𝟓𝟓 𝐳𝐳 + 𝟏𝟏𝟏𝟏𝟏𝟏 = �𝐳𝐳 + 𝟏𝟏𝟏𝟏�
1(x − 1)(12) + 1(x)(12) = 7x(x − 1)
(x − 1)(12) + (x)(12) = 7x 2 − 7x
6. Solve the quadratic equation by
12x − 12 + 12x = 7x 2 − 7x
completing the square: 9x2 – 6x – 4 = 0
24x − 12 = 7x 2 − 7x
Solution: 7x 2 − 31x + 12 = 0
Divide both sides of the equation by 9 to
make the leading coefficient 1. Use the quadratic formula.
9x2 – 6x – 4 = 0
9x2 −6x−4 0 a = 7, b = -31 and c = 12
9
=9
2 6x 4 −b±√b2 −4ac −(−31)±�(−31)2 −4(7)(12)
x − 9
−9=0 x= =
2 4 2a 2(7)
x2 − 3 x − 9 = 0 31±√961−336 31±√625 31±25
= = =
14 14 14

Isolate the variable terms on one side of the 31+25 31−25 𝟑𝟑


equation and the constant term on the other x= 14
= 𝟒𝟒 or x = 14
= 𝟕𝟕
side.
𝟑𝟑
2 4
The solution set is �𝟒𝟒, 𝟕𝟕 �.
x2 − 3 x = 9
8. Compute the discriminant of each
Complete the square. Add the square of half equation. Then determine the number and
the coefficient of x to both sides of the type of solution
equation. a. 3x2 – 8x + 7 = 0
b. 9x2 – 6x + 1 = 0
2 1 4 1
x2 − 3 x + 9 = 9 + 9 c. 3x2 + 4x – 5 = 0
2 1 5
x2 − 3 x + 9 = 9 Solution:
Begin by identifying the values for a, b, and
Factor the resulting perfect square trinomial. c in each equation. Then compute b2 - 4ac,
the discriminant
1 2 5
�x − 3� = 9
a. 3x2 – 8x + 7 = 0
Squaring both sides gives a = 3; b = -8; c = 7
1 5 b2 – 4ac = (-8)2 – 4(3)(7) = 64 – 84 = -20
x − 3 = ±�9

The negative discriminant, -20, shows that


Now solve for x. there are two imaginary solutions. (These
solutions are complex conjugates of each
5 1 √5 1 other.)
x = ±�9 + 3 = ± +3
3
b. 9x2 – 6x + 1 = 0
Marvin Kalngan, Civil Engineer, Plumbing Engineer, AutoCAD Expert, College Lecturer
Algebra – Mathematics of Engineering 50
x=±3
a = 9; b = -6; c = 1 x = 3 or x = -3

b2 – 4ac = (-6)2 – 4(9)(1) = 36 – 36 = 0 answer: 3 mph

The discriminant, 0, shows that there is only The river’s current rate cannot be -3, so the
one real solution. This real solution is a answer is 3 miles per hour.
rational number.
10. Did you know that the size of a
c. 3x2 + 4x – 5 = 0 television screen refers to the length of its
diagonal? If the length of the HDTV screen
a = 3; b = 4; c = -5 shown below is 28 inches and its width is
15.7 inches, what is the size of the screen to
b2 – 4ac = 42 – 4(3)(-5) = 16 + 60 = 76 the nearest inch?

The discriminant, 76, is a positive number


that is not a perfect square. Thus, there are
two irrational solutions. (These solutions are
conjugates of each other.)

9. Marvin’s riverboat averages 12 mph


(miles per hour) in still water. It takes the
boat 1 hour and 4 minutes to go 6 miles
upstream and return. Find the rate of the
current. Solution:
Use the Pythagorean Theorem.
Solution: h = √282 + 15.72 = 32 inches
let V = speed of Marvin’s boat in still water
x = speed of current 11. It takes two painters 4 hours to paint a
tu = time (hours) going upstream room. If each worked alone, one of them
td = time (hours) going downstream could do the job in 1 hour less time than the
t = total time (hours) going upstream and other. How long would it take each painter
downstream to complete the job alone?
= 1 hour and 4 minutes
16
= 15 hours Solution:
d = distance = 6 miles Let x = the number of hours for the slower
painter to complete the job alone. Then the
The current slows down the boat when it is faster painter could do the entire job in
going upstream, so the rate of the boat going (x - 1) hours. The slower person’s rate is
1 1
upstream is its rate in still water less the rate x
and the faster person’s rate is x−1
.
of the current, or V – x = 12 - x (mph). Together, they do the job in 4 hours.

Similarly, the current speeds up the boat as 1 1 1


Formula: A + B = t
it travels downstream, so its rate
downstream is, V + x = 12 + x (mph).
Thus, Substitute:
1 1 1
12 – x = rate upstream in miles per hour x
+ x−1
= 4
12 + x = rate downstream in miles per hour
Multiply both sides by 4(x)(x - 1) to
d
We know that, t = V, where t = time, eliminate denominators.
d = distance and V = speed/rate. 1 1 1
�x + x−1 = 4� [4(x)(x − 1)]
t = tu + td 1(4)(x-1) + 1(4)(x) = 1(x)(x-1)
16 6 6
15
= 12−x + 12+x 4x - 4 + 4x = x2 - x
16 6 6 x2 – 9x + 4 = 0
�15 = 12−x + 12+x� [15(12 − x)(12 + x)]
16(12-x)(12+x) = 6(15)(12+x) + 6(15)(12-x) a = 1, b = -9 and c = 4
16(122 – x2) = 90(12+x) + 90(12-x)
2304 – 16x2 = 1080 + 90x + 1080 - 90x −b±√b2 −4ac −(−9)±�(−9)2 −4(1)(4)
144 = 16x2 x= =
2a 2(1)
x2 = 9 9±√81−16 9±√65
= =
2 2
Marvin Kalngan, Civil Engineer, Plumbing Engineer, AutoCAD Expert, College Lecturer
Algebra – Mathematics of Engineering 51
9+√65 9−√65 Check:
x= ≈ 8.5 or x = ≈ 0.5
2 2 All proposed solutions must be checked in
the original equation.
Only the solution 8.5 makes sense in the
original problem, because if x = 0.5, then x=9
x – 1 = -0.5
which cannot represent the time for the
x + √x = 6
faster worker.
9 + √9 = 6
answer: 8.5 hours and 7.5 hours 9+3=6
12 = 6 [False]
12. Solve x = √6x − 8.
x=4
Solution:
Square both sides of the equation. x + √x = 6
4 + √4 = 6
2 4+2=6
(x)2 = �√6x − 8� 6 = 6 [True]
x2 = 6x – 8
x2 – 6x + 8 = 0 Only the solution 4 checks, so the solution
(x - 2)(x - 4) = 0 set is {4}.
x – 2 = 0 or x – 4 = 0
x = 2 or x = 4 14. Solve the equation x4 – 13x2 + 36 = 0.
Check: Solution:
All proposed solutions must be checked in
the original equation. Let u = x2
x4 – 13x2 + 36 = 0
x=2 u2 – 13u + 36 = 0
x = √6x − 8 (u - 4)(u - 9) = 0
2 = √6 ∙ 2 − 8 u – 4 = 0 or u – 9 = 0
2 = √12 − 8 u = 4 or u = 9
2 = √4 x2 = 4 or x2 = 9
2 = 2 [True] x = ±2 or x = ±3

x=4 The solution set is {±2, ±3}.


x = √6x − 8 Each solution can be verified by substituting
it into the original equation for x.
x = √6 ∙ 4 − 8
x = √24 − 8 15. Solve each equation.
x = √16
x = 4 [True] a. 2(4x − 3)2 + 7(4x − 3) + 5 = 0
Both solutions check, so the solution set is Solution:
{2, 4}. Let y = 4x – 3
2(4x − 3)2 + 7(4x − 3) + 5 = 0
13. Solve x + √x = 6. 2y2 + 7y + 5 = 0
Solution:
Use the quadratic formula.
x + √x = 6 −b±√b2 −4ac −7±�72 −4(2)(5)
√x = 6 − x y= =
2a 2(2)
Square both sides of the equation. −7±√9 −7±3
2 = =
4 4
�√x� = (6 − x)2 −7+3 −7−3
y= or y =
x = 62 – 2(6)(x) + x2 4 4
−4 −10
x = 36 – 12x + x2 y= 4
or y = 4
x2 – 13x + 36 = 0 −5
y = -1 or y = 2
(x - 9)(x - 4) = 0
x – 9 = 0 or x – 4 = 0
x = 9 or x = 4 Substitute:
−5
4x – 3 = -1 or 4x – 3 = 2
4x = 2 or 8x – 6 = -5
𝟏𝟏 𝟏𝟏
x = 𝟐𝟐 or x = 𝟖𝟖

Marvin Kalngan, Civil Engineer, Plumbing Engineer, AutoCAD Expert, College Lecturer
Algebra – Mathematics of Engineering 52
𝐝𝐝𝟐𝟐 𝛑𝛑
1 1
𝐀𝐀 = 𝟒𝟒
The solution set is � 2 , 8�.
c. Solve for x: y = 2x2 + kx
b. 2x 2⁄3 − 11x 1⁄3 + 12 = 0
Solution:
Solution: y = 2x2 + kx
Let y = x 1⁄3 2x2 + kx – y = 0
2x 2⁄3 − 11x 1⁄3 + 12 = 0
2y 2 − 11y + 12 = 0 Use the Quadratic Formula.
a = 2, b = k, and c = -y
Use the quadratic formula. −b±√b2 −4ac −k±�k2 −4(2)(−y)
x= =
−b±√b2 −4ac −(−11)±�(−11)2 −4(2)(12) 2a 2(2)
y= = −k±�k2 +8y
2a 2(2) =
11±√25 11±5 4
= =
4 4
The solutions set is:
11+5 11−5 −𝐤𝐤+�𝐤𝐤𝟐𝟐 +𝟖𝟖𝟖𝟖 −𝐤𝐤−�𝐤𝐤 𝟐𝟐 +𝟖𝟖𝟖𝟖
y= 4
or y = 4
� , �.
𝟒𝟒 𝟒𝟒
3
y = 4 or y = 2
17. Two cars left an intersection at the same
Substitute: time. One heading north, the other due west.
3 Some time later, they were exactly 100
x 1⁄3 = 4 or x 1⁄3 = 2
miles apart. The car headed north had gone
3 3 3 3
�x 1⁄3 � = 43 or �x 1⁄3 � = �2� 20 miles farther than the car headed west.
𝟐𝟐𝟐𝟐 How far had each car traveled?
x = 64 or x = 𝟖𝟖 Solution:
27
After checking, the solution set is � 64, 8 �.

16. Solve each formula for the given


variable.

kFr
a. Solve for v: w = v2

Solution:
Cross multiply first.
kFr
w = v2
wv2 = kFr
Dividing both sides by w, gives
kFr By Pythagorean Theorem,
v2 = w 1002 = x2 + (x + 20)2
Taking the square root of both sides, gives 10000 = x2 + x2 + 40x + 400
kFr √kFr 2x2 + 40x – 9600 = 0
v = ±� w = x2 + 20x – 4800 = 0
√w
Now rationalize the denominator by
multiplying the numerator and denominator Use the Quadratic Formula.
√w a = 1, b = 20 and c = -4800
by .
√w
kFr ±√kFr √w ±√𝐰𝐰𝐰𝐰𝐰𝐰𝐰𝐰 −b±√b2 −4ac −20±�202 −4(1)(−4800)
v = ±� w = ∙ = x= 2a
= 2(1)
√w √w 𝐰𝐰
−20±√19600 −20±140
= =
2 2
4A −20+140 −20−140
b. Solve for A: d = � π x= or x =
2 2
x = 60 or x = -80
Solution:
Square both sides of the equation. Since distance can never be negative,
2 discard the negative solution. The required
4A
(d)2 = �� � distances are:
π
4A
d2 = π x = 60 miles
d2 π = 4A and

Marvin Kalngan, Civil Engineer, Plumbing Engineer, AutoCAD Expert, College Lecturer
Algebra – Mathematics of Engineering 53
x + 20 = 60 + 20 = 80 miles
7+√85 7−√85
x = 2 or x = 2
answer: 60 miles and 80 miles
x = 8.11 or x = -1.11
18. A swimming pool in the shape of a
rectangle is 20 feet wide and 30 feet long. Never on Earth that time is negative. So we
The owner wants to plant a strip of grass of discard the negative solution. The object hits
uniform width around the edge of the pool. the ground about 8.11 seconds after it is
He has enough seed to cover 336 square projected.
feet. How wide will be the strip?
answer: 8.11 seconds
Solution:

Functions and Relations

1. Let f(x) = x2 + 1 and g(x) = 3x + 5. Find


each of the following.

a. (f + g)(1)

area of grass = (30+2x)(20+2x) - (30)(20) Solution:


336 = 600 + 60x + 40x + 4x2 – 600 f(1) = 12 + 1 = 2
4x2 + 100x – 336 = 0 g(1) = 3(1) + 5 = 8
x2 + 25x – 84 = 0
(f + g)(1) = f(1) + g(1) = 2 + 8 = 10
Use the Quadratic Formula.
a = 1, b = 25, and c = -84 b. (f - g)(-3)
−b±√b2 −4ac −25±�252 −4(1)(−84)
x= =
2a 2(1) Solution:
−25±√961 −25±31 f(-3) = (-3)2 + 1 = 10
= =
2 2
g(-3) = 3(-3) + 5 = -4
−25+31 −25−31
x = 2 or x = 2 (f - g)(-3) = f(-3) + g(-3) =10 - (-4) = 14
x = 3 or x = −28
c. (fg)(5)
The width cannot be 28 feet, so the grass
strip should be 3 feet wide. Solution:
f(5) = 52 + 1 = 26
answer: x = 3 feet g(5) = 3(5) + 5 = 20

19. An object is projected upward from the (fg)(5) = [f(5)][g(5)] = (26)(20) = 520
top of a 144 feet building at 112 feet per
seconds, its position (in feet above the f
d. �g� (10)
ground) is given by the equation,
s(t) = -16t2 + 112t + 144
where t is time in seconds after it was Solution:
projected. When does it hit the ground? f(10) = 102 + 1 = 101
g(10) = 3(10) + 5 = 35
Solution:
f f(10) 𝟏𝟏𝟎𝟎𝟎𝟎
When the object hits the ground, its distance �g� (10) = g(10) = 𝟑𝟑𝟑𝟑
above the ground is 0. We must find the
value of t that makes s(t) = 0.
2. Let f(x) = 8x – 9 and g(x) = √2x − 1.
Find each of the following.
-16t2 + 112t + 144 = 0
-t2 + 7t + 9 = 0
a. (f + g)(x)
t2 - 7t - 9 = 0
Solution:
Use the Quadratic Formula.
a = 1, b = -7, and c = -9 (f + g)(x) = f(x) + g(x) = 8x – 9 + √𝟐𝟐𝟐𝟐 − 𝟏𝟏
−b±√b2 −4ac −(−7)±�(−7)2 −4(1)(−9)
x= = b. (f - g)(x)
2a 2(1)
7±√85
= Solution:
2

Marvin Kalngan, Civil Engineer, Plumbing Engineer, AutoCAD Expert, College Lecturer
Algebra – Mathematics of Engineering 54
(f - g)(x) = f(x) - g(x) = 8x – 9 - √𝟐𝟐𝟐𝟐 − 𝟏𝟏 f(-3) = 2(-3) – 1 = -6 – 1 = -7

c. (fg)(x) So,

Solution: (g ∘ f)(−3) = g�f(−3)� = g(−7)


4 4 4
(fg)(x) = f(x)∙g(x) = (8x – 9)� √𝟐𝟐𝟐𝟐 − 𝟏𝟏� = −7−1 = −8 = − 8
𝟏𝟏
f = − 𝟐𝟐
d. �g� (x)
4. Let f(x) = 4x + 1 and g(x) = 2x2 + 5x.
Solution: Find each of the following and simplify.
f f(x) 𝟖𝟖𝟖𝟖 – 𝟗𝟗
�g� (x) = g(x) =
√𝟐𝟐𝟐𝟐−𝟏𝟏
a. (g ∘ f)(x)
2. Let f(x) = x2 - 2x, find the following and
Solution:
simplify each expression.
We know that, (g ∘ f)(x) = g�f(x)�, so
a. f(x + h) – f(x)
(g ∘ f)(x) = g�f(x)� = g(4x + 1 )
Solution: = 2(4x + 1)2 + 5(4x + 1)
f(x + h) – f(x) = 2(16x 2 + 8x + 1) + 20x + 5
= (x + h)2 – 2(x + h) – (x2 – 2x) = 32x 2 + 16x + 2 + 20x + 5
= x2 + 2xh + h2 – 2x – 2h – x2 + 2x = 𝟑𝟑𝟑𝟑𝐱𝐱 𝟐𝟐 + 𝟑𝟑𝟑𝟑𝟑𝟑 + 𝟕𝟕
= h2 + 2xh – 2h
b. (f ∘ g)(x)
f(x+h)−f(x)
b. h Solution:
We know that, (f ∘ g)(x) = f�g(x)�, so
Solution:
f(x+h)−f(x)
h (f ∘ g)(x) = f�g(x)� = f(2x 2 + 5x )
(x + h)2 – 2(x + h) – (x2 – 2x)
= = 4(2x 2 + 5x ) + 1
h
x2 + 2xh + h2 – 2x – 2h – x2 + 2x = 𝟖𝟖𝐱𝐱 𝟐𝟐 + 𝟐𝟐𝟐𝟐𝟐𝟐 + 𝟏𝟏
= h
h2 +2xh−2h 1
= 5. Let f(x) = x and g(x) = √3 − x
h
h(h+2x−2) Find each of the following and simplify.
= h
= h + 2x – 2
a. (f ∘ g)(x)
3. Evaluating composite functions.
4 Solution:
Let f(x) = 2x – 1 and g(x) = x−1. We know that, (f ∘ g)(x) = f�g(x)�, so
Find each of the following.
(f ∘ g)(x) = f�g(x)� = f�√3 − x�
a. (f ∘ g)(2) 𝟏𝟏
=
√𝟑𝟑−𝐱𝐱
Solution:
We know that, (f ∘ g)(2) = f�g(2)�, so we b. (g ∘ f)(x)
4
first find g(2). Since g(x) = x−1,
4 4
Solution:
g(2) = 2−1 = 1
=4 We know that, (g ∘ f)(x) = g�f(x)�, so

So, 1
(g ∘ f)(x) = g�f(x)� = g � �
x
1
(f ∘ g)(2) = f�g(2)� = f(4) = �3 − x
= 2(4) − 1 = 𝟕𝟕
x 1
= �3 ∙ x − x
b. (g ∘ f)(−3)
3x 1
= �x −x
Solution:
We know that, (g ∘ f)(−3) = g�f(−3)�, so =�
𝟑𝟑𝟑𝟑−𝟏𝟏
𝐱𝐱
we first find f(-3). Since f(x) = 2x – 1,

Marvin Kalngan, Civil Engineer, Plumbing Engineer, AutoCAD Expert, College Lecturer
Algebra – Mathematics of Engineering 55

Exponential and Logarithmic Solution:


1 −2
Functions �5� = 25
log1/5 25 = -2
1. Solve each equation.
3. Convert each logarithmic form to
a. 8x = 16. exponential form.

Solution: logarithmic form: y = loga x


8x = 16 exponential form: x = ay
(23)x = 24
23x = 24 a. log10 100,000 = 5
3x = 4
𝟒𝟒
x = 𝟑𝟑 Solution:
log10 100,000 = 5
b. 43x – 1 = 16x+2 100,000 = 105
1
Solution. b. log4 64 = -3
43x – 1 = 16x+2
(22)(3x – 1) = (24)(x + 2) Solution:
2(2)(3x - 1) = 2(4)(x + 2) 1
log4 64 = -3
(2)(3x - 1) = (4)(x + 2) 𝟏𝟏
6x - 2 = 4x + 8 = 4-3
𝟔𝟔𝟔𝟔
6x – 4x = 8 + 2
2x = 10 4. Solve each logarithmic equation.
x=5
a. log4 x = -2
x 1
c. 6 = 216
Solution:
Solution: log4 x = -2
1 1 𝟏𝟏
6x = 216 x = 4-2 = 42 = 𝟏𝟏𝟏𝟏
1
6x = 63
b. log½ (3x + 1) = 2
6x = 6−3
x = -3 Solution:
log½ (3x + 1) = 2
2 x 9
d. �3� = 4 1 2
3x + 1 = �2�
1
3x + 1 = 4
Solution:
2 x 9 4(3x + 1) = 1
�3� = 4 12x + 4 = 1
2 x 4 −1 12x = -3
�3� = �9� 𝟏𝟏
−1 −1 x = − 𝟒𝟒
2 x 22 2 2 2 −2
�3� = �32 � = ��3� � = �3�
x= -2 c. logx 3 = 2

2. Convert each exponential form to Solution:


logarithmic form. logx 3 = 2
3 = x2
logarithmic form: y = loga x x = ±√3
exponential form: x = ay
Only the principal square root satisfies the
2 equation since the base must be a positive
a. 3 = 9
number. The solution set is �√3�.
Solution:
32 = 9 Answer: √𝟑𝟑
log3 9 = 2
3
d. log 49 √7 = x
1 −2
b. �5� = 25

Marvin Kalngan, Civil Engineer, Plumbing Engineer, AutoCAD Expert, College Lecturer
Algebra – Mathematics of Engineering 56
Solution: Solution:
3
log 49 √7 = x log10 (7 ∙ 9) = 𝐥𝐥𝐥𝐥𝐥𝐥 𝟏𝟏𝟏𝟏 𝟕𝟕 + 𝐥𝐥𝐥𝐥𝐥𝐥 𝟏𝟏𝟏𝟏 𝟗𝟗
3
49x = √7
(72 )x = (7)1⁄3 b. log 3 (3x)
72x = (7)1⁄3
1 Solution:
2x = 3 log 3 (3x) = log 3 3 + log 3 x = 𝟏𝟏 + 𝐥𝐥𝐥𝐥𝐥𝐥 𝟑𝟑 𝐱𝐱
𝟏𝟏
𝐱𝐱 = 𝟔𝟔
5. Evaluate each logarithm. c. log 7 8 + log 7 12

For any positive real number b, with b ≠ 1, Solution:


the following are true. log 7 8 + log 7 12 = log 7 (8 ∙ 12)
= 𝐥𝐥𝐥𝐥𝐥𝐥 𝟕𝟕 𝟗𝟗𝟗𝟗
log b b = 1 and log b 1 = 0
d. log 4 x 3
a. log10 10
Solution:
Solution: log 4 x 3 = log 4 (x ∙ x ∙ x)
𝐥𝐥𝐥𝐥𝐥𝐥 𝟏𝟏𝟏𝟏 𝟏𝟏𝟏𝟏 = 𝟏𝟏 = log 4 x + log 4 x + log 4 x
= 𝟑𝟑 𝐥𝐥𝐥𝐥𝐥𝐥 𝟒𝟒 𝐱𝐱
b. log 8 1
8. Use the quotient rule to rewrite each
Solution: logarithm. Assume x > 0.
𝐥𝐥𝐥𝐥𝐥𝐥 𝟖𝟖 𝟏𝟏 = 𝟎𝟎
Quotient Rule for Logarithms:
If x, y, and b are positive real numbers,
c. log √2 √2
where b ≠ 1, then the following is true.
Solution: x
log b = log b x − log b y
𝐥𝐥𝐥𝐥𝐥𝐥 √𝟐𝟐 √𝟐𝟐 = 𝟏𝟏 y

7
6. The function defined by a. log 4 9
f(x) = 27 + 1.105 log10 (x + 1)
approximates the barometric pressure in Solution:
inches of mercury at a distance of x miles 7
log 4 9 = 𝐥𝐥𝐥𝐥𝐥𝐥 𝟒𝟒 𝟕𝟕 − 𝐥𝐥𝐥𝐥𝐥𝐥 𝟒𝟒 𝟗𝟗
from the eye of a typical hurricane.
Approximate the pressure 9 miles from the 27
eye of the hurricane. b. log 3 5

Solution: Solution:
27
Let x = 9, find f(9). log 3 5 = log 3 33 − log 3 5
= 3log 3 3 − log 3 5
f(x) = 27 + 1.105 log10 (x + 1)
= 3(1) − log 3 5
f(9) = 27 + 1.105 log10 (9 + 1)
= 𝟑𝟑 − 𝐥𝐥𝐥𝐥𝐥𝐥 𝟑𝟑 𝟓𝟓
f(9) = 27 + 1.105 log10 10
f(9) = 27 + 1.105 (1)
c. log 5 6 − log 5 x
𝐟𝐟(𝟗𝟗) = 𝟐𝟐𝟐𝟐. 𝟏𝟏𝟏𝟏𝟏𝟏
Solution:
The pressure 9 miles from the eye of the 𝟔𝟔
hurricane is 28.105 inches. log 5 6 − log 5 x = 𝐥𝐥𝐥𝐥𝐥𝐥 𝟓𝟓 𝐱𝐱

7. Use the product rule to rewrite each 8. Use the power rule to rewrite each
logarithm. Assume x > 0. logarithm. Assume b > 0, x > 0 and b ≠ 1.

Product Rule for Logarithms: Power Rule for Logarithms:


If x, y, and b are positive real numbers, If x and b are positive real numbers, where
where b ≠ 1, then the following is true. b ≠ 1, and if r is any real number, then the
following is true.
log b xy = log b x + log b y
log b x r = rlog b x
a. log10 (7 ∙ 9)
a. log 5 42

Marvin Kalngan, Civil Engineer, Plumbing Engineer, AutoCAD Expert, College Lecturer
Algebra – Mathematics of Engineering 57
3
log 2 (x + 5) = 4
Solution: (x + 5)3 = 24
log 5 42 = 𝟐𝟐𝐥𝐥𝐥𝐥𝐥𝐥 𝟓𝟓 𝟒𝟒 (x + 5)3 = 16
Take the cube root of both sides.
b. log b √7 3 3
�(x + 5)3 = √16
3
x + 5 = √16
Solution: 3
𝟏𝟏 x = √16 – 5
log b √7 = log b 71⁄2 = 𝟐𝟐 𝐥𝐥𝐥𝐥𝐥𝐥 𝐛𝐛 𝟕𝟕 x = -2.48
5
d. log 2 √x 2 14. Solve log 2 (x + 1) − log 2 x = log 2 7.

Solution: Solution:
5 𝟐𝟐
log 2 √x 2 = log 2 x 2⁄5 = 𝟓𝟓 𝐥𝐥𝐥𝐥𝐥𝐥 𝟐𝟐 𝐱𝐱 log 2 (x + 1) − log 2 x = log 2 7
x+1
log 2 � x � = log 2 7
x+1
9. The altitude in meters that corresponds to
x
=7
an atmospheric pressure of x millibars is x + 1 = 7x
given by the logarithmic function defined by 1 = 7x – x
f(x) = 51600 - 7457 ln x 1 = 6x
Use this function to find the altitude when 𝟏𝟏
atmospheric pressure is 400 millibars. 𝟔𝟔
= 𝐱𝐱

Solution: 15. Solve log x + log (x - 21) = 2.


f(x) = 51600 - 7457 ln x
f(400) = 51600 - 7457 ln 400 Solution:
f(400) = 6922 meters log x + log (x - 21) = 2
log [x(x - 21)] = 2
10. Evaluate log8 60. log10 [x(x - 21)] = 2
x(x - 21) = 102
Solution: x2 – 21x = 100
Using the Change-of-Base-Rule, we have x2 – 21x – 100 = 0
log x (x - 25)(x + 4) = 0
log a x = logba
b x – 25 = 0 or x + 4 = 0
log10 60
log 8 60 = log10 8
= 𝟏𝟏. 𝟗𝟗𝟗𝟗 x = 25 or x = -4

The value -4 must be rejected as a solution


11. Solve 5x = 20.
since the logarithm of negative numbers are
not defined in the real numbers.
Solution:
5x = 20
Take the logarithm of both sides.
log 5x = log 20 Nonlinear System of Equations
x log 5 = log 20
log 20 1. Solve the system.
x = log 5 = 1.86
x2 + y2 = 9 (1)
2x – y = 3 (2)
12. Solve e0.12x = 10.

Solution:
Solution:
e0.12x = 10 We solve the linear equation (2) for one of
Take the natural logarithm of both sides. the two variables and then substitute the
ln e0.12x = ln 10 resulting expression into the nonlinear
0.12x ln e = ln 10 equation (1).
ln 10
0.12x = ln e 2x – y = 3
ln 10
0.12x = 1 2x – 3 = y
0.12x = ln 10
ln 10 Now substitute and simplify.
x = 0.12 = 19.19 x2 + y2 = 9
x2 + (2x – 3)2 = 9
13. Solve log 2 (x + 5)3 = 4. x2 + 4x2 – 12x + 9 = 9
5x2 – 12x = 0
Solution: x(5x - 12) = 0

Marvin Kalngan, Civil Engineer, Plumbing Engineer, AutoCAD Expert, College Lecturer
Algebra – Mathematics of Engineering 58
x = 0 or 5x – 12 = 0 The solution set of the system is
x = 0 or 5x = 12 𝟏𝟏 𝟒𝟒
12
��− , −𝟖𝟖� , � , 𝟑𝟑��.
𝟐𝟐 𝟑𝟑
x = 0 or x = 5
3. Solve the system.
If x = 0, then x2 + 2xy – y2 = 7 (1)
2x – 3 = y x2 – y2 = 3 (2)
2(0) – 3 = y
-3 = y Solution:
Subtracting equation (2) from equation (1),
12
If x = 5 , then gives
2x – 3 = y 2xy = 4
12
2� 5 � – 3 = y We solve 2xy = 4 for any of the variables.
24
–3=y We choose y.
5
9 2xy = 4
5
=y 2
y=x
The solution set of the system is 2
𝟏𝟏𝟏𝟏 𝟗𝟗 Now, we substitute y = x into one of the
�(𝟎𝟎, −𝟑𝟑), � , ��.
𝟓𝟓 𝟓𝟓
original equations. It is easier to substitute in
equation (2).
2. Solve the system.
6x – y = 5 (1)
x2 – y2 = 3
xy = 4 (2)
2 2
x2 – �x� = 3
Solution: 4
x2 - x2 = 3
Solve xy = 4 for x.
xy = 4
4 Multiply both sides by x2 to remove
x=y
denominators.
4
4
x 2 �x 2 − x2 = 3�
Substitute y for x in equation (1).
x4 – 4 = 3x2
6x – y = 5 x4 – 3x2 – 4 = 0
4
6�y� – y = 5
24 Factor.
y
–y=5 (x2 - 4)(x2 + 1) = 0

Multiply both sides by y to remove Zero-factor property.


denominators. x2 – 4 = 0 or x2 + 1 = 0
24
y � y − y = 5�
Solve each equation.
24 – y2 = 5y x2 = 4 or x2 = -1
y2 + 5y – 24 = 0 x = 2 or x = -2 or x = i or x = -i

Factor. 2
Substituting these four values into y = x
(y + 8)(y - 3) = 0
gives the corresponding values for y.
Zero-factor property.
y + 8 = 0 or y – 3 = 0 If x = 2, then y = 1.
If x = -2, then y = -1.
2 2 2 −i −2i
Solve each equation. If x = i, then y = x = i = i ∙ −i = 1 = -2i
y = -8 or y = 3 2 2 2 i 2i
If x = -i, then y = x = −i = −i ∙ i = 1
= 2i
If y = -8, then
4 4 1 The solution set is
x = y = −8 = − 2 {(𝟐𝟐, 𝟏𝟏), (−𝟐𝟐, −𝟏𝟏), (𝐢𝐢, −𝟐𝟐𝟐𝟐), (−𝐢𝐢, 𝟐𝟐𝟐𝟐)}.

If y = 3, then
4 4
x=y=3 Sequences and Series

1. Given an infinite sequence with


1
an = n + n, find the following.
Marvin Kalngan, Civil Engineer, Plumbing Engineer, AutoCAD Expert, College Lecturer
Algebra – Mathematics of Engineering 59
Fourth Payment:
a. second term of the sequence payment: 500 + 0.01(3500) = $535
balance: 5000 – 4(500) = $3000
Solution: The payments for the first four months are
Replace n with 2. $550, $545, $540, $535.
1
an = n + n
1 𝟓𝟓 The remaining debt at the end of the period
a2 = 2 + 2 = 𝟐𝟐
is $3000.
4. Evaluate the series.
b. tenth term of the sequence
a. ∑6i=1(3i + 2)
Solution:
Solution:
Replace n with 10. ∑6i=1(3i + 2)
1
an = n + n = (3 ∙ 1 + 2) + (3 ∙ 2 + 2) + (3 ∙ 3 + 2)
1 𝟏𝟏𝟏𝟏𝟏𝟏
a10 = 10 + 10 = +(3 ∙ 4 + 2) + (3 ∙ 5 + 2) + (3 ∙ 6 + 2)
𝟏𝟏𝟏𝟏
= 5 + 8 + 11 + 14 + 17 + 20
= 75
c. 12th term of the sequence
b. ∑5i=1(i − 4)
Solution:
Replace n with 12. Solution:
an = n + n
1 ∑5i=1(i − 4)
1 𝟏𝟏𝟏𝟏𝟏𝟏 = (1 - 4) + (2 - 4) + (3 - 4) + (4 - 4) + (5 - 4)
a12 = 12 + 12 = 𝟏𝟏𝟏𝟏 = -3 + (-2) + (-1) + 0 + 1
= -3 – 2 – 1 + 1
2. Determine a general term an of the = -5
sequence.
5, 10, 15, 20, 25, … c. ∑7i=3 3i2

Solution: Solution:
Notice that the terms are all multiples of 5. ∑7i=3 3i2
The first term is 5(1), the second is 5(2), and = 3(32) + 3(42) + 3(52) + 3(62) + 3(72)
so on. The general term is = 27 + 48 + 75 + 108 + 147
an = 5n = 405

3. Hakeem Olajuwon borrows $5000 and 5. Write each sum with summation notation.
agrees to pay $500 monthly, plus interest of
1% on the unpaid balance from the a. 2 + 5 + 8 + 11
beginning of the first month. Find the
payments for the first four months and the Solution:
remaining debt at the end of that period. First, find a general term an that will give
these four terms for a1, a2, a3, and a4,
Solution: respectively. Each term is 1 less than a
The payments and remaining balances are multiple of 3, so try 3i – 1 as the general
calculated as follows. term.
3(1) – 1 = 2
First Payment. 3(2) – 1 = 5
payment: 500 + 0.01(5000) = $550 3(3) – 1 = 8
balance: 5000 – 500 = $4500 3(4) – 1 = 11

Second Payment: 2 + 5 + 8 + 11 = ∑𝟒𝟒𝐢𝐢=𝟏𝟏(𝟑𝟑𝟑𝟑 − 𝟏𝟏)


payment: 500 + 0.01(4500) = $545
balance: 5000 – 2(500) = $4000 b. 8 + 27 + 64 + 125 + 216

Third Payment: Solution:


payment: 500 + 0.01(4000) = $540 These numbers are the cubes of 2, 3, 4, 5,
balance: 5000 – 3(500) = $3500 and 6, so the general term is i3.

8 + 27 + 64 + 125 + 216 = ∑𝟔𝟔𝐢𝐢=𝟐𝟐 𝐢𝐢𝟑𝟑

Marvin Kalngan, Civil Engineer, Plumbing Engineer, AutoCAD Expert, College Lecturer
Algebra – Mathematics of Engineering 60
a19 = 61
6. Determine the general term of the
arithmetic sequence. 9. Evaluate the number of terms in the
-9, -6, -3, 0, 3, 6, … arithmetic sequence.
-8, -2, 4, 10, … , 52
Solution:
d = -6 – (-9) = -3 – (-6) = 3 Solution:
d = 10 – 4 = 4 – (-2) = -2 – (-8) = 6
The nth term of an arithmetic sequence is an = am + (n - m)d
given by the equation an = a1 + (n - 1)(6)
an = a1 + (n - 1)d 52 = -8 + (n - 1)(6)
52 = -8 + 6n – 6
Now, find an 52 = 6n – 14
an = a1 + (n - 1)d 66 = 6n
an = -9 + (n - 1)(3) 11 = n
an = -9 + 3n – 3
an = 3n – 12 10. Evaluate the sum of the first five terms
of the arithmetic sequence in which
7. Thomas Becket’s uncle decides to start a an = 2n – 5.
fund for Thomas education. He makes an
initial contribution of ₱3000 and deposits an Solution:
additional ₱500 each month. Thus, after one Begin by evaluating a1 and a5.
month the fund will have ₱3000 + ₱500 = an = 2n – 5
₱3500. How much will it have after 24 a1 = 2(1) – 5 = -3
months? (Disregard any interest.)
a5 = 2(5) – 5 = 5
Solution:
n
After n months, the fund will contain Sn = 2 (a1 + an )
an = 3000 + 500n 5
a24 = 3000 + 500(24) S5 = 2 (a1 + a5 )
5
a24 = ₱15000 S5 = 2 (−3 + 5)
5
8. Evaluate the indicated term for each S5 = 2 (2)
arithmetic sequence. S5 = 5

a. Given: a1 = 21, d = -3 11. Evaluate the sum of the first 8 terms of


Required: a22 the arithmetic sequence having first term of
3 and common difference of -2.
Solution:
an = am + (n - m)d Solution:
n
a22 = a1 + (22 - 1)(-3) Sn = [2a1 + (n − 1)d]
2
a22 = 21 + (21)(-3) 8
S8 = 2 [2 ∙ 3 + (8 − 1)(−2)]
a22 = -42
S8 = -32
b. Given: a7 = 25, a12 = 40
Required: a19 12. The 3rd and 6th terms of an arithmetic
progression are 94 and 85 respectively. Find
Solution: the value of the 15th term.
an = am + (n - m)d
a12 = a7 + (12 - 7)d Solution:
40 = 25 + 5d a3 = 94
15 = 5d a6 = 85
3=d a15 = ?

a19 = a7 + (19 - 7)d an = am + (n - m)d


a19 = 25 + (12)(3) a6 = a3 + (6 - 3)d
a19 = 61 85 = 94 + 3d
-9 = 3d
or -3 = d

a19 = a12 + (19 - 12)d an = am + (n - m)d


a19 = 40 + (7)(3) a15 = a3 + (15 - 3)(-3)

Marvin Kalngan, Civil Engineer, Plumbing Engineer, AutoCAD Expert, College Lecturer
Algebra – Mathematics of Engineering 61
a15 = 94 + 12(-3)
a15 = 58 Substitute,
an = amrn-m
or an = a1rn-1
𝟏𝟏 𝐧𝐧−𝟏𝟏
an = (15)�𝟐𝟐�
an = am + (n - m)d
a15 = a6 + (15 - 6)(-3)
a15 = 85 + 9(-3) 16. Evaluate the indicated term for each
a15 = 58 geometric sequence.

13. How many terms of the progression 3, 5, a. Given: a1 = 4


7, 9, … must be taken in order that their sum r = -3
will be 2600? Required: a6

Solution: Solution:
d=9–7=7–5=5–3=2 The nth term of a geometric sequence is
given by the equation
n an = amrn-m
Sn = 2 [2a1 + (n − 1)d]
n
2600 = 2 [2(3) + (n − 1)(2)] Substitute,
5200 = n(6 + 2n – 2) an = amrn-m
5200 = n(4 + 2n) an = a1rn-1
5200 = 4n + 2n2 a6 = (4)(-3)6-1
2n2 + 4n – 5200 = 0 a6 = (4)(-3)5
n2 + 2n – 2600 = 0 a6 = -972
(n + 52)(n - 50) = 0
n + 52 = 0 or n – 50 = 0 3 3 3
b. Given: 4 , 8 , 16 , …
n = -52 or n = 50
Required: a7
answer: n = 50
Solution:
3 3
a3 a2 16 8 1
14. Determine the common ratio of the r=a =a = 3 = 3 =2
2 1
geometric sequence. 8 4
15 15 15
15, 2 , 4 , 8 , …
The nth term of a geometric sequence is
given by the equation
Solution: an = amrn-m
15
a4 15 4 𝟏𝟏
r=a = 8
15 = 8
∙ 15 = 𝟐𝟐
3
4 Substitute,
an = amrn-m
or an = a1rn-1
3 1 6
15 a7 = �4� �2�
a3 15 2 𝟏𝟏
r= = 4
15 = ∙ 15 = 𝟐𝟐 𝟑𝟑
a7 = 𝟐𝟐𝟐𝟐𝟐𝟐
a2 4
2

or 17. Write the first five terms of the


geometric sequence whose first term is 5
15 1
a2 15 1 𝟏𝟏 and whose common ratio is 2.
r=a = 2
= ∙ 15 = 𝟐𝟐
1 15 2

Solution:
15. Determine the general term of the a1 = 5
sequence. 1
15 15 15 r=2
15, 2 , 4 , 8 , …
an = amrn-m
Solution: an = a1rn-1
a1 = 15
1
r=2 1 5
a2 = a1r2-1 = a1r = 5 ∙ 2 = 2

The nth term of a geometric sequence is 1 2 1 5


given by the equation a3 = a1r3-1 = a1r2 = 5 �2� = 5 ∙ 4 = 4
an = amrn-m

Marvin Kalngan, Civil Engineer, Plumbing Engineer, AutoCAD Expert, College Lecturer
Algebra – Mathematics of Engineering 62
1 3 1 5
a4 = a1r4-1 = a1r3 = 5 �2� = 5 ∙ 8 = 8 1
a1 = 2
1
1 4 1 5 r=2
a5 = a1r5-1 = a1r4 = 5 �2� = 5 ∙ 16 = 16
1
𝟓𝟓 𝟓𝟓 𝟓𝟓 𝟓𝟓 1 a
answer: 𝟓𝟓, 𝟐𝟐 , 𝟒𝟒 , 𝟖𝟖 , 𝟏𝟏𝟏𝟏 S = 1−r = 2
1 = = 𝟏𝟏
1−
2
18. Evaluate the sum of the first six terms of 22. If the 4th term of a geometric progression
the geometric sequence with first term -2 with common ratio equal to half the initial
and common ratio 3. term is 32, what is the 15th term?

Solution: Solution:
a1 = -2 a4 = 32
r=3 a
r = 21
S6 = ?

a1 −a1 rn an = amrn-m
Sn = an = a1rn-1
1−r
−2−(−2)�36 � a4 = a1r4-1
S6 = 1−3 a4 = a1r3
−2−(−2)�36 � 1456 3
S6 = = a
1−3 −2 32 = a1� 21 �
S6 = -728 a1 3
32 = a1� �
23
19. Evaluate ∑4i=1 3 ∙2. i
32 = 81
a 4

Solution: 256 = a1 4
1
Since the series is in the form of ∑ni=1 a ∙ bi , Raising both sides to 4 gives
it represents the sum of the first n terms of 4 = a1
the geometric sequence with a1 = a ∙ b1 and
r = b. It follows that
a 4
r = 21 = 2 = 2
a1 −a1 rn
Sn = 1−r
6−6�24 � an = amrn-m
S4 = 1−2 an = a1rn-1
6−6�24 �
S4 = a15 = a1r15-1
1−2
−90 a15 = a1r14
S4 = −1 a15 = 4(214)
S4 = 90 a15 = 65536

Alternate solution: 23. The 3rd and 6th terms of a geometric


∑4i=1 3 ∙ 2i progression are 8 and 64 respectively. Find
= (3 ∙ 21 ) + (3 ∙ 22 ) + (3 ∙ 23 ) + (3 ∙ 24 ) the value of the 9th term.
= 6 + 12 + 24 + 48
= 90 Solution:
a3 = 8
20. Evaluate the sum of the terms of the a6 = 64
infinite geometric sequence with a1 = 3 and a9 = ?
1
r = − 3.
an = amrn-m
a6 = a3r6-3
Solution:
a 3 3 𝟗𝟗 a6 = a3r3
S= 1 = 1 = 1 = 𝟒𝟒 64 = 8r3
1−r 1−�− � 1+
3 3
8 = r3
1
1 i Raising both sides to 3 gives
21. Evaluate ∑∞
i=1 �2�
2=r
Solution: an = amrn-m
This is the infinite geometric series a9 = a3r9-3
1 1 1 2 1 3
�2� + �2� + �2� + ⋯ a9 = a3r6 = 8(26)
1 1 1 a9 = 512
2
+4+8+⋯

Marvin Kalngan, Civil Engineer, Plumbing Engineer, AutoCAD Expert, College Lecturer
Algebra – Mathematics of Engineering 63
or 6! 6∙5∙4∙3∙2∙1 6∙5
c. 4!2! = (4∙3∙2∙1)(2∙1)
= 2∙1 = 𝟏𝟏𝟏𝟏

an = amrn-m
a9 = a6r6-3 3. Evaluate each binomial coefficient.
a9 = a6r3 = 64(23)
a9 = 512
24. If the 10th term of a harmonic
progression is 21 and the 21st term is 10, a. 7C2
find the 210th term.
Solution:
n!
Solution: n Cr =
r!(n−r)!
Since harmonic progression is the reciprocal 7! 7!
7C2 = = 2!5! = 21
of arithmetic progression, convert all the 2!(7−2)!
terms to arithmetic progression then
compute for the 210th term. Then take the b. 5C4
reciprocal. So in arithmetic progression,
1 1 Solution:
a10 = 21 and a21 = 10 n!
n Cr =
r!(n−r)!
5! 5!
The nth term of an arithmetic progression is 5C4 = = 4!1! = 5
4!(5−4)!
given by the equation
an = am + (n - m)d
c. 6C3
Substituting, we have
Solution:
a21 = a10 + (21 - 10)d n!
1 1 n Cr =
= + 11d r!(n−r)!
10 21 6! 6!
1 6C3 = = 3!3! = 5
d = 210 3!(6−3)!

Compute the 210th of the arithmetic 4. Expand (2m + 3)4


progression.
an = am + (n - m)d Solution:
1 For any positive integer n,
a210 = a21 + (210 - 21) �210�
(x + y)n
1 1 n! n!
a210 = 10 + (210 - 21) �210� = xn + 1!(n−1)! x n−1 y + 2!(n−2)! x n−2 y 2 + … +
a210 = 1 n!
(n−1)!1!
xy n−1 + yn
Since the reciprocal of 1 is 1, the 210th term
of the harmonic progression is 1. (2m + 3)4
4! 4!
= (2m)4 + 1!3! (2m)3 (3) + 2!2! (2m)2 (3)2
answer: 1 4!
+ 3!1! (2m)(3)3 + 34
= 16m4 + 96m3 + 216m2 + 216m + 81
The Binomial Theorem
b 5
5. Expand �a − 2�
1. Evaluate each factorial.
Solution:
a. 4! = 4 ∙ 3 ∙ 2 ∙ 1 = 24 For any positive integer n,
(x + y)n
b. 7! = 7 ∙ 6 ∙ 5 ∙ 4 ∙ 3 ∙ 2 ∙ 1 = 5040 n! n!
= xn + 1!(n−1)! x n−1 y + 2!(n−2)! x n−2 y 2 + … +
n!
c. 0! = 1
(n−1)!1!
xy n−1 + yn

2. Find the value of each expression. b 5


�a − 2�
8! 8∙7∙6∙5∙4∙3∙2∙1 8∙7
a. 6!2! = (6∙5∙4∙3∙2∙1)(2∙1) = 2∙1 = 𝟐𝟐𝟐𝟐 5! b 5! b 2
= a5 + 1!4! a4 �− 2� + 2!3! a3 �− 2�
5! b 3 5! b 4 b 5
8!
b. 5!3! =
8∙7∙6∙5∙4∙3∙2∙1 8∙7∙6
= 3∙2∙1 = 𝟓𝟓𝟓𝟓 + 3!2! a2 �− 2� + 4!1! a �− 2� + �− 2�
(5∙4∙3∙2∙1)(3∙2∙1)
𝟓𝟓 𝟓𝟓 𝟓𝟓 𝟓𝟓 𝐛𝐛𝟓𝟓
= a5 - 𝟐𝟐 𝐚𝐚𝟒𝟒 𝐛𝐛 + 𝟐𝟐 𝐚𝐚𝟑𝟑 𝐛𝐛𝟐𝟐 - 𝟒𝟒 𝐚𝐚𝟐𝟐 𝐛𝐛𝟑𝟑 + 𝟏𝟏𝟏𝟏 𝐚𝐚𝐛𝐛𝟒𝟒 -
𝟑𝟑𝟑𝟑

6. Find the 4th term of the expansion


Marvin Kalngan, Civil Engineer, Plumbing Engineer, AutoCAD Expert, College Lecturer
Algebra – Mathematics of Engineering 64
10
(a + 2b) to fill the third spot, and 3 ways to fill the
fourth. Since he wants to use 4 books, there
Solution: are only 4 spots to be filled (4 events)
The rth term of the expansion of (x + y)n is instead of 6. There are
n!
(r−1)!(n−r+1)!
x n−r+1 y r−1 6(5)(4)(3) = 360 arrangements

3. Suppose 8 people enter an event in a


10!
swim meet. Assuming there are no ties, in
4th term = (4−1)!(10−4+1)! a10−4+1 (2b)4−1 how many ways could the gold, silver, and
10! bronze medals be awarded?
= 3!7! a7 (2b)3 = 960a7b3
Solution:
Using the Fundamental Principle of
Counting Theory Counting, there are 8 choices for the gold
medal winner, 7 remaining choices for the
1. A restaurant offers a choice of 3 salads, 5 silver, and 6 for the bronze, so there are
main dishes, and 2 desserts. Find the number 8·7·6 = 336 ways the three medals can be
of different 3-course meals that can be awarded to the 8 swimmers.
selected.
Alternate Solution:
Solution: Use the Permutation Formula.
Fundamental Principle of Counting. n!
nPr =
(n−r)!
If one event can occur in m ways and a 8! 8!
second event can occur in n ways, then both 8P3 = (8−3)! = 5! = 336 ways
events can occur in mn ways, provided the
outcome of the first event does not influence 4. In how many ways can 6 students be
the outcome of the second. seated in a row of 6 desks?

Three independent events are involved: Solution:


selecting a salad, selecting a main dish, and 6! = 6(5)(4)(3)(2)(1) = 720 ways
selecting a dessert. The first event can occur
in 3 ways, the second event can occur in 5 5. In how many ways can three letters of the
ways, and the third event can occur in 2 alphabet be arranged if a vowel cannot be
ways. There are used in the middle position, and repetitions
of the letters are allowed?
3(5)(2) = 30 possible meals
Solution:
2. Marvin Kalngan has 6 different books that There are 26 letters in the alphabet, 5 are
he wishes to arrange on his desk. vowels. In the first and third positions, we
a. How many different arrangements are can use any of the 26 letters of the alphabet,
possible? but in the middle position, we can use only
b. Suppose that Marvin wishes to place only one of 26 – 5 = 21 letters (since there are 5
4 of the 6 books on his desk. How many vowels). Using the Fundamental Counting
arrangements of 4 books are possible? Principle, there are
26(21)(26) = 14196 ways
Solution:
a. Six events are involved: selecting a book 6. How many different committees of 3
for the first spot, selecting a book for the people can be chosen from a group of 8
second spot, and so on. Here the outcome of people?
the first event does influence the outcome of
the other events (since one book has already Solution:
been chosen). For the first spot Marvin has 6 Because the order in which the members of
choices, for the second spot 5 choices, for the committee are chosen does not affect the
the third spot 4 choices, and so on. We use result, use combinations.
the fundamental principle of counting. There
are n!
n Cr = r!(n−r)!
8! 8!
6(5)(4)(3)(2)(1) = 720 different 8C3 = 3!(8−3)! = 3!5! = 56 committees
arrangements
7. From a group of 30 bank employees, 3 are
b. Marvin still has 6 ways to fill the first to be selected to work on a special project.
spot, 5 ways to fill the second spot, 4 ways a. In how many different ways can the
Marvin Kalngan, Civil Engineer, Plumbing Engineer, AutoCAD Expert, College Lecturer
Algebra – Mathematics of Engineering 65
employees be selected?
b. In how many different ways can the group
of 3 be selected if it has already been
decided that a certain employee must work
on the project?

Solution:
a. The number of 3-element combinations
from a set of 30 elements must be found.
Use combinations because order within the
group of 3 does not affect the result.
n!
n Cr =
r!(n−r)!
30! 30!
30C3 = 3!(30−3)! = 3!27! = 4060 ways

b. Since one employee has already been


selected to work on the project, the problem
is reduced to selecting 2 more employees
from the 29 employees that are left.
29! 29!
29C2 = = = 406 ways
2!(29−2)! 2!27!

8. A congressional committee consists of 4


senators and 6 representatives. A delegation
of 5 members is to be chosen. In how many
ways could this delegation include exactly 3
senators?

Solution:
“Exactly 3 senators” implies that there must
be 5 – 3 = 2 representatives as well. The 3
senators could be chosen in 4C3 = 4 ways.
The 2 representatives could be chosen in
6C2 = 15 ways. We apply the fundamental
principle of counting, so there are
4(15) = 60 ways

9. An HR officer of a company will hire 4


Civil Engineers from 8 applicants, 4
Mechanical Engineers from 9 applicants, 2
Plumbing Engineers from 4 applicants and 3
Electrical Engineers from 7 applicants. In
how many ways can he do this?

Solution:
no. of ways = (8C4)(9C4)(4C2)(7C3)
= 70(126)(6)(35)
= 1852200 ways

Marvin Kalngan, Civil Engineer, Plumbing Engineer, AutoCAD Expert, College Lecturer

Вам также может понравиться